IM COMAT

अब Quizwiz के साथ अपने होमवर्क और परीक्षाओं को एस करें!

Uremia/ Hyperkalemia/pericarditis

The correct answer is: C This patient is presenting with uremic pericarditis secondary to azotemia as a result of chronic kidney disease (CKD). Signs and symptoms suggestive of acute pericarditis include episodic chest pain relieved by sitting forward, cardiac friction rub, and diffuse ST segment elevation on ECG (characteristic of the early stages of pericarditis). Mild to moderate pleural and/or pericardial effusions may be present due to fluid accumulation within the pleural space or pericardial sac. The laboratory values provided suggest that this patient's renal disease has progressed from stage 3 CKD to end-stage renal disease (ESRD). Renal function is determined by calculating creatinine clearance (CrCl), a measurement used to estimate glomerular filtration rate (GFR). The equation commonly used to calculate CrCl is [140-age x wt (kg)]/ (72 x plasma creatinine). This patient's GFR was approximately 33 mL/min six months ago (stage 3: GFR= 30-59 mL/min/1.73 m2) and now is 12.9 mL/min (ESRD: GFR< 15 mL/min/1.73 m2 ). Patients with chronic kidney disease typically do not develop clinical symptoms associated with metabolic derangements and electrolytes imbalances until GFR is < 30 mL/min. Common abnormalities that may be present include uremia, fluid retention, metabolic acidosis, hyperkalemia, anemia, hypocalcemia, and hyperphosphatemia. Azotemia results from decreased renal excretion of nitrogenous waste products such as urea and creatinine. Uremia can manifest clinically in several different ways because it disrupts normal functioning of almost every organ system (e.g., endocrine, neuromuscular, cardiovascular, pulmonary, dermatology, gastrointestinal, hematology, and immunology). Uremic pericarditis is seen in advanced uremia and used to be a common complication and cause of death in patients with chronic kidney disease. However, with the discovery and advancements in dialysis, it is less common than it once was (incidence rate dropped from approximately 35-50% to < 10%). Uremic pericarditis is an absolute indication for the urgent initiation of dialysis or for intensification of dialysis in patients already receiving dialysis. Immediate dialysis can be performed using a temporary double-lumen hemodialysis catheter placed in either the internal jugular or femoral vein. In most patients, relief of chest pain and reduction in the size of any effusion occurs within 1-2 weeks. Dialysis will also correct this patient's mild hyperkalemia without the need for additional treatment (e.g., bicarbonate, insulin, glucose etc.). Placement of permanent vascular access via either an arterio-venous fistula or graft is necessary for long-term management once the patient's symptoms and current metabolic and electrolyte abnormalities have been properly treated. Remember that osteopathic findings can often be helpful while answering questions on the COMLEX. Additionally, in the clinical setting it is important to recognize associated somatic dysfunction that may need to treated at some point in the management of your patient. In this example you see acute TART changes throughout the upper thoracic spine (T1-4) which are likely due to viscerosomatic reflex from the underlying cardiac pathology. Although not presented in the vignette above, you may also see changes associated with the underlying renal pathology, which would cause TART changes from T6-9 and may also have associated Chapman points or other somatic dysfunctions. Answer A: Indications for urgent treatment of hyperkalemia include clinical evidence of hyperkalemia (nausea, vomiting, areflexia, weakness), potassium levels > 6.5 mEq/L, and/or the presence of associated ECG changes ( peaked T waves, wide QRS, PR prolongation, loss of P waves). Treatment includes stabilization of the myocardium with calcium gluconate and reduction of serum potassium using bicarbonate, insulin and glucose, beta-agonists (e.g., albuterol), kayexalate, loop diuretics, and/or dialysis. Immediate treatment of hyperkalemia is not necessary in this patient because his serum potassium is not > 6.5 mEq/L, he is asymptomatic, and he is not exhibiting the classic ECG findings seen with hyperkalemia. Also, hyperkalemia associated with ESRD is commonly refractory to pharmacological treatment options and usually only responds to hemodialysis. Therefore, it would not be appropriate to treat this patient with insulin and bicarbonate. Answer B: Pericardiocentesis involves inserting a needle (commonly through the 5th or 6th intercostal space at the left sternal border) into the pericardium to drain fluid within the pericardial sac. It is indicated for patients with cardiac tamponade or large chronic effusions resistant to dialysis. Pericardiocentesis should be performed immediately and takes precedence over dialysis if there is evidence of cardiac tamponade. Signs suggestive of tamponade include hypotension, elevated systemic venous pressure with jugular venous distention, and muffled heart sounds. This patient's presentation suggests pericarditis without tamponade physiology, therefore; dialysis is the most appropriate next step in management. Answer D: The presence of uremic symptoms is an accepted indication for initiating CKD patients on maintenance dialysis. An arterio-venous fistula is the preferred type of hemodialysis access and is created by anastomosis of an artery to a vein. The vein arterializes over several weeks and creates a high-flow system for adequate access to circulation. Fistulas require time to mature and should not be used for hemodialysis for approximately 8-12 weeks after placement. Uremic pericarditis requires urgent hemodialysis, therefore; AV fistula placement would not be appropriate at this time. However, this patient will require maintenance dialysis and permanent vascular access should be discussed after urgent dialysis is instituted. Answer E: Patients with end-stage renal disease are commonly volume overloaded because of decreased renal function leading to accumulation of ingested fluid. Signs of volume overload include pleural effusion, peripheral edema, and refractory hypertension. Administration of intravenous fluids without instituting hemodialysis will not address the pericardial process and will worsen the patient's fluid balance, placing more strain on the heart. Bottom Line: Indications for urgent dialysis in patients with CKD include pericarditis or pleuritic, severe metabolic acidosis, severe hyperkalemia (refractory to conservative treatment), progressive uremic encephalopathy, intractable volume overload (despite diuretic therapy), failure to thrive/ malnutrition, peripheral neuropathy, a bleeding diathesis, and intractable gastrointestinal symptoms (e.g., nausea, vomiting). After acute metabolic and electrolyte abnormalities are corrected, patients should be referred to a nephrologist to discuss maintenance dialysis, vascular access placement, and management of advanced CKD complications (e.g., anemia, acidosis, hyperparathyroidism).

Different Criteria

COMBANK Insight : The may be items regarding staging, diagnosing, and prognosis based on specific criteria. It is important to familiarize yourself with commonly tested medical criteria. These include, but are not limited to: NYHA functional classification for CHF TIMI risk score for unstable angina and NSTEMI Duke criteria for infectious endocarditis Duke's staging for colorectal cancer Child's classification to assess severity of liver disease Jones criteria for acute rheumatic fever Light's criteria for exudative pleural effusion

A 25-year-old male is admitted with a 20% total body surface area burn to his legs and chest after an explosion. Upon arrival to the emergency department he is obtunded with poor vital signs. Rapid sequence intubation is successfully performed for airway protection. Soon after the patient later suffers a cardiac arrest. The most likely etiology is A. asphyxiation B. blunt cardiac injury C. hyperkalemia D.hypoxia E. myocardial infarction

The correct answer is: C Depolarizing neuromuscular blockers mimic acetylcholine (ACh) by binding ACh receptors of the motor end plate and causing depolarization of the muscle, which is seen clinically as muscle fasciculations. Succinylcholine, the only depolarizing neuromuscular blocker available for use, is degraded by plasma pseudocholinesterase. In patients with a deficiency of pseudocholinesterase, prolonged effects can occur. Side effects of succinylcholine include muscle pain, rhabdomyolysis, ocular hypertension, malignant hyperthermia, and hyperkalemia. Patients with spinal cord injuries, large burns, upper and lower motor neuron disease, renal failure, or prolonged immobility are at particular risk for hyperkalemia and resultant cardiac dysrhythmias. Answers A & D: Severe burns and environmental smoke may constrict the bronchioles or tighten the chest to the point of limiting respiratory excursion, however, this is an unlikely cause of cardiac arrest. Additionally, the patient has been successfully intubated and should be aerating well at this point in time. Answers B & E: Severe thermal injury is associated with cardiac contractile dysfunction and elevated cardiac troponin, especially if affecting >25% of body surface area. A tissue embolus causing a myocardial infarction in the setting of burns is a rare occurrence. However, this is an unlikely cause of a cardiac dysrhythmia in the setting of this patient. Bottom Line: Succinylcholine can cause hyperkalemia in patients with spinal cord injuries, large burns, renal failure, or prolonged immobility.

A 78-year-old male presents to the office for a routine check up. The patient has no complaints at this time. He is current on his colonoscopy and other preventative medical care. Past medical history is pertinent for hypertension, dyslipidemia, and obesity. Physical examination of the cardiovascular system reveals an irregularly irregular rhythm. This patient's risk for a cardioembolic stroke is best stated as having a CHADS2VASc score of A. 1 B.2 C. 3 D. 4 E. 5

The correct answer is: C The CHADS2VASc score is used for prevention of cardioembolic stroke. Cardioembolism is approximately 20% of all ischemic strokes. The embolism is from thrombotic material forming on the atrial or ventricular wall or left heart valves. These thrombi detach and embolize into the arterial circulation. The thrombus may lyse quickly and produce a transient ischemic accident (TIA) or if the arterial occlusion is longer it produces an ischemic stroke. Non-rheumatic atrial fibrillation is the most common cause of cerebral embolism overall. Patients with atrial fibrillation have an average annual risk of stroke of approximately 5%. This can be calculated by using the CHADS2VASc score: C congestive heart failure 1 point H hypertension 1 point An age greater than 75-years-old 2 point D diabetes mellitus 1 point S stroke or TIA 2 points (hence CHADS2) V vascular disease 1 point An age 65-74 1 point Sc (sex category) 1 point if female The CHADS2VASc score is utilized to determine the treatment plan with patients that have atrial fibrillation. The score breakdown regarding treatment us as follows: 0 aspirin or no antithrombotic 1 aspirin or anticoagulation (warfarin) 2+ These patients are typically put on heparin and bridged with warfarin to an INR of 2-3.

A 56-year-old G2P2 female presents for a routine wellness examination. Past medical history reveals age of menarche 13 years, with last regular menstrual period occurring 5 years ago. She had 3 months of occasional, but diminishing menses until she finally ceased altogether. Social history is positive for a 25 pack year smoking history. Family history reveals the patient's mother suffered a femoral neck fracture after a minor trauma. Vital signs reveal a blood pressure of 148/99 mmHg. At the previous visit, the patient's blood pressure was 145/92 mmHg and a weight of 50 kg (110 lbs). If started, which of the following antihypertensive medications will also delay bone demineralization in this patient? A. amlodipine B. furosemide C. hydrochlorithiazide D. lisinopril E. prazosin

The correct answer is: C This patient has several risk factors for osteoporosis, namely a positive family history, tobacco use, body weight less than 58 kg, and postmenopausal status. Other risk factors include alcohol use, history of fractures, and glucocorticoid administration. Thiazide diuretics, such as hydrochlorithiazide, inhibit sodium and chloride resorption in the distal convoluted tubule, which leads to the osmotic force driving their diuretic effect. However, they stimulate distal tubular resorption of calcium. The decrease in net calcium urinary excretion has beneficial effect on bone mineral density and other conditions, such as recurrent nephrolithiasis or other issues of hypercalciuria. Thiazide diuretics are generally well tolerated and commonly used as first-line therapy in newly diagnosed hypertension. Osteoporosis can be a serious medical threat to the well-being of patients and attention should be paid to associated risk factors. Age-appropriate screening with DEXA scanning and FRAX calculation is recommended. Answer A: Dihydropyridine calcium channel blockers, such as amlodipine, have essentially no effect on the demineralization of bone. The mechanism of action is inhibiting the influx of calcium into vascular smooth muscle and the myocardium. One of the most common side effects is peripheral edema. Answer B: Loop diuretics, such as furosemide, can lead to hypocalcemia by inhibiting calcium reabsorption in the loop of Henle. Answer D: ACE inhibitors, such as lisinopril, work by blocking angiotensin converting enzyme and thus preventing the conversion of angiotensin I to angiotensin II. They are known for their teratogenic effects, cough, and hyperkalemia. They have no significant effect of bone demineralization. Answer E: Alpha blockers, such as prazosin, antagonize alpha-1 adrenergic receptors in vascular smooth muscle. This results in vascular dilatation. This also provides benefit in the setting of BPH. There is no significant effect on bone demineralization. Bottom Line: Thiazide diuretics promote calcium resorption, thus preserve serum calcium concentrations, which in turn decreases the release of PTH.

A 53-year-old female presents to the hospital with right upper quadrant abdominal pain. Structural examination reveals tissue texture changes from T5-T10 on the left. During the workup, a hepatitis panel is obtained and reveals the following: HBsAg negative Anti-HBs antibody positive HBeAg negative Anti-HBe antibody negative Which of the following is most consistent with this pattern? A. acute infection with hepatitis B B. chronic infection with hepatitis B C. previous immunization to hepatitis B D. previous infection and immunization to hepatitis B E. previous infection with hepatitis B

The correct answer is: C This person has received the hepatitis B vaccine. Hepatitis B surface antigen (HBsAg) is the hallmark of the diagnosis of HBV infection and appears in the serum from 1 to 10 weeks after infection. HBsAg usually disappears in 4 to 6 months, but persistence in the serum implies chronic infection. Anti-HBs antibodies usually appear during a window period after the disappearance of HBsAg and mark recovery after HBV infection. Anti-HBs antibodies are also induced by the HBV vaccine. The hepatitis core antigen (HBcAg) is an intracellular antigen that is not detectable in serum. Anti-HBc antibodies, however, are detectable early after infection and persist after recovery and in chronic infections. Hepatitis B e antigen (HBeAg) is a secretory protein that is a marker of HBV replication and infectivity. It is usually present early and may persist for years in chronic infection, but it usually disappears within months in the absence of chronic infection. Quantification of HBV DNA in the serum has now become the most accurate way of assessing HBV activity. Answer A: An acute infection with hepatitis B would likely show a positive HBeAg and HBsAg, and there may be HBcAb present. Answer B: A chronic infection with hepatitis B would likely show HBcAb and may show HBeAg and/or anti-HBeAb. The HBsAg may or may not be present, as it can disappear later in infection. Hep B DNA would also likely be present. Answer D: Individuals who have developed immunity by virtue of prior infection and clearance of the hepatitis B virus will have positive HBsAb and IgG antibody to HBcAg (in other words, a positive HBcAb). Answer E: An individual that was previously infected with hepatitis B would likely have HBsAg (if fairly recent) and/or HBeAg. The presence of HBcAb and/or HBsAb would imply subsequent immunization. Bottom Line: Hepatitis B immunization will result in positive Anti-HBs antibodies but will not have antibodies to HBc or HBe, both of which will be positive with a previous or active infection of hepatitis B. Chronic hepatitis B infection will also have positive HBsAg.

A 48-year-old woman was hospitalized for progressive biventricular failure and eventually expired from a severe low cardiac output state. History reveals that she had breast cancer 16 years ago and was treated with surgery and non-mediastinal radiotherapy. A right axillary ulcer was noted ten years later, and chemotherapy was started. Which of the following is the most likely chemotherapeutic agent provided at that time? A. bleomycin B. cisplatin C. cyclophosphamide D. doxorubicin E.vincristine

The correct answer is: D Doxorubicin is a drug used in cancer chemotherapy. It is an anthracycline antibiotic which works by intercalating DNA. Acute doxorubicin cardiotoxicity is reversible, and clinical signs include tachycardia, hypotension, electrocardiogram changes, and arrhythmias. When the cumulative dose of doxorubicin reaches 550 mg/m², the risks of developing cardiac side effects, including congestive heart failure (CHF), dilated cardiomyopathy, and death, dramatically increase. Answer A: Bleomycin is a glycopeptide antibiotic that inhibits DNA, RNA, protein synthesis in G2, and M phases. It is used extensively in the treatment of germ cell tumors and can cause interstitial pulmonary fibrosis. Answer B: Cisplatin works as a platinum coordination compound that inhibits DNA synthesis; it cross-links and denatures strands of DNA thereby disrupting DNA function by covalently binding to DNA bases. It can also produce DNA intrastrand cross-linking and breakage. Side effects include nephrotoxicity, neurotoxicity, and ototoxicity. Answer C: Cyclophosphamide interferes with malignant cell growth by cross-linking tumor cell DNA. It does not have specificity for any phase of the cell cycle and also has potent immunosuppressive activity. Side effects include hemorrhagic cystitis and syndrome of inappropriate anti-diuretic hormone (SIADH). Answer E: Vincristine is a vinca alkaloid that acts in M & S phases by inhibiting microtubule formation and inhibiting DNA/RNA synthesis. Adverse effects include peripheral neuropathy.

A 4-year-old male presents with increased irritability, abdominal pain, and lethargy. History reveals that he lives in an older house and is often discovered eating paint chips. Physical examination reveals pale conjunctiva, 1+ diminished patellar reflexes bilaterally, and 3/5 weakness of forearm extensor muscles bilaterally. Serum toxin levels are obtained. Which of the following additional studies should be performed in this patient? A. capillary blood toxin levels B. electromyography C. MRI of the brain D. plain film radiography of the abdomen E. radiographs of long bones

The correct answer is: D Lead poisoning is the most common environmental illness in children in the United States. Most exposure in children is due to lead-contaminated household dust from leaded paints. Most adult cases of lead poisoning are due to occupational exposure (employees in storage battery manufacturing and lead and zinc ore mining industries). Symptoms are usually subtle and may include increased irritability, headaches, hyperactivity or lethargy, intermittent abdominal pain, vomiting, and/or peripheral neuropathy. Physical examination may show signs of iron deficiency anemia (common in children with lead poisoning), and neurological deficits (diminished deep tendon reflexes, weak extensor muscles). Whole blood lead levels (BLL) should be obtained in all patients with suspected lead poisoning. Then, a plain x-ray of the abdomen should be performed in all children to detect lead-containing objects. Retained lead objects in the GI tract are considered a medical emergency and require referral for possible removal. Further management depends on the patient's BLL level. Environmental investigation and follow-up testing is required for children with BLLs of 10 µg/dL or higher. Chelation therapy with either oral succimer (DMSA) or IV ethylenediaminetetraacetic acid (EDTA) is warranted in patients with BLLs greater than 45 µg/dL. Answer A: Capillary or fingerstick blood level is the method used for lead poisoning screening. The Center for Disease Control (CDC) recommends blood lead screening at 12 and 24 months for all patients living in high-risk area, which includes houses constructed before the 1950's or in zip codes with high lead level percentages. Capillary blood samples have a high false-positive rate. Therefore, all positive results must be confirmed with venous blood sampling. In a patient with known exposure who is exhibiting symptoms a fingerstick blood level is not appropriate. Answer B: Electromyography (EMG) measures the electrical impulses of muscles at rest and during contraction to detect abnormalities in nerve conduction. Lead poisoning can cause motor neuron damage leading to peripheral neuropathies, such as wrist or foot drop. Neurological deficits, such as diminished deep tendon reflexes and/or decreased muscle strength, may be present on examination. EMG may help to diagnose a peripheral neuropathy. However, it is not necessary in these patients because it does not help in making the diagnosis of lead poisoning. Answer C: MRI of the brain is not routinely included in the diagnostic work-up of lead poisoning. However, it may be performed in patients presenting with acute encephalopathy. Acute encephalopathy can be seen in patients with BLL levels >70 µg/dL. They will present with vomiting, confusion, seizures, and possibly coma. MRI of the brain may show cerebral edema and microhemorrhages. Answer E: Radiographs of long bones in children with chronic exposure to moderate lead levels may show characteristic lead lines (radiodensity in the distal metaphyseal plate that indicates growth arrest). This finding is associated with BLLs > 40 µg/dL over an extended period of time. Lead lines are rarely found in most cases of lead poisoning. Therefore, the CDC does not recommend obtaining long bone radiographs. Bottom Line: The initial step in the work-up of suspected lead poisoning is to obtain a whole blood lead level (BLL). A plain film radiograph of the abdomen is recommended in children to detect retained lead objects that may require surgical intervention for removal.

Arrythmias

Ventricular fibrillation is characterized by irregular, random waveform with no discernible p waves or QRS complexes. Fibrillation refers to multiple foci in the ventricles that fire rapidly resulting in no clear P or QRS complex resulting in no cardiac output; therefore, requiring immediate defibrillation and CPR. Most episodes are precipitated by an ischemic event or use of anti-arrhythmic drugs. Often initial rhythms include atrial fibrillation with RVR or ventricular tachycardia which transforms into ventricular fibrillation. This patient's EKG with a wandering baselines and irregular QRS complexes is consistent with ventricular fibrillation and CPR should be started immediately. Answer A: Atrial fibrillation refers to fibrillation of the atria resulting in multiple foci in the atria firing continuously resulting in no clear p waves. The atrial rate is over 400 beats per minute but most impulses are blocked at the AV node resulting in an irregular rhythm with rapid ventricular rate. Atrial fibrillation can present as paroxysmal, recurrent or permanent. Symptoms consist of palpitations, syncope, angina, dyspnea or even asymptomatic. Physical examination consists of irregular palpable pulse. Uncontrolled atrial fibrillation due to tachycardia can result in congestive heart failure. Additional main complication includes increased risk of thromboembolism due to left atrial dilation. Treatment consist of rate control, rhythm control and anti-coagulation (based on CHADS2 Score) depending on patient's hemodynamic stability and other co-morbidities. Answer B: Atrial flutter is a supraventricular rhythm secondary to irritable automaticity focus in the atria resulting in organized depolarization. Typical atrial rate can be 300 beats per minute resulting in more p waves than q waves. The electrocardiogram is remarkable for a saw tooth baseline due to every two or three flutter waves for every QRS complex. Best way to differentiate from atrial fibrillation is clear presence of p waves and regular rhythm. The main causes of atrial flutter are rheumatic heart disease, chronic obstructive pulmonary disease, pericarditis and atrial septal defect. Treatment is based on hemodynamic stability of the patient. Answer D: Ventricular tachycardia is form of wide complex tachycardia characterized by three or more premature ventricular complexes in a row. The ventricular rate can be between 100 and 250 beats per minute. The electrocardiogram of ventricular fibrillation reflects AV dissociation which is evident by a regular rate between p waves with no relationship to QRS complexes. Wide, bizarre QRS complexes that are monomorphic or polymorphic are pathognomonic for ventricular tachycardia. Although most common causes is coronary artery disease, other etiologies include cardiomyopathies, congenital defects, prolonged QT syndrome and drug toxicity. Patients can be asymptomatic or complain of palpitations, dyspnea, angina or near-syncope. Cannon A waves in the neck may be appreciated on physical examination. Any patient with a history of heart disease with a wide complex tachycardia should be presumed to have ventricular tachycardia unless proven otherwise. If patients are asymptomatic and stable, then they can be treated with lidocaine or amiodarone. Patients that are hemodynamically unstable are subjected to immediate synchronous cardioversion with CPR. Answer E: Wolff-Parkinson-White (WPW) syndrome is a differential for wide complex tachycardia. It is an autosomal dominant disorder characterized by an accessory conduction pathway from the atria to the ventricles through the bundle of Kent. As a result, there is an early depolarization of the ventricle resulting in delta wave which is pathognomonic for WPW. Clinical manifestations of WPW syndrome include sudden onset of tachycardia, rapid pulse and fatigue. The electrocardiogram of WPW syndrome demonstrates a narrow complex tachycardia with a short PR interval and a delta wave. Atrial fibrillation with WPW is commonly confused with ventricular tachycardia as well. Treatment includes ablation of the re-entrant loop or anti arrhythmic agents such as procainamide. Bottom Line: Ventricular fibrillation is characterized by a wandering baseline and irregular QRS complexes.

A 45-year-old male from Africa with a hyperpigmented scaling rash on the sun exposed area of his neck would have a deficiency in... A. calcium B. selenium C. vitamin B1 D. vitamin B3 E. zinc

The correct answer is: D Underdeveloped countries with mainly corn and carbohydrate based diet deficient in niacin (vitamin B3) can develop a "casal's necklace" rash: a hyperpigmented erythematous scaly rash just around the neck in the sun exposed areas. Recall the three D's associated with niacin deficiency (pellagra) include diarrhea, dermatitis and dementia.

A 45-year-old male with HIV infection and a CD4+ cell count of 100/mm3 is brought to the emergency department by his wife because of fever and altered mental status. His wife says he has been complaining of a cough, shortness of breath, and a headache for several months but refused to see a doctor. She says his breathing has gotten consistently worse and the headaches are so bad he is experiencing blurred vision and is vomiting multiple times a day. The patient appears confused and lethargic. Physical examination reveals bilateral rales, papilledema, and multiple red papules on his chest and arms. Upon palpation of the cranial vault you note exaggerated cranial rhythmic impulse. Additionally you note paravertebral muscle spasms and fullness from T1-6 bilaterally. Kernig's and Brudzinski's sign are both negative. The most likely etiologic organism is A. Cryptococcus neoformans B. Histoplasma capsulatum C. Mycobacterium tuberculosis D. Neisseria meningitidis E. Cytomegalovirus

The correct answer is: A This patient is presenting with disseminated cryptococcosis with involvement of the skin, lungs, and central nervous system. Cryptococcosis is caused by the yeast-like fungus Cryptococcus neoformans, which is most commonly found in soils contaminated with bird feces (usually pigeon droppings). Cryptococcal infection is acquired via inhalation of aerosolized infectious particles. It is usually asymptomatic in immunocompetent hosts, but can cause a spectrum of serious diseases in immunosuppressed individuals including meningoencephalitis, pneumonia, and skin or soft tissue infections. Patients who are at risk include patients with hematologic malignancies, solid organ transplant recipients on immunosuppressive agents, and patients with HIV/ AIDS (with a CD4+ cell count< 200/ µL). It is important to note that the vast majority of cases occur among AIDS patients with a CD4 count < 100 cells/microL. Patients with pulmonary cryptococcosis may be asymptomatic or present with chronic cough, mild sputum production, and/or pleuritic chest pain. Central nervous system disease usually presents with symptoms of chronic meningitis including fever, headache, and impaired mentation. Symptoms may go unnoticed for several months until intracranial pressure increases, causing blurred vision, nausea, vomiting, and acute/ subacute altered mental status. Patients commonly do not exhibit classic signs associated with meningeal irritation (e.g., neck stiffness, photophobia, positive Kernig's and Brudzinski's sign). Cutaneous manifestations occur in 10%-15% of patients with disseminated cryptococcosis and may include papules, plaques, purpura, tumor-like lesions, and/or rashes. Other organ systems that are less commonly involved include the myocardium, liver, kidneys, adrenals, muscle, and prostate. Diagnosis of cryptococcal infection can be made by visualizing C. neoformans in the CSF using India ink stain or testing for cryptococcal antigen in blood or CSF. Radiographic imaging of the brain should be performed prior to lumbar puncture to rule out the possibility of increased intracranial pressure (ICP) or other space-occupying lesions. Treatment varies depending on host immunity and presence of CNS involvement. Cryptococcosis in HIV patients should always be treated aggressively. HIV-positive patients with disseminated cryptococcus and CNS involvement should receive induction therapy with IV amphotericin B and flucytosine for two weeks followed by high dose fluconazole (400 mg/day) for at least 10 days. The patient should then be started on maintenance therapy with 200 mg/ day fluconazole indefinitely unless antiviral therapy leads to immunologic improvement and they maintain a CD4+ cell count > 100/ mm3 for > 1 year. Prognosis varies depending on the extent of the patient's underlying immunologic deficit. Poor prognostic indicators include abnormal mental status, a CSF antigen titer of 1:1024, and a pleocytosis of < 20 cells/microL. Early detection and introduction of anti fungal therapy is effective at terminating infection in 70%-75% of patients. However, relapse of cryptococcosis is common even among patients with relatively strong immune systems. Osteopathic findings associated with this presentation include TART changes from T1-4 due to the pathology occurring in the head and neck region as a result of viscerosomatic reflexive changes. You may also see TART changes T2-6 from lung viscerosomatic reflexes. Additionally, cranial somatic dysfunction can occur and is variable in its presentation. In the above case you have a hyperdynamic CRI as a result of the increased intracranial pressure. It is important to note all of these findings for later treatment with OMT but you need to thoroughly workup the patient and treat with standard of care measures prior to employing OMT, given that this patient is acutely decompensating and obtunded. Answer B: Histoplasma capsulatum is a dimorphic fungus endemic to regions along the Mississippi river and is most commonly found in caves where there are significant bird or bat feces. Disseminated histoplasmosis can occur in HIV/AIDS patients with CD4+ cell counts < 200/mm3 and usually presents with fever, weight loss, hepatosplenomegaly, lymphadenopathy, non-productive cough, palatal ulcers and pancytopenia. Symptom onset is acute or subacute with average time of presentation within 14 days of exposure. Answer C: Patients with HIV infection are at increased risk of developing disseminated Mycobacterium tuberculosis infection. TB most commonly infects the lungs and central nervous system causing chronic cough and subacute/chronic meningitis. Other organ systems that may be involved with disseminated disease include the skeletal (e.g., Pott disease, arthritis), gastrointestinal (mouth ulcers, esophagitis, abdominal pain, diarrhea) and/or genitourinary (prostatitis, epididymitis, pelvic inflammatory disease) system. Disseminated TB does not cause papular lesions on the skin. Answer D: Neisseria meningitidis is a common cause of acute meningitis in children and adults. Symptoms include acute onset of intense headache, fever, nausea, vomiting, photophobia, and neck stiffness. A petechial or purpuric rash may be found on the trunk, legs, mucous membranes, and conjunctivae. The chronic onset and lack of signs of meningeal irritation make meningococcal meningitis unlikely in this patient. Answer E: Cytomegalovirus (CMV) can cause serious opportunistic infections in HIV-positive patients when their CD4+ cell count is <100/mm3 or viral load is > 10,000. Disseminated CMV infection most commonly presents with gastrointestinal (abdominal pain, nausea, bloody diarrhea) and/or pulmonary symptoms (fever, cough w/ sparse sputum production). CNS involvement usually does not occur until CD4+ cell count is < 50/mm3 and includes polyradiculopathy, transverse myelitis, and subacute encephalitis. Bottom Line: Cryptococcosis should be considered in any patient that presents with signs and symptoms suggestive for chronic meningitis, especially patients with underlying immunosuppression. Cryptococcus neoformans infection can affect almost any tissue type, but most commonly presents with central nervous system and/or pulmonary involvement. Skin lesions associated with disseminated cryptococcosis are highly variable and may include papules, plaques, purpura, tumor-like lesions and/or rashes. Patients with underlying HIV infection should always be treated aggressively with induction therapy (amphotericin B + flucytosine + fluconazole) and lifelong maintenance therapy (fluconazole).

Treatment of Hypocalcemia

Acute symptomatic hypocalcemia should be treated with IV 10% calcium gluconate to achieve a serum concentration of 7 to 9 mg/dL. However, if the hypocalcemia is severe, calcium chloride is given instead. Calcium gluconate requires a higher dosage than calcium chloride. Approximately three times more IV calcium gluconate is required to achieve the same effects as the equivalent dose of calcium chloride. IV calcium chloride is only appropriate if the hypocalcemia is acute and has occurred over a relatively short time frame. If the hypocalcemia has been chronic, then this regimen can be fatal, because there is a degree of acclimatization that occurs. The neuromuscular excitability, cardiac electrical instability, and associated symptoms are then not cured or relieved by prompt administration of corrective doses of calcium, but rather exacerbated. Such rapid administration of calcium would result in effective over correction - symptoms of hypercalcemia would follow. Associated deficits in magnesium, potassium, and pH must also be corrected. Hypocalcemia will be refractory to treatment if coexisting hypomagnesemia is not corrected first. Chronic hypoparathyroidism requires vitamin D, oral calcium supplements, and often aluminum hydroxide gels to bind dietary phosphate in the intestine. Magnesium depletion can cause hypocalcemia by producing PTH resistance, which occurs when serum magnesium concentrations fall below 0.8 mEq/L (1 mg/dL or 0.4 mmol/L) or by decreasing PTH secretion, which occurs in patients with more severe hypomagnesemia Associated deficits in magnesium, potassium, and pH must be corrected in addition to correcting the calcium levels. Answer A: This answer choice is correct, however, intravenous calcium chloride is not the preferred or standard therapy because it can cause tissue necrosis or fatal arrhythmia when infused. It is generally reserved when a more rapid correction is indicated or when calcium gluconate is unavailable. If the hypocalcemia has been chronic, then this regimen can be fatal because there is a degree of acclimatization that occurs. Answer C: Symptoms of hypokalemia (choices D and E) include muscle weakness, myalgias, and in severe cases, flaccid paralysis, inverted or flattened T waves and eventually QRS widening. Answer D: Asymptomatic hypocalcemia can be treated with oral or IV calcium. Answer E: Symptoms of hypokalemia include muscle weakness, myalgias, and in severe cases, flaccid paralysis, inverted or flattened T waves and eventually QRS widening. Bottom Line: Severe, acute, symptomatic episodes of hypocalcemia should be treated with IV calcium gluconate. Associated deficits in magnesium, potassium, and pH must also be corrected. Hypocalcemia will be refractory to treatment if coexisting hypomagnesemia is not corrected first.

CKD

Diabetes and hypertension are the two main causes of chronic kidney disease and are responsible for up to 66% of cases. Chronic kidney disease is defined as presence of kidney damage and/or decreased level of kidney function, present for 3 or more months. Decreased kidney function is usually estimated by GFR using serum creatinine and several equations like Cockcroft-Gault equation, the Modification of Diet in Renal Disease (MDRD) Study equations, and the Chronic Kidney Disease Epidemiology Collaboration (CKD-EPI) equation. Based on the estimate GFR, chronic kidney disease is categorized into 6 different stages. Staging of chronic kidney disease is important to facilitate application of clinical practice guidelines. The National Kidney Foundation [Kidney Dialysis Outcomes Quality Initiative (KDOQI)] defines stages of CKD according to the estimated GFR. Stages Description GFR (mL/min per 1.73 m2) 1 Normal or High > 90 2 Mildly decreased 60-89 3 a Mild to moderately decreased 45-59 3 b Moderate to severely decreased 30-44 4 Severely decreased 15-29 5 Kidney failure < 15 Therefore, since the patient's GFR is 40 mL/min per 1.73 m2, this patient has stage 3 CKD.

A 65-year-old male with end stage renal disease presents to the emergency department complaining of perioral numbness, generalized malaise, and palpitations. History reveals the patient has end-stage renal disease. Structural examination reveals tissue texture changes from T9-L1 bilaterally. A 12-lead electrocardiogram is obtained which demonstrates a prolonged QT interval, T-wave inversions, and premature ventricular contractions. What is the most likely associated electrolyte abnormality? A. hypernatremia B. hypocalcemia C.hypokalemia D. hyponatremia E. hypophosphatemia

Explanation: The correct answer is: B Hypocalcemia is defined as a serum calcium level below 8.5 mEq/L or a decrease in the ionized calcium level below 4.2 mg/dL. The causes of hypocalcemia include pancreatitis, massive soft tissue infections such as necrotizing fasciitis, renal failure, pancreatic and small bowel fistulas, hypoparathyroidism, toxic shock syndrome, abnormalities in magnesium levels, and tumor lysis syndrome. In general, neuromuscular and cardiac symptoms do not occur until the ionized fraction falls below 2.5 mg/dL. Clinical findings may include paresthesias of the face and extremities, muscle cramps, carpopedal spasm, stridor, tetany, and seizures. Patients may exhibit a positive Chvostek's sign (spasm resulting from tapping over the facial nerve) and Trousseau's sign (spasm resulting from pressure applied to the nerves and vessels of the upper extremity with a blood pressure cuff). Hypocalcemia may lead to decreased cardiac contractility and heart failure. ECG changes of hypocalcemia include prolonged QT interval, T-wave inversion, heart block, and ventricular fibrillation. Answer A: Hypernatremia occurs when osmotically active electrolytes (sodium and potassium) are retained without water, when water is lost without these electrolytes, or a combination of these two. Hypernatremia is associated with a prolonged PR interval. Answer C: Hypokalemia will also prolong the QT interval, but renal failure typically causes hyperkalemia instead of hypokalemia. Hypokalemia is associated with depression of the ST segment, decrease in the amplitude of the T wave, and an increase in the amplitude of U waves which occur at the end of the T wave (waveform 1). U waves are often seen in the lateral precordial leads V4 to V6. Answer D: Hyponatremia can occur in patients with advanced renal failure. These patients may appear either euvolemic or, if they also retain salt and develop edema, hypervolemic. The impairment in free water excretion in advanced renal failure can lead to the retention of ingested water and the development of hyponatremia. Hyponatremia may cause non-ischaemic ST elevations on ECG. Answer E: Chronic renal failure typically results in hyperphosphatemia. The effects of hypophosphatemia may be categorized into those that arise from changes in mineral metabolism and those due to adenosine triphosphate (ATP) depletion. Decreased intracellular ATP levels may cause metabolic encephalopathy, impaired myocardial contractility, respiratory failure due to weakness of the diaphragm, a proximal myopathy, dysphagia, and ileus. Bottom Line: Symptoms of severe hypocalcemia may include paresthesias, muscle cramps, Chvostek sign, Trousseau's sign, decreased cardiac contractility, prolonged QT interval, T-wave inversion, heart block, and ventricular fibrillation.

Refeeding syndrome

In refeeding syndrome, a chronic and total body depletion of phosphate is the hallmark electrolyte derangement. It leads to a variety of disturbances, ranging from cardiomyopathy and arrhythmias to rhabdomyolysis, renal failure, encephalopathy, respiratory failure, and even cardiac arrest and/or death. During the early phases of starvation, the body switches from carbohydrate to fat and protein metabolism as the main source of energy, and the basal metabolic rate can decrease by as much as 20-25%. As more prolonged and chronic starvation sets in, the body switches from ketone metabolism and begins using fatty acids for energy. This is done in an attempt to avoid further protein and muscle breakdown. This leads to an increase in blood levels of ketone bodies, which then become the main source of energy for the brain. The liver also decreases its rate of gluconeogenesis, which preserves muscle protein as well. During refeeding, glycemia leads to an increase in insulin production and decrease in glucagon production. Insulin leads to glycogen, fat, and protein synthesis which requires the use of several minerals, including phosphate and magnesium, as well as cofactors like thiamine. The end result is a shift of several minerals and water into the intracellular space, including phosphate, magnesium, and potassium. These are already functionally depleted, and these shifts combined with an increased basal metabolic rate lead to refeeding syndrome. The malnourished patient may be thiamine deficient at baseline. With refeeding, intracellular uptake of electrolytes leads to increased utilization of thiamine, and Wernicke's encephalopathy may occur, with signs that include encephalopathy, oculomotor dysfunction, and gait ataxia.

Ranson's Criteria

Ranson's criteria are a clinical scoring system used to assess severity and predict outcome in patients with acute pancreatitis. Overall mortality in patients with acute pancreatitis is 10-15%. Clinical presentation can vary greatly from mild to severe. Patients with severe disease are at risk for acute respiratory distress syndrome (ARDS), acute renal failure, cardiac depression, hemorrhage, and hypotensive shock. Therefore, it's important to identify patients with severe disease who require aggressive medical treatment. Ranson's criteria estimate mortality of patients with pancreatitis based on initial and 48-hour lab values. Admission/ Initial Criteria Initial 48 hr Criteria Glucose >200 mg/dL Calcium < 8 mg/dL Age >55 years Decrease in Hct >10% LDH >350 U/L. PaO2 < 60 mmHg AST >250 U/L Base deficit >4 mg/dL WBC >16,000 mm3 BUN increase >8 mg/dL Fluid sequestration >6 L Patients with < 3 criteria have 1% mortality rate, 3-4 criteria have 15-20%, 5-6 criteria have 40%, and patients with >7 criteria have 100% mortality rate. This patient has approximately a 15% mortality rate because he had 3 criteria present on initial presentation (elevated LDH, AST, and WBC).

A 56-year-old male presented to the emergency room complaining of right upper quadrant abdominal pain. Vitals reveal blood pressure 75/46 mmHg, heart rate 165/min, respiratory rate 18/min, oxygen saturation 97% on room air, and temperature of 39.0oC (103.5oF). Physical examination reveals scleral icterus and tenting of the skin. A complete blood count is obtained and reveals the following: hemoglobin 14.8 g/dL hematocrit 41% WBC 18,600 cells/mcL platelets 380,000 cells/mcL The most likely underlying cause is A. ascending cholangitis B. cholelithiasis C. chronic pancreatitis D. gastric outlet obstruction E. peptic ulcer disease

The correct answer is: A This patient presented with Charcot's triad (right upper quadrant pain, fever, and jaundice) and started to progress to Reynold's pentad (Charcot's triad with hypotension and altered mental status). These findings are specific for ascending cholangitis. The precipitating causes may be a gallstone in the biliary tract or biliary tract manipulation. These patients are initially treated with intravenous fluid resuscitation and intravenous antibiotics. Medical management usually provides improvement of symptoms to allow biliary drainage to be performed by ERCP. If biliary drainage does not occur overt sepsis may be a result. Reynold's pentad is associated with a higher morbidity and mortality and prompts intervention to restore biliary drainage. If ERCP cannot be performed percutaneous transhepatic biliary drainage is another option. Answer B: Cholelithiasis is simply gallstones within the gallbladder and is not associated with common bile duct obstruction. It is true it may lead to choledocholithiasis, but there is a vast difference between gallstones in the gallbladder and the common bile duct. Most patients with gallstones are asymptomatic and are unaware that they have gallstones. This would not present as a patient with sepsis and a biliary obstruction pattern causing jaundice. Answer C: These patients typically present with abdominal pain and weight loss due to steatorrhea. The steatorrhea is from lack of digestive enzymes secreted. If the patient has an elevation in the bilirubin it may be secondary to chronic inflammation. It is important to know that they will not present as septic patients as this is a recurrence of a chronic inflammatory process and not an infective process. The treatment would be pancreatic enzymes if the patient has steatorrhea and weight loss. Answer D: With gastric outlet obstruction these patients present with nausea and vomiting after meals. They regurgitate partially digested material. The causes may be recurrent peptic ulcer disease in the antral region of the stomach or even malignancy. The initial treatment consists of decompression with nasogastric suction and with benign pyloric stenosis it can be treated with balloon dilatation of the pyloris. In malignancy the patient may require a stent if the patient cannot go to surgery. Answer E: Peptic ulcer disease usually presents as a gnawing pain in the epigastric or left upper quadrant and may be associated with nausea, abdominal bloating and melena. It may be worse with meals. The patients may have a history of NSAID use. The most common cause in the United States is still Helicobacter pylori. The patient would not present with a leukocytosis of and signs of severe sepsis (hypotension). Unless actively losing blood, but there should not be an elevated WBC. Also, this patient has jaundice which is not related to peptic ulcer disease. Bottom Line: Charcot's triad and Reynold's pentad are key clinical points to help diagnose ascending cholangitis. Treatment is with iv fluids, iv antibiotics, and biliary drainage.

A 23-year-old male with a history of ileal resection for Crohn disease presents with right-sided flank pain. History reveals that he had a kidney stone one year prior. Structural examination reveals tissue texture changes from T9-L1 on the right. A plain film radiograph confirms the presence of a 1 cm stone in the right renal pelvis. What is the most likely type of his kidney stone? A. cystine B. oxalate C. phosphate D.struvite E. uric acid

The correct answer is: B Bile acids are normally absorbed by specific receptors in the distal ileum. Bile salt malabsorption occurs when more than 50 to 60 cm of terminal ileum is diseased or resected. Unabsorbed bile salts enter the colon resulting in a secretory or "bile salt" diarrhea. Steatorrhea may cause the binding of calcium ions by fatty acids, which makes calcium unavailable in the enteric lumen for binding to oxalates. With fat malabsorption, calcium in the bowel lumen is bound by fatty acids instead of precipitating with oxalate, which is left free for excessive absorption in the colon. The absorbed oxalate is excreted in the urine, where it may precipitate to form oxalate urinary stones. Hyperoxaluria may also be caused by bacterial overgrowth syndromes, chronic disease of the pancreas and biliary tract, jejunoileal bypass in the treatment of obesity, or ileal resection. Therefore, patient's with Crohn's disease with involvement of the ileum can present with calcium oxalate stones. Answer A: Cystine stones are hexagon shaped and a rare cause of nephrolithiasis. Typically seen in patients with autosomal recessive disorder with cystinuria. It is not associated with Crohn's disease. Answer C: Phosphate stones typically present as calcium phosphate stones often in combination with calcium oxalate stones as well. They are typically associated with low urine volume, high urinary pH, high urine phosphate and calcium excretion. These are also not associated with Crohn's disease. Answer D: Struvite stones are composed of magnesium ammonium phosphate and calcium carbonate apatite. These struvite stones can be seen secondary to infections by urea-splitting organisms such as Proteus and Klebsiella. Occasionally, staghorn shape struvite stones can be seen as well which fill the kidney pelvis. However, they are not associated with Crohn's disease. Answer E: Uric acid stones are typically seen in patients with low urine volume or hyperuricosuria. Hyperuricosuria is seen in patients with high protein diet, gout, diabetes mellitus, chronic diarrhea and tumor lysis syndrome. Treatment includes increased fluid intake and alkalinization of the urine as well. However, uric acid stones are not associated with Crohn's disease. Bottom Line: Oxalate urinary stones may form after ileal resection secondary to increased oxalate absorption by the colon. COMBANK Insight : A plain film of the abdomen can identify calcium-containing (calcium oxalate), struvite (staghorn) and cysteine stones

A 65-year-old male presents for a routine wellness examination. He has a history of hypertension which is well controlled on hydrochlorothiazide, a 30-pack-year smoking history, and occasional alcohol use. Review of system is negative for any chest pain, dyspnea, nocturia, and heat or cold intolerance. He walks 2-3 miles daily and lives an active lifestyle. All vital signs are within normal limits. Cardiovascular and lung examination are unremarkable. The most appropriate prevention recommendation at this time is A. abdominal ultrasound B. cardiac stress test C. plain film chest radiograph D.prostate specific antigen E. spirometry

The correct answer is: A Abdominal ultrasound has a sensitivity and specificity of 95-100 % for abdominal aortic aneurysm. The United States Preventive Services Task Force (USPSTF) recommends a one time screening abdominal ultrasound for all men aged 65 - 75 years of age who have ever smoked in their lives. A normal diameter of aorta is usually 2.0 cm, abdominal aortic aneurysm is classified as diameter greater than 3.0 cm. Serial ultrasounds are further recommended based on the initial diameter of the abdominal aorta. Therefore, abdominal ultrasound is the most important preventive screening tool for this patient with a history of smoking. Answer B: Current guidelines do not recommend a stress test for asymptomatic patients. This patient has no history of coronary artery disease, no complaint of chest pain and normal cardiovascular examination; therefore, making it a less likely answer choice. Answer C: A chest x-ray is no longer recommended for screening for lung cancer in patients with a history of smoking. The most recent guidelines recommend low dose CT chest for screening of lung cancer in patients with greater than 30 pack year smoking history. Answer D: As per the most recent USPSTF guidelines, a routine prostate specific antigen is no longer recommended as a screening tool for prostate cancer. Instead, physicians are encouraged to discuss with patients the risk and benefit of screening for prostate cancer based on symptoms and family history. However, PSA is recommended as a screening tool in patients with a strong family history of prostate cancer or BRCA 1 mutations as well as in patient's with symptoms. Answer E: Current guidelines do not recommend screening asymptomatic patients with history of smoking for emphysema with spirometry. Since this patients has no history of dyspnea and no wheezing on examination, there is no indication for spirometry at this point. Bottom Line: An abdominal ultrasound for abdominal aortic aneurysm screening is indicated in all men of 65-75 years of age who have ever smoked.

A 67 year-old male with a two-day history of paresthesias and witnessed seizures would have a deficiency in... A. calcium B. selenium C. vitamin B1 D. vitamin B3 E. zinc

The correct answer is: A Acute calcium deficiency can present with various manifestations of neuromuscular dysfunction including peri-oral numbness, paresthesias, muscle spasms, muscle cramps and seizures.

A 4-year-old African American child is admitted to the hospital because of fever, pallor, fatigue, and back pain. Physical examination reveals a febrile child in pain and discomfort. There is pallor of the conjunctiva, rales at the right lung base, and petechiae diffusely over the body. Laboratory analysis reveals a high white blood cell count and an anemia. The most likely diagnosis is A. acute lymphoblastic leukemia B. mononucleosis C. pneumonia D. sickle cell anemia E. vitamin K deficiency

The correct answer is: A Acute lymphoblastic leukemia has a peak incidence between ages 2 and 5. Symptoms reflect bone marrow infiltration (blasts replacing bone marrow): anemia (pallor, fatigue), thrombocytopenia (petechiae) and neutropenia (fever). Upon initial evaluation, obtain a complete blood cell (CBC) count. A hematologist or hematopathologist must evaluate the peripheral smear for the presence and morphology of lymphoblasts (see below). An elevated leukocyte count of more than 10 × 109/L (>10 × 103/µL) occurs in one half of patients with acute lymphoblastic leukemia (ALL). Neutropenia, anemia, and thrombocytopenia are often observed secondary to inhibition of normal hematopoiesis by leukemic infiltration. It is important to recognize that 20% of patients with ALL initially present with pancytopenia and no evidence of peripheral blasts. Answer B: Mononucleosis causes fatigue, but it is typically found in teenagers and is also not associated with petechiae. Answer C: Although the child may have pneumonia with findings on the physical exam, the pallor and petechiae would not be explained by pneumonia. Answer D: While the presenting symptoms of sickle cell anemia are similar to those of ALL, sickle cell disease affects red blood cells and is not associated with petechiae, which in this case represents the thrombocytopenia present in ALL. Answer E: Vitamin K deficiency affects clotting factors II, VII, IX, and X but not platelets and thus would not have petechiae. Bottom Line: Acute lymphoblastic leukemia presents in a 2-5 year-old child with fever, pallor, and petechiae.

A 40-year-old male is admitted to the hospital for fatigue and weakness. History is negative for renal failure. Structural examination reveals a Chapman's point 1" lateral and superior to the umbilicus. You also note paravertebral muscle spasms and tenderness from T1-4 and T6-9. Laboratory studies reveal a serum potassium of 7 mEq/L. An electrocardiogram is obtained and demonstrates peaked T-waves. What is the most appropriate next step in management? A. calcium gluconate B. close observation C. furosemide D.IV insulin and D50 E. kayexylate

The correct answer is: A Although all answers may be appropriate for the treatment of hyperkalemia, this particular situation requires immediate attention due to EKG changes. Cardiac complications such as ventricular fibrillation and cardiac arrest may occur with untreated hyperkalemia. Given the patient's peaked T-waves on EKG, the administration of calcium gluconate is the appropriate first step. Calcium stabilizes membrane potentials and prevent further cardiac sequelae. However, the effect of calcium is only transient, and further steps are required to correct the patient's hyperkalemia. Other treatments of hyperkalemia not provided in the answer choices include administration of IV sodium bicarbonate (also only a temporary effect), peritoneal dialysis, or hemodialysis. Osteopathic findings in this case point to both renal dysfunction and cardiac pathology. You are presented with TART changes from T1-4 representative of viscerosomatic reflex from underlying cardiac pathology, TART changes from T6-9 representative of viscerosomatic reflex from the kidneys, and an anterior Chapman point for the kidney. Answer B: Close observation is incorrect as this patient may soon enter a lethal arrhythmia secondary to hyperkalemia. Answer C: Furosemide promotes the renal excretion of potassium, thus lowering blood levels. Answer D: Administration of 10-20 units of IV insulin causes an intracellular shift of potassium. This dose of insulin must be accompanied by administration of glucose to avoid hypoglycemia. Answer E: Kayexylate is a sodium/potassium exchange resin that promotes excretion of potassium through the GI tract. Bottom Line: The first treatment of hyperkalemia with EKG changes is calcium gluconate to prevent severe cardiac complications.

A 24-year-old male presents with blurred vision, periorbital pain, and photophobia. Physical examination reveals multiple aphthous ulcers on the oral mucosa and three symmetrical, shallow ulcers on the scrotum. Funduscopic examination is significant for positive cell and flare in the anterior chamber. Tzanck smear of the genital lesions is negative for multinucleated giant cells. The most likely diagnosis is A. Behcet's syndrome B. chancroid C. generalized herpes simplex virus 2 infection D.sarcoidosis E. Wegener's granulomatosis

The correct answer is: A Behcet's syndrome is a multisystem disease that presents with recurrent oral and genital ulcers and possible skin and/or ocular lesions. It is an inflammatory vasculitis that mainly involves the venous system. Diagnostic criteria includes recurrent pathos ulcers (defined as at least 3 episodes within a 12-month period) plus two of the following: Recurrent genital ulcers, commonly involving the scrotum in men and labia in women and sparing the glans penis and urethra. Ulcers commonly heal within 1-2 weeks with scarring. Eye lesions, including anterior or posterior uveitis, hypopyon, retinal vasculitis, cystoid macular degeneration. Skin lesions, including folliculitis, erythema nodosum, and/or acne-like exanthem. Positive pathergy test, a diagnostic test that involves irritating the skin with a sterile needle. A positive result is marked by the formation of a erythematous papule 2 mm in diameter or larger that within 48 hours Patients may also have symptoms due to visceral organ disease, involving the gastrointestinal tract, pulmonary, musculoskeletal, cardiovascular and neurological systems. Behcet's disease usually presents in early adulthood with a mean age of onset between 25-30 years of age. Treatment depends on individual symptoms; however, it usually includes immunosuppressive therapy with glucocorticosteroids and azathioprine. Serious complications include blindness, permanent neurological deficits, aneurysms, and thrombotic events. Answer B: Chancroid is a bacterial sexually transmitted infection caused by Haemophilus ducreyi characterized by painful genital ulcerations. Chancroid lesions are usually large (10-20 mm) with irregular, deep margins and bloody or purulent secretions. They are commonly located on the penile shaft, glans penis, and/or urethral meatus, sparing the scrotum. Patients commonly have significant painful inguinal lymphadenopathy, often with abscess formation. Chancroid infections do not involve the eyes or skin. Answer C: HSV-2 infection can also cause painful, ulcerative lesions on the oral mucosa and genitals. Lesions are usually preceded by prodromal symptoms of tingling, burning and pruritus. Herpes simplex virus can also infect the eye, causing unilateral follicular conjunctivitis or acute herpetic keratoconjunctivitis with dendritic corneal ulcers. This is the leading bacterial cause of blindness in the United States. Tzanck smear of fluid from the lesion will show multinucleated giant cells in herpes simplex virus infection. Answer D: Sarcoidosis is a multisystem disease characterized by noncaseating granuloma formation. Patients usually present with systemic (fever, weight loss, malaise) and/or pulmonary (dyspnea on exertion, cough, chest pain) complaints. Other common symptoms include arthritis, uveitis, erythema nodosum, and rash. Chest x-ray will show bilateral hilar lymphadenopathy. Sarcoidosis does not cause oral and genital ulcerations. Answer E: Wegener's granulomatosis is a necrotizing vasculitis of small arteries and veins that commonly affects the upper airway, lungs, and kidneys. The majority of patients (73%) have upper respiratory symptoms (e.g., sinusitis, nasal mucosal ulceration, nasal septal perforation, and serous otitis media) on initial presentation. Lung involvement may cause cough, hemoptysis and chest pain. Glomerulonephritis develops in 77% of patients and can lead to death if left untreated. Eye involvement (e.g., conjunctivitis, scleritis) and skin lesions (papules, ulcers, vesicles) can occur, but are less common. Bottom Line: Behcet's disease is an immune-mediated, small-vessel systemic vasculitis that commonly presents with recurrent oral aphthous ulcers, genital ulcers, and uveitis. Patients may have multisystem involvement affecting the gastrointestinal tract, lungs, musculoskeletal system, and neurological system. It is seen in young adults, with a mean age of onset between 25-30 years old.

A 62-year-old female presents with worsening fatigue and dyspnea. History reveals the patient is comfortable at rest and can ambulate without dyspnea, however, she experiences shortness of breath when gardening or walking upstairs. Physical exam reveals elevated JVD, paravertebral hypertonicity from T1-4, and a tender nodule in the second intercostal space to the left of the sternum. The most appropriate NYHA classification of this patient is A. I B. II C. III D.IV

The correct answer is: B Congestive heart failure is characterized by a failure of the heart to pump enough blood to meet the metabolic demands of the body tissues. It can be divided into high- or low-output failure, left- or right-sided failure, systolic or diastolic failure. There is often significant crossover between these categories. Elevated jugular venous distention (JVD) indicates a state of hypervolemia or fluid overload. Additionally, you can see osteopathic findings associated with CHF as presented in the above vignette. You note TART changes from T1-4 representing a viscerosomatic reflex from the cardiac tissue. Additionally you note the anterior cardiac chapman point in the left 2nd ICS adjacent to the sternum. Once the diagnosis of CHF is made, the New York Heart Association (NYHA) functional classification system can be used to assess the patient's severity of symptoms and determine the best course of therapy. Classification is as follows: Class I (Mild): No limitation of physical activity. Ordinary physical activity does not cause symptoms (fatigue, palpitation, or dyspnea). Class II (Mild): Slight limitation of physical activity. Comfortable at rest, but prolonged or moderate physical activity results in symptoms(e.g., climbing stairs, carrying groceries). Class III (Moderate): Marked limitation of physical activity. Comfortable at rest, but less than ordinary activity causes symptoms (e.g., walking across room, getting dressed). Class IV (Severe): Unable to carry out any physical activity without discomfort. Symptoms of cardiac insufficiency at rest. If any physical activity is undertaken, discomfort is increased. This patient has class II CHF because she is asymptomatic at rest and with ambulation but becomes symptomatic with moderate exertion. All patients with CHF, regardless of class, should be on a salt restricted diet and an ACE inhibitor (or ARB if ACEI not tolerated). Addition of a beta-blocker should be considered in patients with moderate disease (class II-III), especially if they have coronary artery disease. Digoxin can be added to the regimen for symptomatic relief in patients with class IV CHF. Lastly, studies have shown that the addition of spironolactone reduces mortality and morbidity in patients with class III and IV disease.

A 32-year-old female complains of low-grade fever and worsening otalgia of the left ear for four days' duration. Otoscopic examination reveals a bulging, erythematous, immobile left tympanic membrane and fluid within the middle ear. The right tympanic membrane is mobile and non-erythematous. The outer ear is non-tender and the external auditory canal appears normal bilaterally. Upon palpation of the cranial vault you note decreased amplitude of cranial rhythmic impulse and restriction of the squamous portion of the temporal bone on the left. You also note paravertebral tenderness and muscle spasms from T1-4. The most appropriate next step in management is A. myringotomy with placement of tympanostomy tubes B. observation C. ofloxacin ear drops D. oral amoxicillin E. oral trimethoprim-sulfamethoxazole

The correct answer is: D Otitis media (OM) is an infection of the middle ear that commonly occurs in children, but can also occur in adults (especially if they suffered from recurrent OM as children). The most common bacterial pathogens are Streptococcus pneumonia, Haemophilus influenza, and Moraxella catarrhalis. It may present with ear pain, hearing loss, and/ or vertigo. On otoscopic examination, the tympanic membrane (TM) is erythematous and bulges due to fluid in the middle ear. Pneumatic otoscopy reveals an immobile tympanic membrane. High-dose amoxicillin (80-90 mg/kg/day) for 10 days is the initial antimicrobial treatment of choice in acute OM. Amoxicillin is a beta-lactam antibiotic that inhibits synthesis of the bacterial cell wall. It has high clinical success, acceptability, limited side effects, and relatively low cost. Symptoms of OM usually resolve in 24-48 hours after initiating antibiotic treatment. If the patient fails to show improvement after 48-72 hours of amoxicillin therapy and is still experiencing symptoms (e.g., ear pain, fever, or bulging tympanic membrane), Amoxicillin-clavulanate can be used to treat resistant cases. Complications of OM include persistent fluid buildup, TM perforation, acute mastoiditis, petrositis, cholesteatomas and labyrinthitis. Cranial somatic dysfunction is very commonly associated with otitis media and cranial manipulation can be helpful in treating the infection and preventing recurrence. Additionally, you may see viscerosomatic reflex resulting in TART changes from T1-4 as a result of head and neck pathology. Answer A: Myringotomy is a procedure in which a small incision is made in the tympanic membrane to relieve pressure caused by excessive fluid build-up in the middle ear. Tympanostomy tubes are commonly placed in the membrane to drain the middle ear and prevent abscess formation. Decisions regarding the placement of tympanostomy tubes are made on a case-by-case basis with tube placement generally reserved for those with chronic or recurrent acute otitis media. Myringotomy is also indicated in patients with increased hearing loss, poor response to medical therapy, or intractable pain. Answer B: Observation is appropriate for patients who are > 2 years old with recent onset of non-severe illness. This is to prevent the misuse of antibiotics in cases of OM caused by viral pathogen. Close follow-up is necessary in these patients and antimicrobial therapy should be initiated if symptoms worsen or do not improve within 48-72 hours. Observation would not be appropriate in this patient because she has already had symptoms for > 72 hours and they have continued to worsen over this time period. Answer C: Antimicrobial ear drops, such as ofloxacin otic or ciprofloxacin otic, are liquid preparations of the fluoroquinolone antibiotics that can be used as a topical agent. They are the treatment of choice for otitis externa, a bacterial infection of the external auditory canal. They are commonly used in conjunction with a steroid ear drop. Ear drops are ineffective at treating otitis media. Therefore, it is important to be able to differentiate otitis externa from otitis media. Unlike otitis media, otitis externa causes severe tenderness with palpation of the outer ear tragus. Otoscopic examination reveals erythema and swelling of the external auditory canal with serous drainage. Answer E: Trimethoprim-sulfamethoxazole (TMP-SMX) is a combination of two antibiotics that act synergistically together to inhibit bacterial folate synthesis. It can be used to treat otitis media in patients with a penicillin allergy. However, it is not the initial drug of choice. Bottom Line: Otitis media is an infection of the middle ear cavity that is common in children, but also can be seen in adults. Common pathogens include Streptococcus pneumonia, Haemophilus influenza, and Moraxella catarrhalis. Pharmacologic treatment of choice is amoxicillin 80-90 mg/kg/day for 10 days.

A 67-year-old female presents for a general wellness examination. A bone density scan is obtained and reveals a T score of -3.50. Which of the following is recommended to prevent hip and vertebral fractures in this patient? A. alendronate B. calcitonin C. raloxifene D. teriparatide E. zoledronic acid

The correct answer is: A Bisphosphonates are the most commonly used therapies for osteoporosis. They are potent inhibitors of bone resorption. Alendronate was the first bisphosphonate approved in the U.S. It reduces the incidence of vertebral and non-vertebral fractures by about half. Current guidelines recommend treatment of osteoporosis with alendronate or risedronate for 5 years, followed by a drug holiday and re-assessment of therapy. Adverse drug effects include gastroesophageal reflux, esophageal ulcer, myalgia, and arthralgia. Jaw bone necrosis is a rare, but serious, adverse effect. Answer B: Salmon calcitonin (Miacalcin) is approved in the United States for the treatment, but not prevention, of postmenopausal osteoporosis. It is available as a nasal spray and subcutaneous injection. Calcitonin is an inhibitor of bone resorption but the reduction in bone turnover is much less than that associated with other antiresorptive agents. The use of intranasal calcitonin for five years significantly reduces the risk of new vertebral fractures in postmenopausal women with osteoporosis but does not reduce the risk of hip fracture. After five years of therapy, significant increases in bone density can only be achieved with higher doses. Answer C: Raloxifene is a selective estrogen receptor modulator (SERM) and has been shown in short-term studies to prevent vertebral fractures, but does not reduce risk of non-vertebral fractures. Answer D: Human parathyroid hormone, teriparatide, is the first approved drug for osteoporosis treatment that acts by stimulating osteoblastic activity. This medication required daily injections and is quite expensive. Teriparatide has demonstrated increases bone density and decreases back pain associated with recurrent vertebral fractures, but no efficacy in prevention of non-vertebral fractures. Answer E: Zoledronic acid is a yearly intravenous bisphosphonate primarily used for the prevention of skeletal fractures in patient with multiple myeloma and prostate cancer. Zoledronic acid is used in the treatment of osteoporosis when a patient is unable to tolerate oral medication. Like alendronate, zoledronic acid prevents vertebral and non-vertebral fractures. Bottom Line: Medications for the treatment of osteoporosis that have demonstrate efficacy in the prevention of vertebral and non-vertebral fractures include alendronate, risedronate and zoledronic acid. Current guidelines suggest therapy with oral bisphosphonates for 5 years, followed by a drug holiday and re-evaluation. Zoledronic acid is reserved for patients with prostate cancer, multiple myeloma or for those who do not tolerate oral therapy.

A 45-year-old man who underwent a kidney transplant for diabetic nephropathy complains of abdominal pain, bloody diarrhea, and vomiting. Physical examination reveals a well-healed surgical scar but is otherwise unremarkable. Structural examination reveals tissue texture abnormalities from L1-L3 on the left. A complete blood count confirms pancytopenia secondary to immunosuppression. A colonoscopy is performed and reveals large atypical cells with intranuclear inclusion bodies. The most appropriate pharmacologic management is A. ganciclovir B. interferon and ribavirin C. laparotomy with resection D. observation E. vancomycin

The correct answer is: A Cytomegalovirus (CMV) colitis is suspected in any immunocompromised patient who presents with bloody diarrhea. Additional findings of pancytopenia further support CMV infection over other acute bacterial infections. Diagnosis is confirmed by endoscopy showing punctuate and superficial erosions to deep ulcerations in the colon. Biopsy of these ulcers reveals characteristic cytomegalic cells which are large cells containing eosinophilic intranuclear intracytoplasmic inclusions, as present in this patient. Therefore, prompt treatment should be initiated to avoid complications of toxic megacolon and necrotizing colitis. IV ganciclovir or foscarnet are typical first line agents and are typically continued for 3-6 weeks for treatment of CMV colitis. Answer B: Interferon and ribavirin combination therapy eradicates recurrent hepatitis C infection in some patients after transplant. However, this patient's symptoms of abdominal pain with bloody diarrhea are not consistent with hepatitis C infection, making this an incorrect answer choice. Answer C: CMV colitis can often be confused with ischemic colitis due to similar presentation of abdominal pain and bloody diarrhea. However, ischemic colitis patients may present with acutely worsening abdominal pain that is out of proportion to physical exam findings. Laboratory work up may also reveal an elevated LDH and lactate as well. These patients may demonstrate signs of peritonitis which may warrant immediate laparotomy with resection. However, this patient does not demonstrate any of these signs making it a less likely answer choice. Answer D: CMV colitis if left untreated can result in mucosal hemorrhage and perforation which can be life-threatening. Therefore, observation would be inappropriate for this patient with evidence of CMV colitis on biopsy. Answer E: Clostridium difficile colitis is an important differential diagnosis for patients presenting with bloody diarrhea. However, it is usually related to explosive, foul-smelling diarrhea in a patient with prior antibiotic use or hospitalization. Bloody bowel movements are not typical but can be present, especially in severe cases in which there is associated mucosal sloughing. Treatment consists of stopping antibiotics, supportive care, and oral or intravenous metronidazole or oral vancomycin. However, vancomycin is not an appropriate treatment for this patient with CMV colitis making it an incorrect answer choice. Bottom Line: The mainstay of cytomegalovirus (CMV) therapy is ganciclovir.

A 51-year-old male presents with a diffusely erythematous skin rash with an associated fever. History reveals that he takes atenolol and allopurinol for hypertension and gout. Physical examination reveals marked maculopapular erythema over 90% of the body that is tender to palpation. There is lymphadenopathy noted in the neck and inguinal region. Laboratory studies reveal a WBC count of 15,000/mm3 with 25% eosinophils, an AST level of 100 U/L, and an ALT level of 110 U/L. The most likely diagnosis is A. DRESS syndrome B. red man syndrome C. Stevens-Johnson syndrome D. toxic epidermal necrolysis E. toxic shock syndrome

The correct answer is: A Drug Reaction with Eosinophilia and Systemic Symptoms (DRESS) syndrome is a drug-induced condition characterized by an extensive rash, fever above 38°C, lymphadenopathy, hematologic abnormalities, hepatitis, and involvement of at least one internal organ. The onset of symptoms occurs 2-6 weeks after drug initiation. It carries a mortality rate of 10-20%, with most fatalities secondary to liver failure. Treatment consists of supportive therapy, corticosteroids, and antihistamines to prevent or minimize additional organ system involvement. Answer B: The most common adverse reaction to vancomycin is "red man syndrome" (RMS). RMS is an idiopathic infusion reaction, which may develop acutely with the first administration of vancomycin. RMS is characterized by flushing, erythema, and pruritus, usually affecting the upper body, neck, and face more than the lower body. Pain and muscle spasms in the back and chest, dyspnea, and hypotension may also occur. Symptoms may be treated or prevented with antihistamines and are less likely to occur with slower infusion of vancomycin. Answer C: Symptoms of Stevens-Johnson syndrome include purpuric macules and targetoid lesions, full-thickness epidermal necrosis with detachment in pressure areas and mucous membrane involvement. Inciting agents include medications and Mycoplasma infections. The mortality rate is much lower than in toxic epidermal necrolysis, approaching 5% of cases. Lesions begin symmetrically on the face and the upper part of the torso and extend rapidly, with maximal extension in 2-3 days. Painful, edematous erythema may appear on the palms and the soles. The hairy scalp typically remains intact, but the entire epidermis, including the nail beds, may be affected. Answer D: Toxic epidermal necrolysis is an acute disorder characterized by widespread erythematous macules and targetoid lesions with focal full-thickness epidermal necrosis and involvement of more than 30% of the cutaneous surface. The mucous membranes are commonly involved. Nearly all cases of toxic epidermal necrolysis are induced by medications, and the mortality rate can approach 40%. Answer E: Toxic shock syndrome (TSS) is a toxin-mediated acute life-threatening illness, usually precipitated by infection with either Staphylococcus aureus or group A Streptococcus (GAS). It is characterized by high fever, rash, hypotension, multiorgan failure and desquamation of the palms and soles 1-2 weeks after the onset of acute illness. Bottom Line: Drug Reaction with Eosinophilia and Systemic Symptoms (DRESS) syndrome caused by multiple medications, including allopurinol, anticonvulsants, sulfa derivatives, antidepressants, nonsteroidal anti-inflammatory drugs, and antimicrobials. This disease has a high mortality rate and is treated with steroids and antihistamines to prevent or minimize organ system involvement.

A 44-year-old female with a history of multiple episodes of deep venous thrombosis presents to the hospital with complaints of lower extremity pain and swelling. She notes that her father had similar episodes as well. What hereditary disorder does this patient most likely have? A. activated protein C resistance B. antithrombin deficiency C. hyperhomocysteinemia D. protein C deficiency E. prothrombin gene mutation G20210A

The correct answer is: A Factor V Leiden is the most common hereditary hypercoaguable disorder that occurs in about 5% of the population. Patients with Factor V Leiden have protein C resistance which increases the risk for thrombotic disease. Approximately 30% of patients with Factor V Leiden will have a thrombotic event by the age of 60. Answer B: AT deficiency is usually inherited in an autosomal dominant fashion, with variable clinical penetrance, affecting both sexes equally. About 60% of affected individuals have recurrent thrombotic episodes that start to occur after puberty. Answer C: Homocystinuria or severe hyperhomocysteinemia is a rare autosomal recessive disorder characterized by severe elevations in plasma and urine homocysteine concentrations. Less marked elevations in plasma homocysteine are much more common, occurring in 5-7% of the population. Answer D: Protein C is a vitamin K-dependent protein produced in the liver. Protein C deficiency is inherited in an autosomal dominant fashion at a frequency ranging from 1/200 to 1/500 in the general population. Answer E: A report in 1996 identified a transition mutation (guanine to adenine) at nucleotide 20210 in the 3' untranslated region of the prothrombin gene as a risk factor for thrombosis. Follow-up studies have identified a prevalence of 0.5-6.5% in Europe, largely in Caucasians. Bottom Line: Factor V Leiden is the most common hereditary hypercoaguable disease.

A 40-year-old obese female presents with sudden-onset acute abdominal pain described as sharp and stabbing at a 10/10 on the pain scale. The pain is located in the right lower quadrant with radiation to the back. Vital signs include: Temperature 38.4ºC (101.2ºF) Blood pressure 100/60 mmHg Heart rate 120/min Respiratory rate 30/min Oxygen saturation 92% on ambient air Physical examination reveals hyperactive bowel sounds and mild abdominal distention. The patient is exquisitely tender to palpation in the right upper and lower quadrants. Cardiovascular examination reveals normal S1 and S2 with irregularly irregular rhythm. Routine laboratory studies reveal leukocytosis but are otherwise normal. A fecal occult blood test is obtained and read as positive. Which of the following is the most appropriate diagnostic modality? A. abdominal CT scan B.acute abdominal series C. emergent colonoscopy D. plain film radiography of the abdomen and pelvis E. ultrasound of the right upper quadrant

The correct answer is: A In any patient with abdominal pain, early recognition of acute abdomen or surgical abdomen is very important. Acute abdomen generally refers to sudden onset of abdominal pain with rapid clinical deterioration. It is often a clinical diagnosis based on presence of guarding, rebound tenderness and "pain out of proportion" to physical exam, such as this patient. Other objective signs include the presence of fever, leukocytosis, tachycardia and tachypnea which indicates sepsis from possible intra-abdominal pathology. Early recognition and early diagnosis of etiology of acute abdomen can be life saving. In this patient, a differential diagnosis includes appendicitis (right lower quadrant pain with radiation to back), cholecystitis (fat, female, fertile, forty years of age with right upper quadrant pain), peritonitis (fever, leukocytosis, signs of infection) and mesenteric ischemia (atrial fibrillation with positive fecal occult). Only one of the answer choices listed above could help narrow down the differential diagnosis and give the correct diagnosis in a timely fashion, that is CT abdomen. A complete CT abdomen is a quick efficient way to look at the gall bladder, appendix, mesentery and the colonic mucosa all together; therefore, making it the most appropriate next step in management. Answer B: An acute abdominal series is a set of plain film radiograph views of the abdomen to rule out obstruction. Obstruction and peritonitis are the two most common etiologies of acute abdomen. Although an obstruction series is a very useful test in diagnosing partial vs. complete bowel obstruction, it does not help us rule out other possible etiologies of acute abdomen in this patient as mentioned above. Answer C: A positive fecal occult blood test makes colonoscopy a tempting option. A possible differential for positive fecal occult blood includes mesenteric ischemia (atrial fibrillation with possible embolic phenomenon) vs diverticulitis/diverticulosis (fever, leukocytosis, lower quadrant pain) vs perforated peptic ulcer disease. However, colonoscopy would not help in urgent management of this patient with an acute abdomen and could be performed at a later time interval. Answer D: In patients with acute abdomen, an upright abdominal radiograph (KUB) can be a quick test to rule out perforation by presence of free air under the diaphragm. Abdominal radiograph can also show dilated loops of bowel suggestive of ileus as well. However, it is not very useful to help differentiate between peritonitis, cholecystitis, appendicitis and mesenteric ischemia, which are the top few differentials in this patient. Therefore, even though an abdominal radiograph may be the quickest test to diagnose a perforation, it is not the most appropriate next step in management of this patient with an acute abdomen. Answer E: This patient has all four risk factors for cholecystitis including fat, female, fertile and forty years of age making acute cholecystitis very high on our differential diagnosis. Also, a right upper quadrant ultrasound is the best diagnostic procedure for acute cholecystitis. Some of the signs to look for in the ultrasound include gallbladder wall thickening or edema and sonographic "Murphy's sign:" when the examiner presses the gall bladder as the patient is taking a deep breath resulting in the patient catching his/her breath due to pain. However, a positive fecal occult blood is less likely with cholecystitis. Therefore, a CT scan would be a better test to rule out other differential diagnosis as a right upper quadrant ultrasound would only test us information about the gall bladder. Additionally, a FAST ultrasound exam or a complete abdominal ultrasound study would be more applicable to this acute situation. Bottom Line: CT abdomen is the best initial imaging test for an acute abdomen.

A 47-year-old male undergoes a routine pre-operative examination for a long-standing umbilical hernia repair. The patient receives a plain film chest radiograph during the encounter and is discovered to have a right lower lobe nodule. Further workup includes a CT scan of the chest which delineates this nodule to be 7 mm with a popcorn like calcification as shown in the exhibit. The most likely diagnosis is A. hamartoma B. hydatid cyst C. lung adenocarcinoma D.progressive fibrosis E. tuberculosis

The correct answer is: A Popcorn like calcifications are typically benign and are the result of pulmonary hamartomas. A hamartoma is a benign, focal malformation that resembles a neoplasm in the tissue of its origin. This is not a malignant tumor, and it grows at the same rate as the surrounding tissues. It is composed of tissue elements normally found at that site, but which are growing in a disorganized mass. They occur in many different parts of the body and are most often asymptomatic and undetected unless seen on an image taken for another reason. Answer B: Hydatid cysts typically do not calcify. Answer C: Primary lung cancer calcifications can appear as punctuate, amorphous, or reticular. Answer D: Progressive massive fibrosis often presents as a mass like consolidation with lung scarring and bullae. Answer E: Tuberculosis calcifications are dystrophic and present as parenchymal granulomas. Bottom Line: Popcorn-like calcifications are typically associated with pulmonary hamartoma and do not have malignant potential.

A 62-year-old male comes to the emergency department complaining of abdominal pain and blood clots in his stool. Structural examination reveals ropy tissue texture changes from T5-T9 bilaterally. A CT scan is performed that shows gastric varices and a thrombus in the splenic vein. What is the most common etiology for this condition? A. chronic pancreatitis B. ductal adenocarcinoma of the pancreas C. factor V Leiden D.intraductal papillary mucinous tumor E. pancreatic pseudocyst

The correct answer is: A Splenic vein thrombosis most commonly occurs due to chronic pancreatitis. This condition can result in gastric varices without elevation in portal pressures. The treatment for isolated gastric variceal bleeding secondary to splenic vein thrombosis is a splenectomy. Answer B: More than 95% of pancreatic cancers arise from the exocrine elements. The most common presenting symptoms in patients with exocrine pancreatic cancer are pain, jaundice, and weight loss. For tumors arising in the tail, regional nodal basins are located along the common hepatic artery, celiac axis, splenic artery and splenic hilum. Answer C: Factor V Leiden is the most common cause of inherited thrombophilia in Caucasians. Clots can occur anywhere, but this condition is not specific to splenic vein thrombosis. Answer D: Intraductal papillary mucinous neoplasm of the pancreas (IPMN) is a cystic neoplasm of the pancreas with malignant potential. It may involve the main pancreatic duct, the branch ducts, or both. Progression to cancer would increase the risk of thrombosis, but this neoplasm only has malignant potential. Answer E: Pseudocysts develop in approximately 10% of patients with chronic pancreatitis. Most are asymptomatic, but when present they can cause abscess formation, pseudoaneurysms of adjacent vessels, ascites or pleural effusion, and biliary obstruction. The diagnosis is usually made with an abdominal CT scan. Bottom Line: The most common cause of splenic vein thrombosis is chronic pancreatitis.

A 38-year-old female presents to the office after a syncopal event at home. The patient states that during the event she was sitting on her couch watching television with no preceding symptoms other than light-headedness. Vital signs reveal: Blood pressure 136/84 mmHg Heart rate 78/min Respiratory rate 18/min Temperature 36.89oC (98.4oF) Oxygen saturation 98% on room air Physical examination is negative for carotid bruits and cardiac murmurs with excellent pulses throughout. Orthostatic vital signs reveal a drop in blood pressure from supine to standing of 12 mmHg systolic and 4 mmHg diastolic. An electrocardiogram is obtained and read as having a normal sinus rhythm. Which of the following is the most appropriate diagnostic modality? A. 24 hour Holter monitor B. cardiac stress test C. echocardiogram D. electroencephalogram E. tilt table test

The correct answer is: A Syncope can be caused by many different etiologies. Due to the patients history and physical it would be pertinent to rule out arrhythmias as a cause. Electrocardiograms only capture a snap shot of the patient's cardiac rhythm status. For a patient that has syncopal events it is important to rule out arrhythmias as a cause. This is better evaluated with a 24 hour Holter monitor. If the patient continues to have syncopal events a 30 day Holter monitor may be performed. There are a variable of arrhythmias that may cause syncope. Bradyarrhythmias that cause syncope are attributable to sinus node disease (sinus arrest or sinoatrial block) or atrioventricular block (Mobitz type II, complete AV block). The patients may experience a tachyarrhythmia-bradyarrhythmia syndrome with sinus node disease. Typically after the tachyarrhythmia with the sudden decrease in heart rate is the cause of the syncope. Syncope due to bradycardia or asystole is referred to as Stokes-Adams attack. Ventricular tacchyarrhythmias such as ventricular tacchycardia with a rate above 200/minute are more likely to cause syncope. Long QT syndrome is a hereditary disorder known to cause ventricular arrhythmias that cause syncope and even sudden death. Patients with dilated ventricles are more prone to ventricular arrhythmias. Answer B: This patient does not have risk factors for coronary artery disease such as diabetes, hypertension, and dyslipidemia. Also, there was no mention of chest pain prior or during the event. Answer C: Physical exam is unremarkable. It would be unlikely that the patient has structural heart disease such as aortic stenosis, hypertrophic cardiomyopathy, cardiac tumors, or aortic dissection due to the history and physical exam of the patient. Answer D: This patient did not have signs of seizure activity. Although the event was not witnessed, there was no mention of bowel or bladder incontinence as well as any post-ictal state regarding the patient's mental status. Additionally, there are a high rate of false negative tests when using an EEG to rule out seizure activity. Answer E: This patient has normal vital signs with normal orthostatic vitals. There is no signs of dehydration, no mention of medication use, and no history of autonomic disorders that may cause autonomic neuropathy such as diabetes or amyloidosis. Also, a big key point was the fact that the patient was sitting down during the event and did not change positions. That detail eliminates orthostatic hypotension as a cause. Orthostatic hypotension (intolerance) can be diagnosed with a drop in systolic blood pressure of 20 mmHg or a drop in diastolic blood pressure of 10 mmHg, although there are a large number of false positive results. Bottom Line: For a patient that has a normal physical exam with a syncopal event. It is important to rule out arrhythmias as a cause. A 24 hour Holter monitor should be performed.

A 41-year-old male was in a motor vehicle accident and sustained a traumatic brain injury. His initial hypotension was resolved with normal saline boluses. Several hours later the patient is noted to be making over 1 liter of urine every hour. Laboratory studies reveal his urine osmolarity to be 285 mmol/kg. The patient is given desmopressin and his urine output begins to decrease. The most likely diagnosis is A. central diabetes insipidus B. central pontine myelinosis C. cerebral salt wasting D.nephrogenic diabetes insipidus E. syndrome of inappropriate antidiuretic hormone hypersecretion

The correct answer is: A The patient has developed central diabetes insipidus (DI) as a result of his traumatic brain injury. In central DI, antidiuretic hormone (ADH) synthesis is disrupted due to injury of the hypothalamus or pituitary. Therefore, decreased release of ADH results in an inability to concentrate urine and subsequent polyuria with decreased urine osmolarity. Common etiologies including neurosurgery or brain trauma, infiltrative diseases and tumor. Treatment of central DI consist of ADH replacement in the form of desmopressin which can be given as subcutaneous, oral or nasal form. Other therapeutic modalities include low salt diet and a thiazide diuretic as well. Answer B: Central pontine myelinosis refers to cerebral edema due to rapid over correction of chronic hyponatremia. It is typically characterized with altered mental status including lethargy, obtundation, dysphagia and even paraparesis or coma. However, none of this symptoms are present in this patient making it an incorrect answer choice. Answer C: Cerebral salt wasting is characterized with inappropriately elevated urine osmolality and hyponatremia. It usually is seen as a consequence of acute brain injury like subarachnoid hemorrhage, but it is less common than SIADH. However, it does not lead to polyuria as present in this patient making it an incorrect answer choice. Answer D: Nephrogenic or peripheral diabetes insipidus refers to kidney's inability to concentrate the urine due to resistance to ADH at the collecting tubules. It presents with polyuria, nocturia, and decreased urine osmolarity like central diabetes insipidus. The most common etiology is genetic, drugs, electrolyte abnormalities like hypercalcemia and hypokalemia. Nephrogenic or peripheral DI will not respond to desmopressin because there is a defect in the ADH receptors at the level of the kidney, not ADH production, making it an incorrect answer choice. Answer E: SIADH presents with hyponatremia, hypoosmolality, and high urine sodium concentration. However, it is characterized by reduced urine output due to excess absorption of water by excess ADH secretion. Etiologies including brain trauma, drugs, tumor and paraneoplastic syndrome. It can present as polyuria only if patients also drink a lot of water which is not the case in this scenario making it an incorrect answer choice. Bottom Line: Central DI will cause excessive urine output, a decreased urine osmolality, and can be treated with desmopressin

A 65-year-old female presents to the emergency department for respiratory distress. Past medical history is pertinent for chronic obstructive pulmonary disease secondary to a long-standing history of tobacco abuse. Vitals reveal a temperature of 37.1oC (98.8oF), blood pressure of 146/85 mmHg, heart rate of 106/min, respiratory rate of 12/min, oxygen saturation of 86% on room air. Physical examination reveals tachycardia, with bilateral rales and wheezing upon auscultation. An arterial blood gas is obtained and reveals the following: pH 7.2 pCO2 60 mmHg pO2 65 mmHg Oxygen saturation 84% Base excess -2.0 mEq/L Bicarbonate 30 mEq/L Which of the following is most pertinent to manage at this time? A. hypercapnia B. hypoxia C. metabolic acidosis D. metabolic alkalosis E. tachycardia

The correct answer is: A The patient is having an acute COPD exacerbation. This is noted by the decreased respiratory rate, physical exam, and the ABG which confirms a CO2 retention issue. This causes an acute respiratory acidosis. COPD patients have difficulty breathing air out. This mechanism causes a retention of CO2. The issue that needs to be corrected first is to help the patient relieve that obstruction; otherwise respiratory failure develops. This is done by steroids, inhaled nebulizers (albuterol and ipratropium), and if needed non-invasive positive pressure ventilation (BIPAP) or invasive ventilation (intubate the patient and place on a ventilator). The steroids decrease inflammation, the nebulizers help open the airways (albuterol) and decrease mucous production (ipratropium), and the BIPAP or invasive ventilation assists the patient in ventilation to allow the CO2 to be exhaled. By correcting the respiratory acidosis the other issues will resolve (hypoxia and tachycardia). It is advised that for patients with a moderate to severe exacerbation be started on antibiotics. The antibiotic that should be chosen should be based on the local patterns of antibiotic susceptibilities. Answer B: The hypoxia is concerning and needs to be treated. However, this patient needs to have their ventilation issue resolved otherwise the patient will continue to retain more CO2. Supplemental oxygen should be applied to keep arterial oxygen saturation >90%. Hypoxemic respiratory drive plays a very small role in COPD patients. Studies have shown that supplemental oxygen does not reduce minute ventilation in both acute and chronic hypercapnia. Answer C: The patient's primary acid base disturbance is respiratory acidosis. The bicarbonate level would need to be low for a metabolic acidosis to occur. Answer D: The patient does not have an alkalosis issue. The patient's primary acid base disturbance is respiratory acidosis. This is due to the elevated PaCO2. It is apparent this issue has been going on for a while because the kidneys have tried to compensate by reabsorbing bicarbonate to compensate for the respiratory acidosis. Answer E: Managing the tachycardia would only mask the underlying issue. The patient's issue that need to be corrected is the COPD exacerbation. By treating the COPD the tachycardia will resolve. Bottom Line: In an acute respiratory acidosis that is caused by a COPD exacerbation, correct the ventilation issue first. This allows the other common symptoms (tachycardia, dyspnea, hypoxia) seen to resolve.

A 65-year-old male presents with left buttock pain for the last 3 months that has progressively worsened. His pain is not noticeable at rest, however, it increases with ambulation of more than 2 blocks. He is currently taking metformin, lisinopril, and hydrochlorothiazide daily. He is also asking for a trial of sildenafil because he has noted a recent decrease in his ability to maintain an erection. Social history is positive for a 30 pack-year history of cigarettes. Physical examination reveals atrophic toenails and a pulse that is 1+ to the left dorsalis pedis. Which of the following additional findings is an indicator for intervention? A. ankle-brachial index of 0.80 B. elevated C-reactive protein C. LDL-C of 175 mg/dL D.low homocysteine levels E. low factor VIII levels

The correct answer is: A This is a patient who is presenting with known risk factors for peripheral vascular disease including age >65, male gender, smoking history, and diabetes. The presenting symptoms in this case provide clues of arterial disease and claudication including onset of pain with a consistent walking distance, atrophic toenails, and change in hair growth. The location of the claudication is suggested by the arteries which provide tissue perfusion to the left leg, gluteal muscles, and the penis which is the cluster found with aortoiliac disease also known as Leriche's syndrome. Of the listed options, the low ABI is a clue that the there is a decreased blood pressure in the lower extremity compared to the upper extremity. ABI, or ankle brachial index, is a ratio of blood pressures comparing the systolic blood pressure taken from the calf divided by the ipsilateral arm systolic blood pressure. A ratio ABI greater than 1.0 is considered normal, while a ABI less than 1.0 represents the presence of arterial disease with less than 0.7 representing severe disease. Answer B: C-reactive protein is an inflammatory marker used to assess the degree of compliment activation. Use usefulness for assessing the degree of peripheral arterial disease for treatment options is less than direct measure of blood pressure by ABI, and measurable lumen flow and velocities by US. Answer C: Elevated LDL-C, low density lipoprotein, has been demonstrated to be an independent risk factor for cardiovascular disease making it a target for treatment goals in the prevention of corona arterial and vascular diseases. The use of statin medications to reduce cholesterol levels in patients with known peripheral arterial disease (PAD) has reduced the incidence of cardiovascular mortality. Additionally, results from the Heart Protection Study found a benefit in decreasing first time peripheral vascular events (ischemia) when patients with PAD were treated with statin medications as well. However, in this case, indications for intervention are limited to findings suggestive of present disease making ABI the correct answer. Answer D: Homocysteine is an amino acid found in the serum. It has been brought into question by many clinicians as well as the American Heart Association as an independent risk factor for vascular disease. If measured, however, it would be used more for risk calculation rather than treatment criteria for peripheral vascular disease. Answer E: Low factor VIII levels are often seen in von Willebrand's disease, a bleeding disorder. Since normal vWF protects factor VIII from degradation, low vWF levels can lead to deficient factor VIII. If factor VIII is decreased, it causes a prolonged aPTT. This patient would require further intervention if a clotting disorder were present, not a bleeding disorder. Bottom Line: Peripheral arterial disease is a limb threatening condition owing to the decrease in available oxygen-rich blood perfusion distal to the occluded site. Ankle-brachial index (ABI) is a useful, and highly sensitive, tool in the clinic setting used as a screen for disease and a measure of degree of disease. A low ABI, especially exercise ABI, should be further investigated with Doppler electrocardiography and considered for intervention.

A 45-year-old male with alcohol dependence presents to the emergency department with sudden onset of epigastric pain. The patient admits to binge drinking one night prior to presentation to the emergency department. Vital signs reveal a pulse rate of 98/min, respiratory rate of 18/min, and oxygenation saturation of 98% on room air. Abdomen examination has normal bowel sounds with moderate tenderness to palpation in the epigastric region. Lung fields are clear to auscultation bilaterally. Structural examination reveals a Chapman's reflex point in the seventh intercostal space on the right. Laboratory studies are obtained and reveal the following: Hemoglobin 15.2 g/dL Leukocyte count 14.2 x 103/mcL Hematocrit 45.6 % Alanine aminotransferase 35 U/L Aspartate aminotransferase 45 U/L Albumin 3.8 g/dL Total bilirubin 1.0 mg/dL Direct bilirubin 0.2 mg/dL The most appropriate initial step in the management of this patient is A. 0.9% normal saline B.abdominal CT scan with contrast C. endoscopic retrograde cholangiopancreatography D. immediate surgical consultation E. plain film abdominal radiography

The correct answer is: A This patient has a Chapman's point for the pancreas in the right seventh intercostal space. The two most common causes of pancreatitis in America include alcoholism and gall stones. A male patient with a history of alcoholism and recent binge drinking episode with epigastric pain radiating to the back is very suggestive of acute pancreatitis. Several studies have suggested that intra-acinar activation of proteolytic enzymes like trypsinogen into trypsin results in autodigestive injury to the pancreatic and peripancreatic organs. This starts the cascade of release of different enzymes and cytokines which results in vascular damage, interstitial hemorrhage and cell necrosis. Hence, amylase and lipase levels two to three times upper limit of normal are consistent with pancreatitis. Aggressive early hydration is a key step in the management of acute pancreatitis to prevent organ hypoperfusion secondary to third spacing of fluid from vascular damage. During the first 24-48 hours, patients are given nothing by mouth so that, theoretically, less pancreatic enzymes are stimulated. Answer B: In patients with worsening pancreatitis despite conservative treatment and/or severe pancreatitis at initial presentation, a CT abdomen can be helpful in predicting the extent of tissue necrosis and presence of a pseudocyst. Although most patients undergo a CT abdomen to find radiologic evidence of pancreatitis, it is not in fact indicated as an initial step. Answer C: Urgent endoscopic retrograde cholangiopancreatography (ERCP) is only indicated in patients with acute biliary pancreatitis with high suspicion for cholangitis. However, given normal liver function tests, biliary pancreatitis is unlikely, thus ERCP is not indicated as an initial step of management for this patient. Answer D: Surgical consultation is indicated with complications of severe pancreatitis including necrotizing pseudocyst formation and abdominal compartment syndrome. In this clinical scenario, there is no information suggestive of such complications. Thus, a surgical consultation is not an appropriate initial step at this point. Answer E: Peptic ulcer disease with possible perforation is a good differential diagnosis for this patient with acute epigastric pain. An upright abdominal radiograph is helpful in patients with an acute abdomen to rule out free air under the diaphragm from perforation. Although, chronic calcified pancreas can be visualized on x-ray, it is not the initial step in management in case of acute pancreatitis. Bottom Line: Early and aggressive fluid administration is the most important initial step in the management of acute pancreatitis.

A 32-year-old female presents to the emergency department complaining of low-grade fever, fatigue, and chest pain. The pain is located in the substernal region and is described as non-radiating and burning. Nothing seems to relieve the pain and swallowing makes it worse. A review of systems is positive for an unintended 9 kg (20-lb) weight loss within the past 6 months. Physical examination reveals cachexia and multiple white patches on the soft palate that extend into the esophagus. Scrapping of a lesion leaves a red, erosive base. The underlying condition responsible for this presentation is A. acquired immunodeficiency syndrome B. Burkitt's lymphoma C. chronic lymphocytic leukemia D. oral candidiasis E. squamous cell carcinoma

The correct answer is: A This patient is presenting with candidal esophagitis and wasting syndrome secondary to human immunodeficiency virus (HIV) infection and development of acquired immunodeficiency syndrome (AIDS). HIV is transmitted via semen and blood. Once it enters circulation, it targets and destroys host CD4+ T lymphocytes, weakening cellular immunity of the host over years to decades. High-risk individuals include homosexuals, bisexual men, IV drug users, and unborn or newborn infants of HIV positive women. The initial infection may be asymptomatic or present with vague flu-like symptoms (e.g., malaise, fever, headache, rash) and commonly goes undiagnosed. Patients will then remain asymptomatic for several years as their CD4+ T cell population is steadily destroyed (average rate of CD4+ T cell decline is ~50/µL per year). Clinical disease usually does not become apparent until the CD4+ T cell count falls below 200/ µL, placing the patient at risk for opportunistic infections and neoplasms. A diagnosis of acquired immunodeficiency syndrome (AIDS) is made in patients with HIV infection and a CD4+ T cell count < 200/ µL or who develops one of the HIV-associated diseases. The list of AIDS-defining illnesses is extensive but includes invasive infections with opportunistic pathogens (e.g, coccidioidomycosis, candidiasis, cryptococcosis, cryptosporidiosis, disseminated CMV disease, refractory HSV infections, histoplasmosis, Mycobacterium tuberculosis, Pneumocystis jiroveci, toxoplasmosis), wasting syndrome, HIV encephalopathy, and particular neoplasms (cervical cancer, Kaposi's sarcoma, CNS lymphoma, Burkitt's lymphoma). The three most commonly seen are pneumocystis pneumonia (40%), cachexia in the form of HIV wasting syndrome (20%) and esophageal candidiasis. HIV wasting syndrome is defined as involuntary weight loss of > 10% with either chronic diarrhea (2 or more stools/day for > 1 month) or fever and fatigue lasting > 30 days. Patients commonly have significant muscle wasting with myofiber degeneration and myositis. There is no definitive treatment for wasting syndrome, however; androgenic steroids, growth hormone, and total parenteral nutrition have been used with variable success. Thrush (oral candidiasis) presents with white, cheesy exudates on an erythematous base that can be scrapped off (unlike hairy leukoplakia). The presence of odynophagia and retrosternal/ substernal chest pain is highly suggestive of concomitant esophagitis and warrants further evaluation with upper endoscopy. Treatment is usually with nystatin swish and swallow. Prognosis is good and candidal infection usually resolves in 1-2 weeks without complications. This patient requires a thorough HIV work-up, investigation for other AIDS-defining illnesses, and should be started on highly active antiretroviral therapy (HAART) as soon as possible. Answer B: Burkitt lymphoma is a non-hodgkin's B cell lymphoma most commonly caused by a translocation mutation that places c-myc oncogene next to a heavy-chain Ig gene [t(8;14)]. Presentation varies depending on lesion location. Endemic Burkitt's lymphoma is commonly seen in adolescents and young adults in Africa and presents with a large mandibular or maxillary mass. Sporadic Burkitt's lymphoma is associated with abdominal and pelvic masses and may present with nausea, vomiting, loss of appetite, GI bleed, and/or renal failure. Almost all cases are associated with an underlying EBV infection. Underlying Burkitt's lymphoma is not associated with oral candidiasis or wasting syndrome. It is important to note that patients with HIV are at higher risk of developing Burkitt's lymphoma than the general population and approximately 40% of HIV-related non-Hodgkin lymphoma (NHL) cases are Burkitt lymphoma. Answer C: Chronic lymphocytic leukemia (CLL) is a hematological malignancy associated with monoclonal proliferation of mature lymphocytes that are functionally defective. Opportunistic infections such as candidal esophagitis and cachexia can be seen in patients with advanced CLL. However, CLL is unlikely in this patient because she does not fit the appropriate demographic (CLL is commonly seen in males > 60 years old) and she is not exhibiting any other symptoms associated with CLL ( e.g., generalized painless lymphadenopathy, splenomegaly). Answer D: This patient's presentation is consistent with both oral and esophageal candidiasis. Esophageal candidiasis would be more appropriate considering that it is an AIDS-defining illness. Additionally, the question asks for the underlying condition associated with the presentation, not for the diagnosis. Answer E: Squamous cell carcinoma of the head and neck commonly presents with a lump or sore that does not heal, a sore throat that does not go away, difficulty swallowing, and/or a change or hoarseness in the voice. Important risk factors include tobacco use, alcohol use, and human papillomavirus infection. This patient's history, presentation, and physical examination are more consistent with oral candidiasis and wasting syndrome, likely secondary to underlying HIV infection and AIDS. Bottom Line: The diagnosis of AIDS depends on the identification of particular AIDS-defining illness or on finding a CD4+ cell count < 200 micro liters in a HIV-1 seropositive patient. The list of AIDS associated diseases is extensive and patients may present with pulmonary, gastrointestinal, neurologic, cutaneous, or systemic symptoms. candidal esophagitis and wasting syndrome are two commonly seen conditions that indicate a severe defect in cell-mediated immunity due to HIV infection.

A 45-year-old female who is 6 months status post uncomplicated laparoscopic appendectomy for acute appendicitis presents with abdominal pain lasting 10 hours with radiation to the right shoulder blade. An abdominal ultrasound is obtained using a 4-MHz transducer which demonstrates: Common Bile Duct, diameter: 8 mm Pericholecystic fluid: present Gall bladder wall, thickness: 4 mm Which of the following is the most appropriate next step in management? A. computed tomography of the abdomen B. direct percutaneous biliary puncture C. endoscopic retrograde cholangiopancreatography D. laparoscopic cholecystectomy E. open cholecystectomy

The correct answer is: C Right upper quadrant (RUQ) postprandial pain with radiation to the right shoulder blade is classic for gallbladder disease. However, to rule in disease, RUQ ultrasound can be useful to help identify gallbladder pathology and rule out liver disease such as hepatitis. Findings on the ultrasound help dictate the correct next course of actions. In this case, the ultrasound is significant for a CBD dilation of 8mm which is dilated beyond normal range (normal mean CBD diameter is 4.1mm) suggesting the presence or recent passing of a gallstone. Presence of fluid, pericholecystic fluid, along with gallbladder wall thickening is suggestive of in inflamed gallbladder as seen in cholecystitis. Her history of prolonged pain also fits this scenario. Normal procedure would be surgical removal of the diseased gallbladder. However, given the CBD dilation which suggests the possibility of a recently passed stone that may still be in the CBD or in the pancreatic duct leading to a high risk of pancreatitis. Removing the gallbladder alone without attention to the stone would lead to high morbidity. Endoscopic retrograde cholangiopancreatography (ERCP) is an endoscopic procedure that allows the physician to visually identify the sphincter of oddi and remove contents found in the distal pancreatic duct. Once any obstruction is removed, the surgeon would proceed with surgery. Answer A: Abdominal CT is usually unnecessary in the diagnosis of acute cholecystitis, although it can easily demonstrate gallbladder wall edema associated with acute cholecystitis Answer B: Direct percutaneous biliary puncture is a useful approach to drain gallbladder when diseased from benign of malignant growth obstruction of the CBD. It has no use, however, in draining the distal CBD from stone obstruction. Answer D: This patient's history, physical exam, and diagnostic results indicated the need for surgical removal of the gallbladder. However, cholecystectomy has limitations with obstructed CBD and pancreatic duct. We also are not provided any information that indicates the patient has sepsis or an otherwise infected gallbladder, and the ultrasound does not reveal definitive gallstones. This patient's risk of pancreatic and/or hepatitis needs to be addressed first, followed by gallbladder removal. Answer E: This patient's history, physical exam, and diagnostic results indicated the need for surgical removal of the gallbladder. However, cholecystectomy has limitations with obstructed CBD and pancreatic duct. This patient's risk of pancreatitis and/or hepatitis needs to be addressed first, followed by gallbladder removal. Furthermore, laparoscopic approach has improved after the past decade significantly decreasing the risk of complications with rapid healing and shortened hospital stays making laparoscopic the preferred choice over open approach. Bottom Line: In the cases of acute cholecystitis, cholecystectomy is a necessary treatment. However, proper workup and intervention is needed when evidence of stone passing is identified by common bile duct dilation. ERCP is an important intervention as it can identify and remove obstruction to relieve congestion.

A 55-year-old male presents with an upper extremity tremor which has now progressed to a tremor in his head as well. History reveals the symptoms have been presents for a few years and have progressively worsened over time. He believes it to be exacerbated by stress and physical activity and relieved with alcohol ingestion. He vaguely recalls that one of his uncle's also had a tremor, but he does not recall any details regarding his condition. Past medical history is significant for diabetes and hypertension that are well controlled with medications. On neurological exam, the patient is alert, awake, and oriented to person, place, and time. No focal deficits are noted and sensation is intact to temperature and vibration throughout. Cranial nerves II-XII are intact and the patient has a normal, steady gait. A fine tremor of 4-6 Hz is noted in the upper extremities bilaterally when the arms are outstretched. The most likely diagnosis is A. alcohol withdrawal B. essential tremor C.Parkinson's disease D. physiologic tremor E. rubral tremor

The correct answer is: B Essential tremor is also known as benign familial tremor which is most often confused with Parkinson's disease. Essential tremor usually presents as bilateral hand tremor (frequency 4 to 6 Hz) that exacerbates with actions example lifting an object, writing, eating etc. In contrast, Parkinson's disease is usually worse at rest and better with activity. Essential tremor can progress to involve the neck which results in head oscillations as well, an unlikely finding in Parkinson's disease. Beta blockers, anticonvulsants and small amounts of alcohol have been used in the treatment of essential tremor. Essential tremors also have a familial basis with an autosomal dominant mode of inheritance, as suggested in the question stem by the patient's uncle having a history of a tremor as well. Answer A: Although the question stem gives us the information that the tremor improves with alcohol ingestion, it does not give any history of alcohol abuse to suggest dependence. In patients who are alcohol abusers, early signs of withdrawal include tachycardia and tremors. Asterixis can be seen in patients with advanced alcohol related liver disease complicated with hepatic encephalopathy. However, given the chronicity and familial history of tremors and lack of alcohol dependence, this is an unlikely diagnosis. Answer C: Parkinson's disease is linked to loss of dopaminergic neurons located in the substantia nigra pars compacta. Hallmark features of Parkinson's disease include rigidity, bradykinesia, mask facies, postural instability, shuffling gait, micrographia and tremors. Parkinson's tremor presents as a "pill-rolling" tremor, fine frequent movements (frequency 4 to 6 Hz) of the distal muscles of the hand. However, Parkinson's disease has a resting tremor which gets better with activity and worse with rest, which makes this a less likely diagnosis. Answer D: Physiologic tremor is a very low amplitude and high frequency (10 to 12 Hz) tremor that is usually invisible under normal daily circumstances but exaggerated in certain circumstances. It can be exaggerated by many drugs including anti-depressants, amphetamines, nicotine and many more. Other examples include increase sympathetic activity with anxiety, agitation and alcohol or opioid withdrawal. Although this patient's tremors get better with alcohol, it does not mean that they necessarily get worse with alcohol withdrawal, making this choice a less likely diagnosis. Answer E: Rubral tremor is result of a midbrain or cerebellar injury which produces a mixture of posture, intention and resting tremor with a low-frequency of 3 to 5 Hz only. In this question, we have a clear case of tremor that is worse with activity and better with rest, with no history of head trauma, which makes this a less likely diagnosis. Bottom Line: Essential tremors are benign familial tremors that are usually worse with activity and better with rest.

A 40-year-old male presents to the emergency department with sudden onset chest pain associated with a syncopal event. The pain is described as severe and sharp. History reveals he was mowing the lawn and lost consciousness for a brief moment after the onset of pain one hour ago. He also describes upper back pain, possibly from his fall. Home medications include hydrochlorothiazide for hypertension, omeprazole for gastroesophageal reflux disease, and metformin for diabetes. Social history is significant for a 35 pack-year history of smoking cigarettes. Vitals include the following: Blood pressure 200/110 mmHg Heart rate 130/min Respiratory rate 28/min Oxygen saturation 92% Physical examination reveals a heart that has a regular rate and rhythm without extra sounds, lungs with decreased breath sounds in the right lower lobe, and an abdomen that is soft and non-tender without masses or bruits. You note paravertebral muscle spasms and bogginess in the tissue surrounding T1-4 that is tender to palpation. A portable chest radiograph displays a mediastinal width of 8cm with an associated small pleural effusion in the right lower lobe. An electrocardiogram is obtained and reveals sinus tachycardia with nonspecific ST and T wave changes. Which of the following is the most likely diagnosis? A. acute myocardial infarction B. aortic dissection C. perforated peptic ulcer D. pericardial tamponade E. pulmonary embolism

The correct answer is: B In a patient presenting to the ER with acute chest pain, 6 fatal causes of chest pain called "serious six" must be ruled out which includes acute coronary syndrome, pericarditis with acute tamponade, aortic dissection, pulmonary embolus, pneumothorax, and esophageal rupture. Physicians must rapidly evaluate patients and rule out each of these diagnosis based on history and physical examination. The acute chest pain, history of smoking (at risk for aneurysm), chest pain radiating to the back, and widened mediastinum is highly suggestive of aortic dissection. The right sided pleural effusion is most likely secondary to hemothorax from acute aortic dissection as well. His history of heavy tobacco use and hypertension put him at increased risk for aortic dissection. The next best step in diagnosis would be to get an emergent CT chest to establish early diagnosis and proceed with medical or surgical management depending on the type of dissection. Type A dissections involving the aortic arch are taken for emergent surgery whereas Type B dissections are managed medically with beta blocker therapy. As with any cardiac pathology, you may see acute TART changes from T1-4 as a result of viscerosomatic reflexes along the sympathetic innervation to the heart and aorta. Answer A: Severe chest pain in a middle-aged man with multiple risk factors for coronary disease should always raise your suspicion for an acute coronary syndrome; however in this case, his ECG shows only sinus tachycardia (no specific ST segment changes). Given lack of specific ST segment changes along with widened mediastinum, aortic dissection is a more likely diagnosis. Answer C: Perforated peptic ulcer would present with free air under the diaphragm, not a widened mediastinum. There is no mention of free air under the diaphragm in the question stem making it a less likely answer choice. Answer D: Cardiac tamponade classically presents with the triad of (1) distended neck veins (2) muffled heart sounds and (3) hypotension, which is not consistent with this patient's presentation. ECG may show electrical alternans. However, it is part of the serious six which must be ruled out upon initial presentation of the patient to the ER. Answer E: Pulmonary embolism (PE) can present with mild dyspnea and pleuritic chest pain to severe acute right-sided heart failure and death, depending on the size and location of the embolus. Sinus tachycardia as present in this patient is usually the most common presentation of PE followed by S1Q3T3 pattern. PE on chest x-ray can have non-specific findings including normal, pleural effusion or infarct; however, it would not explain this patient's widened mediastinum. Bottom Line: Suspect aortic dissection in a patient presenting with sudden, severe, "tearing" chest pain radiating to the back with a widened mediastinum on chest imaging.

A 56-year-old male presents with chest discomfort and fatigue. History reveals that he recently started a prednisone taper for an acute exacerbation of chronic obstructive pulmonary disease. Past medical history is also pertinent for osteoarthritis for which he takes ibuprofen as needed. Structural examination reveals tissue texture changes from T5-T9 on the left. The most likely cause is A. esophagitis B. gastritis C. pancreatitis D.pericarditis E. pleuritis

The correct answer is: B Peptic ulcer disease may be secondary to Helicobacter pylori, NSAIDs, elevated acid secreting states, and Crohn's disease. In this question stem it mentions that the patient was taking ibuprofen chronically for joint pain. This medication already increases the likelihood for peptic ulcers. However, this was significantly increased when the patient was placed on corticosteroids. This process interrupts the production of prostaglandins which protects the gastric lining by inhibiting the COX enzyme, specifically COX-1. Therefore allowing the gastric acid to eat away at the mucosal lining causing a peptic ulcer. The patient would be recommended to stop NSAID use and be placed on antacid therapy. The somatic dysfunction is correlated with the upper GI tract that is anything before the ligament of Treitz. Specially, T5-T10 on the left correlates to the stomach. Answer A: Gastroesophageal reflux disease (GERD) is due to impaired clearance of gastric juice through the lower esophageal sphincter. This may cause esophagitis. The symptoms and physical signs are consistent with burning chest discomfort, excessive belching, hoarse voice, chronic cough, asthma, and dental erosions. This patient does not display these symptoms and is at high risk for peptic ulcer disease due to NSAID and corticosteroid therapy. First line treatment for GERD is lifestyle modifications; avoidance of foods that lower the esophageal sphincter pressure (spearmint, peppermint, coffee, tea, fatty foods, alcohol, and tomato based foods), avoidance of acidic foods, and adoption of behaviors to minimize symptoms. If the patient does not improve with lifestyle modifications they may be placed on an H2 blocker or PPI therapy. Answer C: The role of glucocorticoids in causing acute pancreatitis is uncertain. Pancreatitis usually presents with severe epigastric abdominal pain that radiates toward the back. Most common causes in the United States are secondary to gallstones and alcohol abuse. The patient did not present with a history of right upper quadrant pain and there is no mention in the history regarding gallstones or recent alcohol use. Treatment for pancreatitis is usually supportive care that consists of pain management, intravenous fluids, and nothing by mouth. Answer D: Steroid therapy can increase the risk of cardiovascular events such as coronary artery disease or heart failure, but it is not likely related to pericarditis because aspirin and NSAIDs are the standard treatment for this condition. Common drugs that cause pericarditis include: procainamide, tocainide, hydralazine, isoniazid, methyldopa, and phenytoin. Answer E: Pleuritis is an inflammation of the parietal and serous pleura of the lung, but it is not commonly related to NSAIDs or steroid therapy. Drugs that may cause SLE or RA-related pleuritis include procainamide, hydralazine, and isoniazid. Bottom Line: Know the causes of peptic ulcer disease and the risks with NSAIDs and steroids. NSAIDs with corticosteroids dramatically increase the risk of peptic ulcer disease and complications.

A 13-year-old male recently emigrated from China after being diagnosed with congestive cardiomyopathy would have a deficiency in... A. calcium B. selenium C. vitamin B1 D. vitamin B3 E. zinc

The correct answer is: B Selenium deficiency is seen in some parts of China, New Zealand and Finland. Keshan disease presents as congestive cardiomyopathy with an enlarged heart in children and young females living in these endemic areas with low soil concentration of selenium. Some selenium rich foods include fish, shellfish and eggs. Answer A: Acute calcium deficiency can present with various manifestations of neuromuscular dysfunction including peri-oral numbness, paresthesias, muscle spasms, muscle cramps and seizures. Answer C: Recognize that in developed countries the most common cause of chronic thiamine (B1) deficiency is alcoholism. Patients can present with horizontal nystagmus (abnormal eye movements), ophthalmoplegia, cerebellar ataxia (lack of co-ordination while walking) and mental impairment (dementia) along with confabulation psychosis termed Wernicke-Korsakoff syndrome. Answer D: Underdeveloped countries with mainly corn and carbohydrate based diet deficient in Niacin (B3) can develop a "casal's necklace" rash. A hyperpigmented erythematous scaly rash just around the neck in the sun exposed areas. 3 D's associated with niacin deficiency include diarrhea, dermatitis and dementia Answer E: In Middle Eastern countries, chronic zinc deficiency can result in stunted growth in children with hypopigmented hair. Zinc deficiency is also associated with impaired immunity and decrease taste sensation (hypogeusia). Some foods rich in zinc include oysters, beef, crabs and cereal.

A 40 -year-old male presents to the emergency department after being an unrestrained passenger in a motor vehicle accident. He is non-responsive to noxious stimuli for eye opening, verbal response or motor responses. An electroencephalogram is obtained revealing auditory brainstem response. The most likely diagnosis is A. brain death B. coma C. locked-in syndrome D.minimally conscious state E. vegetative state

The correct answer is: B The Glasgow Coma Scale (GCS) is a neurological scale that provides an objective assessment of assessing and recording a patient's level of consciousness using eye opening, verbal and motor responses. Patients who have no response to stimuli in eye opening, verbal or motor response have a GSC score of 3, compared to a patient with spontaneous eye opening, oriented verbal responses and follows motor commands has a GCS score of 15. Coma is a complete failure of the arousal system with no spontaneous eye opening and inability to be awakened by application of a vigorous sensory stimulus. Reflexes and postural responses are intact. Auditory brain responses test the brain stem and can be normal or abnormal in coma patients. An abnormal auditory brain response with a GSC of 3 has an increased risk of death compared to patients with a normal response. Answer A: Brain death presents with no response to pain and no cranial nerve reflexes, including pupillary response, oculocephalic reflex, corneal reflex, no response to the caloric reflex test, and lack of spontaneous respiration. Electroencephalography (EEG) including auditory brain responses of the brainstem are negative for at least 24 hours, and after all other reversible causes have been ruled out. Answer C: Locked-in syndrome presents with anarthria and quadriplegia with general preservation of cognition. Patient has an intact sleep-wake cycle and vertical eye movement and blinking. Using adaptive technology, patient can use the vertical eye movement to communicate. Answer D: The minimally conscious state is a state of inconsistent behavioral partial consciousness. There is presence of a sleep/ wake cycle, spontaneous eye opening with sustained visual fixation and the ability to localize to sound a noxious stimuli. Patients may reach for objects and perform automatic movements (such as scratching) but inconsistently follow commands. Patient also have some intelligible and contingent verbalization or gestures (smiling, crying) but inconsistent answers to yes/ no questions. Answer E: Vegetative state is a complete absence of behavioral self or environmental awareness. Patient has a preserved capacity for spontaneous or stimulus induced arousal, such as the sleep-wake cycle. Patient postured or withdrawals to noxious stimuli. Bottom Line: Coma is a complete failure of the arousal system with a positive auditory brain responses but no response in eye opening, verbal response or motor response to stimuli.

A 57-year-old male presents to the office complaining of shortness of breath and a dry cough. A plain film chest radiograph displays evidence of bilateral hilar adenopathy. Physical examination is consistent with a pretibial rash on his bilateral lower extremities. Structural examination reveals tissue texture abnormalities from T1-T4. The patient also had a recent history of being treated for an eye infection. A bronchoscopy is performed and biopsies are taken of the hilar nodes. Which of the following is most likely to be seen on histopathology? A. acid fast bacilli B. leukocytoclastic vasculitis C. non-caseating epitheloid granulomas D.poorly differentiated squamous cells E. Reed-Sternberg cells

The correct answer is: C Sarcoidosis is a disease in which abnormal collections of chronic inflammatory cells (granulomas) form as nodules in multiple organs. These granulomas are focal, chronic inflammatory reaction with accumulation of multinucleated giant cells, epithelial cells , lymphocytes and macrophages that gradually resolve. These non-caseating granulomas most often appear in the lungs or the lymph nodes, but virtually any organ can be affected. The most common presenting symptoms of sarcoidosis are dyspnea and dry cough. Bilateral hilar adenopathy is commonly associated with sarcoidosis and the diagnosis can be confirmed with a biopsy via bronchoscopy or open lung biopsy. Sarcoidosis is associated with other clinical entities such as erythema nodosum and uveitis as well. Acute sarcoid arthritis can present as Lofgren's syndrome characterized by the triad of hilar lymphadenopathy, erythema nodosum and arthralgias. First line treatment for sarcoidosis is usually corticosteroids. Answer A: Acid fast bacilli on histopathology is seen in infection by Mycobacterium Tuberculosis. TB granulomas have a characteristic caseation "cheese-like" necrosis within the granuloma that consist of epithelioid macrophages, langhans giant cells, and lymphocytes. Therefore, these granulomas are called "caseating granulomas" which is not consistent with findings in sarcoidosis. Also, although TB can also present with dry cough, this patient's presentation of bilateral hilar lymphadenopathy with erythema nodosum is more consistent with sarcoidosis. Answer B: Leukocytoclastic vasculitis refers to inflammation of the small blood vessels without any immune deposits. It is typically associated with microscopic polyangitis, polyarteritis nodosa, cryoglobulinemia and hepatitic C infection. However, it is not associated with sarcoidosis making it an incorrect answer choice. Answer D: Poorly differentiated squamous cells can be observed in cancer with unknown primary site or lung cancer. However, there is no history of weight loss, night sweats or tobacco abuse to suggest presence of lung cancer making this answer choice incorrect. Answer E: Reed-sternberg cells are clonal malignant cells that are pathognomonic for hodgkin's lymphoma. These cells are not typically found in sarcoidosis, making it an incorrect answer choice. Bottom Line: Sarcoidosis presents with a dry cough and bilateral hilar adenopathy.

70-year-old male presents for his annual physical examination. His past medical history is positive for hypertension and hyperlipidemia. History reveals he started smoking age 20 and has a 40 pack-year smoking history, quitting five years ago. He denies any weight changes, cough, shortness of breath, or hemoptysis. Which of the following is the most appropriate recommendation at this time? A. bronchoscopy B. helical CT scan of the chest C. MRI of the chest without contrast D. observation E. plain film radiography of the chest

The correct answer is: B The U.S. Preventive Services Task Force (USPSTF) now has strong (Grade B) evidence for screening for lung malignancy. Their recommendations are: annual screening for lung cancer with low-dose computed tomography in adults ages 55 to 80 years who have a 30 pack-year smoking history and currently smoke or have quit within the past 15 years. Screening should be discontinued once a person has not smoked for 15 years or develops a health problem that substantially limits life expectancy or the ability or willingness to have curative lung surgery.

A 46-year-old HIV positive male complains of shortness of breath and profuse epistaxis. He has suffered from several episodes of bloody diarrhea and is experiencing pain in the left flank. Physical examination reveals a diffuse petechial rash, a non-focal neurological exam, and tissue texture changes from T9-L1 bilaterally. His stool is found to be positive for hemoccult. A peripheral blood smear shows the presence of schistocytes. Routine laboratory studies reveal the following: Platelet count 90,000/mm3 Serum creatinine 4.6 mg/dL LDH 4503 u/dL Indirect bilirubin 2.5 mg/dL Hemoglobin 6.3 g/dL Prothrombin time 12.4 sec Partial TT. 31.2 sec What is the most likely diagnosis? A. disseminated intravascular coagulation B. hemolytic uremic syndrome C. hereditary spherocytosis D. idiopathic thrombocytopenia E. thrombotic thrombocytopenic purpura

The correct answer is: B The classic presentation of hemolytic uremic syndrome (HUS) consists of the triad of anemia, thrombocytopenia, and acute renal failure, which parallel the laboratory findings above. Hemolytic uremic syndrome produces a micorangiopathic hemolytic anemia with a hemoglobin level that is typically less than 8 g/dl. This is a consistent finding and is necessary to establish the diagnosis. The hallmark of hemolytic uremic syndrome in the peripheral smear is the presence of schistocytes. These consist of fragmented, deformed, irregular, or helmet-shaped RBC's and reflect the partial destruction of RBC's that occurs as they traverse vessels partially occluded by platelet and hyaline microthrombi. The peripheral smear may also contain giant platelets. This is due to the reduced platelet survival time resulting from the peripheral consumption/destruction. Additionally, other laboratory tests such as the elevated LDH and indirect bilirubin are also characteristic of HUS as presented in the case above. However, a key distinguishing factor from TTP is the presence of bloody diarrhea often precipitated by E. coli O157:H7 or Shigella. This is a key clinical presentation in the setting of HUS. Answer A: Disseminated intravascular coagulation (DIC) can also present with diffuse bleeding, schistocytes and hemolysis, making it difficult to differentiate from TTP/HUS. However, DIC is associated with an increase in both PT and PTT and bleeding usually occurs at venipuncture sites. Answer C: Hereditary Spherocytosis is a genetic disorder of the proteins spectrin and anykrin, which are principally involved in allowing erythrocytes to maintain their bi-concave shape. If these proteins are dysfunctional then the erythrocytes will be spherocytes and are subject to hemolytic stress. Hereditary spherocytosis does not present with signs and symptoms consistent with microangiopathic hemolytic anemia (MAHA), but does have evidence of hemolysis on laboratory evaluation (e.g. elevated LDH and indirect bilirubin). Thus, hereditary spherocytosis will not present with schistocytes seen on peripheral smear. Answer D: Idiopathic thrombocytopenia (ITP) is typically a quiescent thrombocytopenia that does not present with anemia and certainly not hemolysis. Patients with ITP only develop anemia if there is severe bleeding associated with the thrombocytopenia. Answer E: Thrombotic Thrombocytopenic Purpura (TTP) also presents with fever, anemia, and acute renal failure, however TTP characteristically elicits focal neurological deficits which are not present in this case. Both TTP and HUS can present with elevated LDH, indirect bilirubin, and schistocytes on peripheral smear (consistent with MAHA). The distinguishing factor in the above case is the prodromal bloody diarrhea and lack of neurological deficits, both of which are consistent with HUS, not TTP. TTP can be remembered using the mnemonic FATRN, F-Fever A-microangiopathic hemolytic Anemia, T-Thrombocytopenia, R-Renal failure N-Neurological manifestations. Bottom Line: Hemolytic uremic syndrome is characterized by the triad of microangiopathic hemolytic anemia, thrombocytopenia, and acute renal failure. (MAT)

A 28-year-old female presents with altered level of consciousness. Vitals reveal a blood pressure of 85/56 mm Hg, heart rate of 125 beats/minute, respiratory rate of 25 breaths/minute, oxygen saturation 96% on 2L of nasal cannula, and a temperature of 39.8°C (103.6°F). Observation reveals the patient is awake, appears disheveled, and has difficulty following commands. Physical examination reveals a normocephalic and atraumatic head, reactive pupils that are equal in size, intact extraocular movements, and moist mucous membranes of the nares and oral cavity. Auscultation reveals tachycardia with a regular rhythm and no murmurs, rubs, or gallops. Rhonchi are appreciated in the right lower lobe with associated positive egophony and a negative percussion exam. A chest xray reveals a lower lobe pneumonia. Extremities are absent for edema with pulses of +1/4 in all four extremities. A complete blood count is obtained as follows: Erythrocyte count 4.6 x 106/mcL Leukocyte count 17.5 x 103/mcL Hemoglobin 14 g/dL Hematocrit 42% Platelet count 275 x 103/mcL Mean corpuscular Volume 88 mcm3 Red blood cell distribution width 12.5% A reflex blood culture is obtained and results are negative. The most likely diagnosis is A. bacteremia B. sepsis C. septic shock D. severe sepsis E. systemic inflammatory response syndrome

The correct answer is: B The patient has signs of infection with a known microbial etiology as noted in the blood culture. This was caused by presumed Streptococcus pneumoniae due to the respiratory exam and the blood cultures. The vitals are a clue to giving us the answer of how severe the infection is along with the patient's clinical exam. The patient is having difficulty following commands which leads us to the understanding that there is decreased oxygen delivery to the organs as revealed by the hypotension. This is caused by the sepsis, which causes a buildup of lactic acid due to anaerobic respiration secondary to decreased oxygen delivery as a consequence of the hypotension. The body is trying to overcome this by the response of tachycardia. SIRS is defined as 2 or more of the following variables. - Fever of more than 38°C (100.4°F) or less than 36°C (96.8°F) - Heart rate of more than 90 beats per minute - Respiratory rate of more than 20 breaths per minute or arterial carbon dioxide tension (PaCO 2) of less than 32 mm Hg - Abnormal white blood cell count (>12,000/µL or < 4,000/µL or >10% immature [band] forms) Sepsis is SIRS with a source of infection. In this case, the source of infection is in the lungs. Severe sepsis is sepsis with end organ damage however this definition is being phased out. Septic shock occurs if an individual has persistent hypotension and perfusion abnormalities despite adequate fluid resuscitation. Answer A: Bacteremia is the presence of bacteria in the blood as evidence of positive blood cultures. Answer C: Septic shock occurs if an individual has persistent hypotension and perfusion abnormalities despite adequate fluid resuscitation. Answer D: Severe sepsis is an old definition and is currently not recommended in medicine as of 2016. In general, it may take a few years for the board to reflect this update but it is important to be aware of this. According to a study released in JAMA, absent from the new sepsis definitions is the term "severe sepsis" - a significant change from previous definitions. The task force has deemed this term redundant, as sepsis has a mortality rate of 10 percent or higher, making the condition already severe Answer E: Systemic inflammatory response syndrome (SIRS) is made by 2 or more of the following Temperature >38°C or <36°C, heart rate >90 beats/min, respiratory rate >20 breaths/min, and leukocytosis >12x103/mcL, leukopenia <4 x103/mcL or >10% bands. However, this is made if the patient does not have a known source or suspected source of infection. The source is known due to the blood cultures and we can see from physical exam that the patient has a pneumonia that led to the sepsis. Bottom Line: Commit to memory the following for all COMLEX-related examinations. SIRS requires 2 of 4 of the following: temperature >38°C or < 36°C, respiratory rate >24/min, heart rate > 90/min, and leukocyte count > 12x103/mcL or < 4x103/mcL or >10% bands. Sepsis is SIRS with a proven or suspected microbial etiology. Severe sepsis is Sepsis in a patient with signs of organ dysfunction (hypotension systolic blood pressure < 90 mmHg or mean arterial pressure < 70 mmHg that responds to fluids). Septic shock is severe sepsis unresponsive to fluid resuscitation for at least 1 hour despite adequate fluid resuscitation, therefore requires vasopressors.

A 65-year-old female presents to the emergency department complaining of sudden-onset, right-sided chest pain that began one hour ago. The pain is worse with deep inhalation, and she denies cough, orthopnea, and leg swelling. Past medical history is significant for hypertension, congestive heart failure, and breast cancer. Vitals reveal a pulse of 113/min, respirations at 25/min, blood pressure of 136/84 mmHg, and oxygen saturation at 90% on room air. Her saturation increased to 97% with 3 L oxygen via nasal cannula. Physical examination is significant for crackles at the lower right lung field. There is no evidence of jugular venous distention, ascites, or peripheral edema on examination. A plain film radiograph of the chest is unremarkable. An electrocardiogram reveals tall, peaked P waves in lead II, right axis deviation, and sinus tachycardia. Which of the following is the most appropriate next step in management? A. inferior vena cava filter B. low-molecular-weight heparin C. percutaneous embolectomy D. tissue plasminogen activator E.warfarin

The correct answer is: B The patient in this clinical vignette has a pulmonary embolus (PE). Risk factors for PE include age > 60, malignancy, prior deep vein thrombosis or PE, hereditary hyper coagulable states (factor V leiden, protein C or S deficiency, antithrombin III deficiency), prolonged immobilization or long distance travel, cardiac disease (especially CHF), obesity, nephrotic syndrome, major surgery (especially pelvic or orthopedic), major trauma, pregnancy, and estrogen use. This patient had three risk factors, including age > 60, breast cancer, and CHF, placing her at risk for PE development. Clinical manifestations of pulmonary embolism vary and are nonspecific. Signs and symptoms include dyspnea, pleuritic chest pain, cough, hemoptysis, tachycardia, and rales. Although the majority of emboli originate from lower extremity deep vein thromboses, only approximately one-third of patients with PE will have signs and symptoms of DVT. The Modified Wells Criteria is commonly used to determine whether imaging for PE in appropriate. It consists of a list of factors assigned points. If a patient has a Modified Wells score > 4 then PE is clinically likely and Spiral CT should be performed immediately. If their score is < 4, then PE is unlikely and a d-dimer lab study can be used to rule out PE. Electrocardiogram is nonspecific in patients with PE and the most common finding is sinus tachycardia. However, electrocardiogram is still important in the work-up of PE to rule out cardiac causes of chest pain and shortness of breath (this patient's demonstrates sinus tachycardia with no ST segment changes). Management include administration of supplemental oxygen and acute anticoagulation therapy with either unfractionated or low-molecular-weight heparin. Oral warfarin therapy should be started on the same day. Heparin therapy can be discontinued after the international normalized ratio (INR) is 2.0 for at least 24 hours, but no sooner than 5 days after warfarin therapy has been started. This is done to prevent a paradoxical hypercoagulable state that may occur as a result of the relatively long half-life of factor II, along with the short half-lives of protein C and protein S. Answer A: Indications for inferior vena cava (IVC) filter placement include an absolute contraindication to therapeutic anticoagulation and failure of anticoagulation in the presence of acute proximal deep vein thrombosis (DVT). In a patient with a massive pulmonary embolism, prevention of further embolic events is of obvious desire; however, the evidence for IVC placement in these patients is less conclusive. This patient also does not have any contraindications to anticoagulation. Answer C: Catheter-guided or surgical embolectomy is a reasonable treatment option for massive pulmonary embolism in patients who have contraindications to fibrinolysis or who remain unstable after receiving fibrinolysis. The principal criteria for categorizing PE as massive are arterial hypotension and cardiogenic shock. Embolectomy may also be considered in patents with submassive pulmonary emboli who appear clinically unstable and at risk of poor prognosis (ie, new hemodynamic instability, worsening respiratory failure, severe right ventricular dysfunction, or major myocardial necrosis). This patient is hemodynamically stable and there is no evidence of right ventricular dysfunction on examination ( jugular venous distention, ascites, peripheral edema). Therefore, embolectomy is not warranted in this patient. Answer D: Due to potential serious bleeding consequences, thrombolytic therapy (e.g., streptokinase, tissue plasminogen activator) should be reserved for patients with confirmed pulmonary embolism and persistent hypotension, severe hypoxemia, large perfusion defect, or evidence of right ventricular strain. Answer E: Anticoagulation with warfarin therapy is the appropriate long-term treatment for pulmonary embolism. oral warfarin can be given with heparin at the time of diagnosis. However, it takes 36-72 hours to begin having a therapeutic effect and titration can be difficult. INR is used to assess the adequacy of warfarin therapy. The recommended therapeutic range for venous thromboembolism is an INR of 2-3. Patients on warfarin must have INR checked every 1-2 weeks to make sure they remain in the therapeutic range at their current dose. Duration of warfarin therapy ranges depending on cause and recurrence of PE.A patient with a first thromboembolic event occurring in the setting of reversible risk factors, such as immobilization, surgery, or trauma, should receive warfarin therapy for at least 3 months. Recurrence of PE requires longer duration of therapy, usually for 12 months. Patients who are at risk for multiple recurrent PE (e.g., malignancy, hypercoagulable states) may be considered for lifelong anticoagulation. Bottom Line: Initial management of pulmonary embolism includes supplemental oxygen, acute anticoagulation with unfractionated or low-molecular-weight heparin, and initiation of long-term anticoagulation therapy with oral warfarin. Thrombolytic therapy and embolectomy should be considered in patients with massive PE who are hemodynamically unstable or have evidence of right heart failure.

A 15-year-old male presents to the emergency department after losing consciousness during a soccer game. Patient was not noted to have any seizure like activity and bowel or bladder incontinence. He has had two prior episodes of dizziness and near syncope earlier in the week at practice but attributed it to dehydration. He denies any vision changes, headache, chest pain or dyspnea. Family history is unremarkable for any diseases in parents but positive for an uncle who died at the age of 35 for unknown reasons. Vitals are obtained and reveal the following: Heart rate 90/min Blood pressure (sitting) 110/65 mmHg Blood pressure (standing) 95/56 mmHg Physical examination is significant for a grade 2/6 systolic murmur heard best over the left sternal border. The murmur improves with squatting. You note a well circumscribed, tender nodule on the left lateral border of the sternum in the intercostal space between rib 2 and 3. Which of the following is the most likely underlying cause of this patient's syncope? A. bicuspid aortic valve B. hypertrophic cardiomyopathy C. Marfan syndrome D. second degree heart block E. vasovagal syncope

The correct answer is: B The patient in this vignette is a young, male athlete who experienced syncope during vigorous exercise with a systolic murmur on examination which is suggestive of hypertrophic cardiomyopathy (HCM). HCM refers to asymmetric hypertrophy of the left ventricular wall causing left ventricular outflow obstruction. Symptoms vary from chest pain, dyspnea, syncope to sudden cardiac death depending on the extent of dynamic outflow obstruction. Loss of consciousness in these patients is thought to be related to sudden left ventricular outflow obstruction and rapid drop in cerebral perfusion pressure. The degree of LV outflow obstruction in these patients is worsened by factors that reduce left ventricular end diastolic volume (preload), including tachycardia and dehydration in the setting of vigorous exercising in this vignette. Other maneuvers that decrease preload like valsalva and standing cause an increase in the intensity of the murmur. Maneuvers that augment venous return and increase preload, such as leg elevation or squatting, decrease the murmur. The history of sudden death in a male relative is also suggestive of this diagnosis. Sudden cardiac death in patients with HCM is typically the result of ventricular arrhythmias. Diagnosis is made by echocardiography which reveals left ventricular hypertrophy with or without left ventricular dynamic outflow obstruction. Treatment options include beta blockers, defibrillator and septal myotomy. As with any other cardiac pathology you may have osteopathic findings that help confirm diagnosis and can be important for management of the patient. In this case you are provided with the anterior Chapman point (left 2nd ICS) for the heart. You may also see TART changes from T1-4 as a result of viscerosomatic reflexes from the underlying cardiac pathology. Answer A: Bicuspid aortic valve is an autosomal dominant condition and is one of the most common types of congenital heart defects. The most common abnormal heart sound associated with a bicuspid aortic valve is a systolic ejection click. Patients are at risk of developing aortic stenosis, aortic dilatation, and aortic dissection. Answer C: Marfan syndrome is associated with mitral valve prolapse and aortic root disease leading to aneurysmal dilatation, aortic regurgitation, and dissection. Manifestations include arachnodactyly, pectus deformity, hindfoot valgus, scoliosis, and an abnormal arm span vs. height such that arm length is much longer than expected. Answer D: Second degree heart block with Mobitz type II is characterized by constant prolonged PR interval with random dropped beats. It can also present with syncope but rarely causes sudden cardiac death if it advances to an advanced degree block. However, it does not present with a systolic murmur on examination as present in HCM. Answer E: Although this patient has a history of dehydration with long vigorous practices in the summer, he has negative orthostatic vitals. Positive orthostatic changes include a decrease in SBP by 20 mm Hg, DBP by 10 mm Hg and increase in HR by 10/min. Negative orthostatic vitals with positive family history of sudden cardiac death makes vasovagal syncope a less likely diagnosis than HCM. Bottom Line: Suspect hypertrophic cardiomyopathy in a young male athlete with syncope and family history of sudden death.

A 40-year-old female presents complaining of painful blisters in her mouth that started two months ago. Over the past few days, similar painful blisters have appeared on her abdomen and chest. She denies any recent medication use. Vital signs include: Temperature 37.3oC (99.2oF) Blood pressure 110/70 mmHg Respiratory rate 16/min Heart rate 80/min Physical examination reveals multiple blisters in the oral mucosa, trunk, and groin area. Firm sliding pressure with a finger separates normal-appearing epidermis, producing an erosion. Total body surface area involvement is approximately 30%. She became concerned when she noticed sloughing of the skin upon touching the blisters. The most likely diagnosis is A. bullous pemphigoid B. pemphigus vulgaris C. Staphylococcal scalded skin syndrome D.Stevens-Johnson syndrome E. toxic epidermal necrolysis

The correct answer is: B This patient has a positive Nikolsky's sign which refers to sloughing and separation of the epidermis upon gentle pressure to the lesions. Four diseases that are associated with a positive Nikolsky's sign include pemphigus vulgaris, toxic epidermal necrolysis, Stevens-Johnson syndrome and Staphylococcal scalded skin syndrome. Pemphigus vulgaris is the most common subtype of pemphigus disease. Three other subtypes include pemphigus violaceus, IgA pemphigus, and paraneoplastic pemphigus. Pemphigus vulgaris is an intraepidermal autoimmune disease with IgG autoantibodies directed against desmoglein resulting in acantholytic epidermal cells. Due to loss of adhesion between epidermal cells, the skin layer comes off easily upon only gentle touch, giving a positive Nikolsky's sign in this patient. Mucosal membranes are almost always involved in addition to the scalp, trunk, neck, axilla, groin and face. It usually affects younger population compared to bullous pemphigoid which commonly affects elderly. Clinical symptoms include pain, pruritus and secondary skin infections, which if left untreated can be fatal. Mucosal lesions may precede cutaneous lesions by weeks or months. Glucocorticoids are the first line treatment for pemphigus vulgaris. Answer A: Bullous pemphigoid is a subepidermal autoimmune disease with autoantibodies directed against the basement membrane of the epidermis (between dermis-epidermis), as compared to the epidermal cell junctions (epidermis-epidermis) in pemphigus vulgaris. Therefore, nikolsky sign is negative as the blisters are deeper and epidermis does not separate easily on touch. Symptoms are similar to pemphigus vulgaris including pruritus, urticaria and blister formation. However, blisters form most commonly over the trunk, groin, and the flexor surfaces and less often in the oral mucosa. Uncomplicated blisters often heal without any scarring whereas in pemphigus vulgaris blisters often heal with hyperpigmented spots. Bullous pemphigoid also generally affects the elderly as compared to the younger age group in pemphigus vulgaris. However, both diseases are treated with glucocorticoids. Answer C: Staphylococcal scalded skin syndrome (SSSS) is a blistering disease caused by exotoxins released by Staphylococcus aureus resulting in detachment of the epidermal layer. Therefore, a positive nikolsky sign is an important clinical finding. SSSS can present anywhere from a few small blisters after a local impetigo infection to a complete exfoliation of the skin, depending on the severity of the infection. Patients can present much sicker with constitutional symptoms including fever, lethargy, and poor feeding. Treatment includes targeting the underlying infection with antibiotics along with aggressive hydration and supportive care. However, it is typically found in newborn and children making it a less likely diagnosis. Answer D: Stevens-Johnson syndrome (SJS) is another skin blistering disease with associated epidermal necrosis with a history of antibiotic use. It is also associated with a positive nikolsky sign like pemphigus vulgaris and TEN. Stevens-Johnson syndrome and TEN are distinguished based on the body surface area involvement, typically 10-30% in SJS and greater than 30% in TEN. However, onset is typically acute with a clear inciting event like new drug or infection, making SJS a less likely diagnosis. Answer E: Toxic epidermal necrolysis (TEN) is also characterized by blisters resulting in epidermal necrosis involving greater than 30% of the body surface area. Nikolsky sign is present as epidermis easily separates upon gentle pressure due to skin necrosis. Similar sites are involved including mucosa and trunk area with painful and pruritic blisters. However, TEN usually presents as an acute onset disease with a clear inciting event like a drug reaction. Although, this patient's history is positive for an antibiotic course suggestive of a possible drug reaction, slow month long course is not consistent with TEN. TEN is also associated with a high morbidity and mortality due to high risk of secondary skin infections and these patients are typically monitored closely in a burn unit. Bottom Line: Pemphigus vulgaris is an intraepidermal autoimmune blistering disease with a positive Nikolsky's sign.

A 58-year-old female presents with sudden-onset shortness of breath and right-sided chest pain. Vitals are obtained and reveal the following: Temperature 37.6°C (99.6°F) Heart rate 106/min Blood pressure 99/68 mmHg Respiratory rate 26/min Oxygen saturation 88% on room air Physical examination reveals jugular venous distention to 8 cm and an accentuated pulmonic component of S2. A plain film chest radiograph is obtained and is read as unremarkable. An ECG is obtained and reveals sinus tachycardia with a right-axis deviation, S wave changes in lead I, Q wave changes in lead III, and T wave inversion in lead III. Which of the following is the most accurate modality to confirm the most likely diagnosis? A. arterial blood gas analysis B. computed tomography scan of the chest with contrast C. coronary angiography with cardiac catheterization D. radionuclide contrast imaging with pharmacologic stress E. ventilation/perfusion scan

The correct answer is: B This patient is suffering from a pulmonary thromboembolism (PE). Pulmonary embolism occurs when a thrombus originating from the venous circulation (commonly from pelvis or lower extremities) or right side of the heart embolizes to the pulmonary vessels via the right ventricle and pulmonary artery. The embolus blocks distal blood flow leading to increased pulmonary vascular resistance, pulmonary hypertension, and possible right ventricle failure. PE usually presents with abrupt onset of tachypnea, pleuritic chest pain, and dyspnea. Physical examination may reveal a loud pulmonary component to S2 and a prominent jugular A wave due to right heart failure. ECG in acute PE commonly only shows sinus tachycardia with nonspecific ST-T wave abnormalities. Electrocardiographic abnormalities historically associated with PE (S1Q3T3 pattern, right ventricular strain, new incomplete right bundle branch block) are rarely present. If present, they are suggestive of acute cor pulmonale due to massive PE. However, this ECG pattern can also occur with other causes of acute cor pulmonale (e.g., tension pneumothorax, bronchospasm). Therefore, an S1Q3T3 pattern alone is not sufficient for diagnosis of PE. Pulmonary angiography is the definitive diagnostic technique or "gold standard" in the diagnosis of acute PE and can detect emboli as small as 1-2 mm. However, it is an invasive procedure and should only be used if noninvasive testing is equivocal and risk of anticoagulation is high or if the patient is hemodynamically unstable and embolectomy may be necessary. Answer A: Arterial blood gases are neither sensitive nor specific in diagnosing PE. ABG analysis commonly reveals a respiratory alkalosis due to hyperventilation with a low PaO2 and PaCO2. The A-a gradient is usually elevated. However, it is not uncommon for a normal partial oxygen pressure, partial carbon dioxide pressure and A-a gradient to be present during a PE. Answer C: This would be indicated for someone with a suspected myocardial infarction. For the left side of the heart, the coronary vessels are accessed through the brachial, femoral, radial, or axillary artery. For the right side of the heart, the coronary vessels are accessed through the femoral, internal jugular, or subclavian veins. Blockages are directly observed through injection of radiocontrast via real-time radiography and stents are placed in the appropriate ischemic locations. Answer D: A cardiac stress test would be indicated for someone with a myocardial infarction. Adenosine (A2 receptor agonist leading to increased cAMP), dipyridamole (increases intravascular adenosine), and dobutamine (inotrope and chronotrope) are commonly used to 'stress' the heart by dilating the coronary vessels. More stable patients are stressed by walking and running on a treadmill. The radionucleotides measure cardiac muscle uptake before and after the stress and is able to note anatomic areas of the heart that are ischemic. An EKG is also recorded throughout this diagnostic procedure. Answer E: Ventilation/perfusion (V/Q) scan uses scintigraphy and isotopes to evaluate the circulation of air and blood within a patient's lungs. It has been replaced by helical CT as the initial diagnostic modality in evaluation of PE. However, it is still commonly used for patients with contraindications to helical CT, specifically the pregnant population. Bottom Line: The classic ECG findings associated with PE are tachycardia, right-axis deviation, and a S1Q3T3 pattern. These findings are transient and nonspecific. However, their presence suggests massive PE blocking the central pulmonary arterial circulation leading to cor pulmonale. Pulmonary angiography is the "gold standard" for diagnosing PE. Due to its invasive nature, it is reserved for patients who are hemodynamically unstable or those with a high clinical pretest probability of PE but equivocal noninvasive test findings

A 35-year-old female presents to the emergency department with the complaint of vertigo, nausea, and vomiting lasting for the past 24 hours' duration. History reveals constant ringing in her ears that she attributes to a recent flu-like illness. Physical examination reveals the patient has mild hearing loss and a positive Dix-Hallpike maneuver. The most likely diagnosis is A. benign paroxysmal positional vertigo B. labyrinthitis C. Meniere's disease D.vascular insufficiency E. vestibular neuritis

The correct answer is: B This patient most likely has labyrinthitis. Labyrinthitis refers to inflammation of the labyrinth of the inner ear. Though the cause is not clear, it is most often seen after a viral upper respiratory tract infection. The inflammation may cause sudden vertigo with temporary hearing loss or tinnitus. Attacks usually last several hours to days. Both labyrinthitis and BPPV may test positive for Dix-Hallpike maneuver. To perform the Dix-Hallpike maneuver, the patient initially sits upright. The examiner turns the patient's head 30-45 degrees to the side tested. Then, as the examiner supports the patient's head, the patient quickly lies supine (within 2 sec), allowing the neck to hyperextend and hang off the edge of the examining table 20-30 degrees past horizontal. After a 2-20 second period, the onset of torsional upbeat or horizontal nystagmus denotes a positive test for benign paroxysmal positional vertigo. Nystagmus changes direction when the patient sits upright again. The Dix-Hallpike maneuver has a positive predictive value of 83% and a negative predictive value of 52% for the diagnosis of BPPV. Answer A: BPPV is vertigo experienced only in specific positions or position changes. Onset is abrupt and lasts several seconds. It is not associated with hearing loss. Answer C: Meniere's disease includes a triad of vertigo, tinnitus and hearing loss. Attacks last hours to days and can recur for several months. It is not associated with a recent upper respiratory infection. Answer D: Vertebrobasilar insufficiency is vascular ischemia leading to syncope lasting 8-15 minutes that may be accompanied by facial numbness, paresthesias and visual changes. Patients usually have a history of atherosclerosis and cardiovascular risk factors. Answer E: Vestibular neuritis has a similar presentation to labyrinthitis but is generally distinguished with preserved auditory function. Bottom Line: Labyrinthitis presents with sudden vertigo lasting hours to days and accompanied by hearing loss and tinnitus

A 42-year-old female presents with lower extremity swelling and pain of one weeks' duration. History reveals the patient suffers from dry cracked skin. Vitals reveal a temperature of 38.6oC (101.5oF), blood pressure of 145/86 mmHg, heart rate of 88/min, respiratory rate of 16/min, and oxygen saturation of 98%. Physical exam reveals edematous and erythematous skin of the right lower extremity just inferior to the patella that is mildly tender to palpation. The most appropriate pharmacological management is A. amoxicillin B. aztreonam C. nafcillin D. piperacillin/tazobactam E. vancomycin

The correct answer is: C Cellulitis can be caused by many organisms depending on the route of infection. In the question stem it leads us with the clue that the patient suffers from dry cracked skin. This informs us that the most likely pathogens are streptococci and/or staphylococci because they are normal skin flora. Cellulitis is an acute inflammatory skin condition with localized pain, erythema (redness), edema (swelling), and heat. This is a nonpurulent cellulitis, as noted by the physical exam due to the absence of purulent drainage, exudate, and/or no signs of abscess (if those signs were present on physical exam this would be a purulent cellulitis). Due to the rapid progression of the disease over a one week period this warrants inpatient hospitalization with intravenous antibiotics. Cultures are rarely obtained and often nonspecific. Nafcillin would be the best choice in this case as it covers both of the suspected pathogens and would be the least in creating antibiotic resistance. The patient does not have any risk factors for MRSA infection. Risk factors for MRSA consist of recent hospitalization, long hospital stay, resident of a long-term care facility, hemodialysis, recent antibiotic therapy, HIV, men who have sex with men, sharing sports equipment, diabetes, IV drug use, incarceration, military service, sharing needles, razors or other sharp objects. By obtaining the history we can safely narrow the spectrum of the antibiotics to the appropriate choice. Answer A: Amoxicillin would cover Streptococci, but not Staphylococci. The mechanism of action is inhibition of cell wall synthesis by binding to penicillin binding proteins. Answer B: Aztreonam is used for patients with penicillin allergy for gram negative rod infections. Think of using this medication for patients allergic to penicillin to cover for pseudomonas. The mechanism of action is inhibition of cell wall synthesis by binding to penicillin binding proteins. Answer D: Piperacillin/tazobactam is a drug used for pseudomonas coverage and too broad spectrum for the use of cellulitis. Piperacillin's mechanism of action is inhibition of cell wall synthesis by binding to penicillin binding proteins. Tazobactam inhibits beta-lactamases. Answer E: This patient does not have risk factors for a MRSA infection. Studies have shown that with MSSA infections, patients do better with a penicillinase-resistant medication like dicloxacillin verses vancomycin. The mechanism of action is inhibition of bacterial cell wall synthesis by blocking glycopeptide polymerization through binding tightly to D-alanyl-D-alanine portion of cell wall precursor. Bottom Line: For cellulitis due to dry cracked skin the suspected pathogens are Streptococci and/or Staphylococci. Nafcillin covers both and is considered a first line therapy. Other first line therapies for beta hemolytic streptococci and MSSA cellulitis infections are cefazolin (IV), oxacillin (IV), clindamycin (IV and PO), dicloxacillin (PO), and cephalexin (PO).

A 53-year-old male presents to the emergency department after ingesting Drain-O. An adequate history cannot be obtained because the patient is presently unconscious. Vital signs reveal a temperature of 36.8oC (98.2oF), blood pressure of 110/75 mmHg, heart rate of 85/min, respiratory rate of 16/min, and oxygen saturation of 98%. Physical examination is unremarkable. Which of the following is CONTRAINDICATED at this time? A. famotidine B. IV fluid resuscitation C. NG tube insertion D.omeprazole E. upper endoscopy

The correct answer is: C Drain-o is a corrosive agent that contains sodium hydroxide (lye). It is used to unclog many household plumbing drains and causes significant mucosal damage if ingested. After ingestion the patient should be hospitalized, kept NPO because ingesting further substances may cause more harm, and have serial chest and abdominal radiographic images performed to evaluate for perforation. If the patient is hypotensive, intravenous fluids should be started first followed by vasopressors if unresponsive to fluids. If there is a decrease in hemoglobin, blood products should be considered. Gastric acid suppression with intravenous proton pump inhibitors is often used to prevent stress ulcers of the stomach. Endoscopy ideally should be performed within 24 hours to assess the severity of damage caused by the chemical erosion. It is indicated unless the patient is unstable (respiratory distress, hypotensive and/or actively bleeding), or esophageal perforation. If a perforation is noted on radiographic images a surgical consult is indicated. Contraindications in the setting of esophageal chemical burns include the following: Nasogastric tube insertion: may induce retching and vomiting. This may worsen the injury and subsequently cause a perforation of the damaged tissue of the esophagus and/or stomach. Emetics: vomiting causes re-exposure to the esophagus and the oropharynx to the corrosive agent, causing a further decline of the injury. Neutralizing agents: weakly acidic or basic substances should not be administered because damage due to the corrosive ingestion is generally immediate. Also, the neutralization releases heat; therefore, a consequence of thermal injury may occur causing more tissue damage. Answer A: Although not first line therapy, it is not contraindicated to lower the pH of the gastric acid secretions. Proton pump inhibitor infusion is the first line therapy. This is to lower the gastric acid to prevent further erosion from the corrosive ingestion. Answer B: IV fluid resuscitation is indicated due to the patient requiring an NPO order and may need more aggressive fluids if hypotensive. Answer D: Proton pump inhibitor infusion is first line therapy to reduce gastric acid secretions to prevent stress ulcers from the corrosive ingestion. The gastric lining is very susceptible at this stage due to the corrosive agent. To protect the gastric endothelium it is necessary to reduce gastric acid secretions. Answer E: Endoscopy is indicated and should be performed as soon as possible. This should be done within 24 hours in order to fully assess the injury of the esophagus due to the corrosive agent. Bottom Line: Any treatment that may induce vomiting or retching is contraindicated when a patient ingests a corrosive substance. Treatment consists of supportive care, intravenous fluids, NPO, proton pump inhibitor infusion, and endoscopy if stable and no signs of rupture of the esophagus and/or stomach.

A 3-year-old female presents with a rash. History reveals that the patient presented one week ago with fever, cough, and purulent sputum and was prescribed ampicillin. The rash started one week later beginning in the arms and spreading to the chest, became pruritic, and was accompanied by a low-grade fever 37.8°C (100°F) for 48 hours. Physical examination reveals a flat, contiguous mobilliform rash with areas of well circumscribed elevation measuring less than 1 cm in size with blanching erythema. The most likely cause is A. erythema infectiosum B. exanthema subitum C. exanthematous drug eruption D.rubella E. scarlet fever

The correct answer is: C Exanthematous drug eruption is the most common form of cutaneous adverse drug reaction (75-80%). Rash appears 5-14 days after initiation of the medication. Rash starts distally and spreads centrally. Rash is self-limiting and resolves 1-2 days after discontinuing the medication. Answer A: Fifth Disease (erythema infectiosum) is caused by human parvovirus B19 and affects preschool to young school-aged children. This rash presents as a raised red rash that first appears on the child's cheeks ("slapped cheek"). A lace-like rash starts on the torso and arms after 1-4 days then spread to the rest of the body. Flu-like symptoms may be present 7 days prior to appearance of the rash. Answer B: Commonly known as Roseloa infantum or Sixth Disease, this rash is caused by human herpesvirus type 6 (HHV-6) and affects children ages 6-36 months. Patients first develop a fever, usually greater than 39.5°C (103.1°F), with a rash developing as the fever is resolving. The rash is rosy pink, blanchable and develops first on the neck and chest before spreading to the rest of the body. Answer D: Also known as German Measles, this rash is caused by the rubella virus and affects children, adolescents and adults with little to no prodrome. Patients can develop a low-grade fever 37.2- 37.8°C (99-100°F) with swollen and tender lymph nodes. The rash following the fever is a raised, red, pinpoint rash that starts on the face and spreads downward. The rash can itch and last up to 3 days. Answer E: Scarlet fever is cause by streptococcal bacterial infection. Patients may develop a red rash that looks like a sunburn with a rough texture like sandpaper, most often starting on the chest and stomach then spreading to the rest of the body. The rash may last between 2-7 days and is associated with red, swollen lips with a strawberry-like tongue. Skin may peel on hands and feet after the rash fades. If left untreated, patients can develop rheumatic fever. Bottom Line: Exanthemous rash are the most common cutaneous adverse drug reaction (75-80%). The rash is a maculopapular eruption that starts distally and spreads centrally 5-14 days after starting the medication. Delay in rash presentation is a result of cell-mediated type IV hypersensitivity reaction. Common medication drug reactions include penicillins, cephalosporins, sulfonamide, anticonvulsants and allopurinol. Rash resolves within 1-2 days of drug discontinuation.

A 60-year-old female presents for a routine wellness examination. Past medical history is significant for hypertension and congestive heart failure treated with aspirin, carvedilol, enalapril, and spironolactone. Vitals include a temperature of 37oC (98.6oF), a blood pressure of 100/55 mmHg, a respiratory rate of 20/min, and a heart rate of 95/min. Physical examination is significant for bibasilar crackles and bilateral peripheral lower extremity edema. The most likely additional finding upon laboratory analysis is A. hypercalcemia B. hyperchloremia C. hyperkalemia D. hypernatremia E. hyperuricemia

The correct answer is: C Medical management of congestive heart failure with ejection fraction < 40% includes a beta blocker, ace inhibitor (ACE Inhibitor) or angiotensin receptor blocker (ARB) and spironolactone. Spironolactone is a potassium sparing diuretic with the most important side effect of life-threatening hyperkalemia. Hyperkalemia is more prone to occur in patients on other potassium sparing drugs like ACE inhibitors or ARBs, oral potassium supplements and/or underlying renal insufficiency. This patient's presentation of bibasilar crackles and bilateral lower extremity edema is suggestive of heart failure exacerbation and renal insufficiency therefore, putting this patient at risk for hyperkalemia. Therefore, in this patient who is taking both enalapril and spironolactone, hyperkalemia is the most likely finding of concern on laboratory analysis. Below is a list of common side effects of some of the drugs often tested on boards: Drugs (Class) Common side effects Hydrochlorothiazide (Diuretic) Hypercalcemia, hyperuricemia, hyperglycemia, hypokalemia, hyponatremia, hypomagnesemia Carvedilol (Beta-blocker) Bradycardia, heart block, hyperglycemia, weight gain, impotence Enalapril (ACE Inhibitor) Hyperkalemia, renal insufficiency, cough, angioedema Spironolactone (Selective Aldosterone Blocker) Hyperkalemia, gynecomastia, renal insufficiency Aspirin (Anti-platelet agent) Tinnitus, bleeding, interstitial nephritis Answers A & E: Although hypercalcemia and hyperuricemia can be seen with hydrochlorothiazide, it is not the most likely finding on laboratory analysis for this patient with signs of heart failure exacerbation on potassium sparing drugs. Answer B: Hyperchloremia is seen in metabolic acidosis and especially in the setting of renal tubular acidosis (RTA). With diuretics, it can be caused by carbonic anhydrase inhibitors. Typically hyperchloremia is associated with fluid loss or inadequate intake (as a compensatory mechanism) secondary to dehydration, diarrhea, or renal failure. Excessive sodium chloride fluid replacement is also a common cause. Answer D: Typically hyponatremia is observed in patients with heart failure exacerbation due to hypervolemia, instead of hypernatremia. Also, hyponatremia can be seen in patients on hydrochlorothiazide, therefore making hypernatremia a wrong answer choice. Bottom Line: Hyperkalemia is commonly noted in patients on ACE Inhibitors and aldosterone antagonists especially in the setting of renal insufficiency.

A 52-year-old male presents with symptoms of being lightheaded upon standing. History reveals the patient was sitting down and then felt lightheaded and subsequently passed out after standing up. Past medical history reveals benign prostatic hypertrophy and hypertension for which he was recently prescribed prazosin. Vitals reveal the following: Supine Heart rate an >72/min Blood pressure an >140/92 mmHg Standing Heart rate an >94/min Blood pressure an >126/84 mmHg The most likely diagnosis is A. antihypertensive overdose B.Brugada syndrome C. orthostatic intolerance D. postural tachycardia syndrome E. vasovagal event

The correct answer is: C Orthostatic hypotension may be iatrogenic (drug induced), volume depletion (dehydration and blood loss), post prandial (after meals), primary autonomic (Lewy body diseases such as Parkinson's), and secondary autonomic (diabetes, amyloidosis, and Sjogren's). Patients may present with symptoms of lightheadedness or dizziness, weakness, difficulty concentrating, blurred vision, tremulousness, pallor, clammy feelings, and nausea. The history is classic with changes in laying down or sitting position to a standing position and have these symptoms and then pass out - likely orthostatic intolerance. The criteria for diagnosing orthostatic hypotension with vital signs includes one or both of the following (within 2-5 min of quiet standing after 5 min of supine rest): Systolic: 20 mmHg Diastolic: 10 mmHg Although the patient does not fit this criteria, it is pointed out that the patient is on an alpha-blocker for the hypertension and benign prostatic hypertrophy (BPH). Prazosin is known to cause first dose syncope due to orthostatic hypotension. This is due to the fact that the body's natural response to changing positions from sitting or laying to a standing position is to increase peripheral vascular resistance. This natural physiologic response is now inhibited by the medication which causes a decrease in cerebral blood flow and subsequently syncope due to orthostatic hypotension. Answer A: Prazosin is an alpha blocker that antagonizes the peripheral alpha-1 adrenergic receptors. It is indicated for hypertension and benign prostatic hypertrophy and is known to cause orthostatic hypotension as a side effect, especially after taking the first dose. It is important to note that plasma prazosin levels do not correlate with the risk of syncope. Answer B: Brugada syndrome is a disorder characterized by sudden death associated with one of several ECG patterns characterized by incomplete right bundle-branch block and ST-segment elevations in the anterior precordial leads. Answer D: The postural tachycardia syndrome is a form of orthostatic intolerance with an increase in heart rate (>30/min) that occurs upon standing without arterial hypotension. Manifestations include lightheadedness, vision changes, and fatigue upon standing. Answer E: Vasovagal syncope (common faint) presents with provoked intense emotion, pain, anxiety, sight of blood, and unpleasant odors. The vasovagal response is due to a trigger with an improper reaction of the autonomic nervous system with relation to heart rate and blood pressure. The patient has a decrease in heart rate and the blood pressure drops, which causes decreased cerebral blood flow that causes syncope. Bottom Line: Orthostatic hypotension can be from several different causes. Commonly tested topics are with medication adverse reactions. Alpha-blockers treat both hypertension and BPH with a known side effect of orthostatic hypotension.

A 30-year-old female presents with complaints of a salmon colored spot on her back with a red outline. During physical examination this new skin lesion is identified as a herald patch. The most likely diagnosis is A. pityriasis alba B. pityriasis lichenoides C. pityriasis rosea D. pityriasis rubra pilaris E. tinea versicolor

The correct answer is: C Pityriasis rosea typically starts out with a single skin lesion (herald patch) that is usually on the trunk. This skin lesion is typically salmon colored in the center with a dark red outline that is the shape of an oval. The herald patch can then enlarge and become several centimeters in diameter. After the initial lesion develops, several other lesions develop diffusely over a period of a few weeks. These lesions are oval in shape that are macules or papules with a scaling texture and wrinkling. Treatment is supportive. Water, soap, and sweat may cause irritation. If the patient suffers from pruritus topical zinc oxide or calamine lotion may be applied. For severe cases topical or oral steroids may be used. Answer A: Pityriasis alba presents as multiple erythematous patches usually in the face, not one lesion and then multiple after weeks. These erythematous patches evolve into hypopigmented areas that eventually resolve after time. Answer B: Pityriasis lichenoides starts out as multiple lesions and not just one. It may be over the trunk, buttocks, or extremities. These lesions are described as papules that become vesicles and may even develop into hemorrhagic crusts. Answer D: Pityriasis rubra pilaris typically presents as a descending rash that is described as redness and scaliness. This is found on the face and descends to where they may even have thickening of the skin on the soles of their hands and feet. Answer E: Tinea versicolor is noted as hypopigmented lesions on darker skinned patients and hyperpigmented lesions on lighter skinned patients. It is found to be secondary to a fungal infection that is superficial. It is a relatively common benign skin disorder. That is located on the chest and the back. Bottom Line: A herald patch that is described as a single oval lesion with a salmon colored center with a red border is the first clue of pityriasis rosea. If you see or hear herald patch think pityriasis rosea.

A 31-year-old HIV positive male, non-compliant with HAART, presents with acute onset of diffuse abdominal pain. History reveals the pain began suddenly about 4 hours ago. He also notes that he had been experiencing bloody diarrhea for the past several days. He denies any nausea or vomiting. Physical examination reveals a diffusely tender abdomen with guarding and rebound tenderness. A standing plain film chest and abdominal series shows free air under the diaphragm. This is most likely caused by A. Mycobacterium avium-intracellulare B. cryptosporidium C. cytomegalovirus D.herpes simplex virus E. microsporidium

The correct answer is: C Pneumoperitoneum refers to air under the diaphragm secondary to intestinal perforation. Given this patient's history of HIV and non-compliance with HAART, the differential diagnosis of spontaneous peritonitis must include a host of opportunistic infections. These infections may be caused by cytomegalovirus (CMV), Mycobacterium avium-intracellulare, microsporidium, and Cryptosporidium among others. Complicated or untreated CMV colitis can result in mucosal perforation as present in this patient making CMV the most likely answer. CMV colitis diagnosis is confirmed by endoscopy showing punctuate and superficial erosions to deep ulcerations in the colon. Biopsy of these ulcers reveals characteristic cytomegalic cells which are large cells containing eosinophilic intranuclear intracytoplasmic inclusions. Therefore, prompt treatment should be initiated to avoid complications of toxic megacolon, necrotizing colitis and submucosal vasculitis resulting in perforation as present in this patient. IV ganciclovir or foscarnet are typical first line agents and are typically continued for 3-6 weeks for treatment of CMV colitis. This can cause thrombosis of the vessels, ischemia, ulceration, gangrene, and perforation of the involved bowel. Answer A: Mycobacterium avium complex (MAC) infection in HIV patients results in non-specific findings of fever, night sweats, abdominal pain, diarrhea, and weight loss. Lab abnormalities are typically significant for anemia with elevated LDH and alkaline phosphatase and typical CD4 cell counts <100 cells/microL. However, it is not typically associated with necrotizing colitis resulting in perforation as present in this patient. Answer B: Cryptosporidium is an intracellular protozoan parasite that presents as gastrointestinal infections in immunocompromised patients including HIV patients with CD4 cell counts <100 cells/microL. It typically results in upper or mid-abdominal cramps with chronic watery diarrhea. Bloody diarrhea is very rare and usually seen if there is a co-infection with another pathogen, thus making it an incorrect answer choice. Answer D: Although in rare cases, herpes simplex virus can result in lower abdominal cramps, tenesmus and bloody diarrhea, it is not typically associated with diarrhea in HIV patients making it a less likely answer choice. Answer E: Microsporidia are intracellular spore-forming organisms that typically present as opportunistic infections in HIV patients with CD4 <100 cells/microL. It presents with water, non-bloody diarrhea with diffuse abdominal pain as well. However, it is not associated with vasculitis resulting in intestinal perforation like CMV. Bottom Line: CMV is a potential infectious etiology found in HIV positive patients with bowel perforation.

A 62-year-old male with a history of alcoholism presents with abdominal muscle cramps and difficulty breathing. Physical examination reveals ipsilateral contraction of the facial muscles when the physician taps 2 cm anterior to the external auditory meatus. Laboratory tests reveal a calcium level of 6.7 mg/dL and albumin level of 3.7 mg/dL. Replacement therapy is initiated with intravenous calcium gluconate. Repeat calcium level is unchanged at 6.7 mg/dL. The most likely additional finding upon obtaining an electrocardiogram is A. Osborne J wave B. prolonged PR interval C. prolonged QT interval D. prominent U wave E.widening of the QRS complex

The correct answer is: C The hallmark ECG finding associated with hypocalcemia is prolongation of the QT interval due to lengthening of the ST segment. The length of the ST segment is directly proportional to the degree of hypocalcemia or inversely proportional to the serum calcium level. Prolongation of the QT interval can result in deadly ventricular dysrhythmias. Hypocalcemia also causes decreased myocardial contractility, which can lead to hypotension and heart failure in severe cases. Answer A: Those with hypothermia (< 93ºF) may display J wave or Osborne wave in leads V2-V5 due to distortion of the early phase of membrane repolarization. This is also seen in the setting of hypercalcemia, subarachnoid hemorrhage, ventricular fibrillation, and cardiopulmonary arrest from over-sedation. Answer B: Prolongation of the PR interval is commonly seen in hyperkalemia. The PR interval does not exhibit characteristic changes in hypocalcemia. Answer D: A U wave is a small wave on the EKG that, if present, commonly follows the T wave. It represents repolarization of the papillary muscles or Purkinje fibers. A prominent U wave is most commonly associated with hypokalemia. Other less common causes include hypercalcemia, thyrotoxicosis, intracranial hemorrhage, and digitalis use. A prominent U wave is not seen with hypocalcemia. Answer E: Widening of the QRS complex is commonly seen in hyperkalemia. The QRS complex usually is not affected by hypocalcemia. Bottom Line: Hypocalcemia causes prolongation of the QT interval, which may lead to ventricular dysrhythmias. ECG findings commonly resolve after calcium levels return to normal.

A 25-year-old male presents with nausea, vomiting, polydipsia, polyuria, and weakness. History reveals he had an upper respiratory infection a few days ago. Physical examination reveals dry mucous membranes as well as labored and deep respirations and generalized tenderness of the abdomen. Vitals reveal a heart rate of 122/min and respiratory rate of 32/min. Urinalysis reveals 2+ ketones. An ABG reveals a decreased PCO2. Laboratory studies reveal the following: Na+ 139 mEq/L K+ 4.5 mEq/L Cl- 103 mmol/L HCO3- 16 mEq/L BUN 10 mg/dL Cr 0.9 mg/dL Glucose 470 mg/dL The most likely acid-base disorder is A. high anion gap metabolic acidosis B. high anion gap metabolic acidosis with respiratory acidosis C. high anion gap metabolic acidosis with respiratory compensation D. normal ann gap metabolic acidosis with respiratory compensation E. normal anion gap metabolic acidosis without respiratory compensation

The correct answer is: C This patient has a typical presentation of diabetic ketoacidosis (DKA). DKA is characterized by an elevated anion gap metabolic acidosis. Anion gap is calculated by the formula [Na+] − ([Cl-] + [HCO3−]). An anion gap greater than 12 is considered elevated. The arterial pH in DKA is less than 7.30 and can be lower than 6.90. This occurs in type I diabetics who do not take their insulin as directed or has some other inciting event such as infection, high stress, or myocardial infarction. The lack of insulin in diabetics leads to an increase in the break down of triglycerides and subsequent conversion to beta-hydroxybutyrate and acetoacetate. These ketoacids cause the serum bicarbonate concentration to be reduced giving the arterial pH as described above. Compensatory hyperventilation results in a fall in the partial pressure of CO2 that minimizes the fall in arterial pH. The final result is a high anion gap metabolic acidosis with respiratory compensation. This would be evident on an arterial blood gas by a partial pressure of CO2 that is low, a partial pressure of bicarbonate that is low, and a pH that is variable depending on the degree of acidosis and compensation. Apart from the acidosis the patient also has high glucose levels resulting in polyuria and dehydration which usually is the most noticeable aspect on presentation. Treatment is initiated by replacing fluids intravenously, starting on IV insulin, and correcting electrolyte abnormalities. 1. H&P. The most clinical useful information comes from the clinical description of the patient by the history and physical examination. The H&P usually gives an idea of what acid base disorder might be present even before collecting the ABG sample 2. Look at the pH. Is there an acid base disorder present? - If pH < 7.35, then acidemia - if pH > 7.45, then alkalemia - If pH within normal range, then acid base disorder not likely present. - pH may be normal in the presence of a mixed acid base disorder, particularly if other parameters of the ABG are abnormal. 3. Look at PCO2, HCO3-. What is the acid base process (alkalosis vs acidosis) leading to the abnormal pH? Are both values normal or abnormal? - In simple acid base disorders, both values are abnormal and direction of the abnormal change is the same for both parameters. - One abnormal value will be the initial change and the other will be the compensatory response. 3a. Distinguish the initial change from the compensatory response. - The initial change will be the abnormal value that correlates with the abnormal pH. - If Alkalosis, then PCO2 low or HCO3- high - If Acidosis, then PCO2 high or HCO3- low. Once the initial change is identified, then the other abnormal parameter is the compensatory response if the direction of the change is the same. If not, suspect a mixed disorder. 3b. Once the initial chemical change and the compensatory response is distinguished, then identify the specific disorder. - If PCO2 is the initial chemical change, then process is respiratory. - if HCO3- is the initial chemical change, then process is metabolic. Answer A: Metabolic acidosis with an elevated anion gap can have many causes. The mnemonic MUDPILES is often used to remember these causes: Methanol, Uremia, Diabetic ketoacidosis, Paraldehyde, Isoniazid, Lactic acidosis, Ethylene glycol, and Salicylates. History and physical can give you an idea as to which of these causes need to be considered and many of them can be confirmed with laboratory tests. This patient has a typical presentation of diabetic ketoacidosis which will usually have some respiratory compensation which is confirmed by the ABG results (low PCO2). Answer B: Patients with diabetic ketoacidosis will have an elevated anion gap metabolic acidosis. They will hyperventilate as a results of this acidosis to decrease CO2 causing a respiratory alkalosis. Common causes of respiratory acidosis are respiratory depression and COPD. Answers D & E: Common causes of normal gap metabolic acidosis include diarrhea, renal tubular acidosis, and total parenteral nutrition. This patient presents with signs and symptoms of diabetic ketoacidosis.

A 56-year-old male presents with palpitations and chest pain. History reveals the chest pain radiates toward the back and the left arm with associated painful respiration. Vital signs reveal a blood pressure of 138/84 mmHg, heart rate of 86/min, respiratory rate of 16/min, temperature of 36.5oC (97.7oF), and oxygen saturation of 98% on room air. Physical examination reveals a normal rate and regular rhythm without murmurs, rubs, or gallops. The lungs are clear to auscultation bilaterally, and the abdomen has normal bowel sounds and is soft, non-tender, and non-distended. An electrocardiogram is obtained and reveals ST segment elevation in leads I, II, III, and V2-V6. The most appropriate management is A. ceftriaxone B. heparin C. indomethacin D.levofloxacin E. percutaneous coronary intervention

The correct answer is: C This patient has acute pericarditis. Pericarditis can have many various causes. The most common is a viral infection although the exact etiology is typically unknown: Coxsackievirus, echovirus, adenovirus, EBV, CMV, influenza, varicella, rubella, HIV, hepatitis B, mumps, parvovirus B19. Radiation, cancer, trauma, and autoimmune disorders are also common causes. The key to this case is the diffuse ST segment elevation. If this was a ST segment elevation myocardial infarction (STEMI) the EKG would have 2-3 consecutive leads of ST segment elevation, but not diffuse ST segment elevation. The patient presentation of pericarditis is very similar to acute coronary syndrome. Surprisingly this patient does not have a pericardial friction rub. The treatment for acute pericarditis is with non-steroidal anti-inflammatory drugs. The patient may be treated with aspirin 2-4 g/day, ibuprofen 400-600 mg TID, or indomethacin 25-50 mg TID. Colchicine may also be given or even glucocorticoids if treatment has been refractory. Answer A: Ceftriaxone is a third generation cephalosporin that has coverage for both gram positive and gram negative bacteria. It is used as a first line treatment for community acquired pneumonia in conjunction with a macrolide (azithromycin). Answer B: Heparin is an anticoagulant that is an antithrombin III inhibitor. It is used for treatment in acute coronary syndrome, deep venous thrombosis (DVT), pulmonary embolism (PE), and prophylaxis treatment to prevent a DVT and/or PE. Answer D: Levofloxacin is a respiratory fluoroquinolone. It has coverage for both gram negative and gram positive bacteria. It is used as a first line treatment in pneumonia. It may also be used in urinary tract infections and abdominal infections. Answer E: Percutaneous coronary intervention is used in treatment of acute coronary syndrome. In acute ST segment elevation myocardial infarction (STEMI) the door to balloon time is 90 minutes for percutaneous coronary intervention. For non-ST segment elevation myocardial infarction (NSTEMI) percutaneous coronary intervention is also used, but not necessary to be performed within 90 minutes. This process may involve angioplasty and or stent placement to allow better coronary blood flow. Bottom Line: Diffuse ST segment elevation on EKG is a key sign of acute pericarditis.

A controversial government official is brought into the emergency department with perioral burning, abdominal pain, and bloody diarrhea. He says his symptoms began after ingesting several unusual brown, speckled beans that were found on his salad two days ago. An image of the beans is shown in the exhibit. His temperature is 38.2°C (100.8°F), blood pressure is 90/55 mmHg, and heart rate is 93/min. Physical examination reveals dry mucus membranes, scleral icterus, and diffuse abdominal tenderness. Laboratory studies show: Aspartate transaminase: 102 U/L Alanine transaminase: 97 U/L Alkaline phosphatase: 300 U/L Blood urea nitrogen: 43 mg/dL Creatinine: 4.7 mg/dL The most likely etiologic agent is A. anthrax B. ebola C. ricin D. smallpox E. tularemia

The correct answer is: C This patient ingested ricin, a poison derived from the plant Ricinus communis, also referred to as the castor oil plant. Castor oil plants grow in tropical regions worldwide, but can also be found in the southwestern region of the United States (southern California). The beans or seeds of the plant contain the most concentrated amount of ricin and are most commonly used to make castor oil, a brake and hydraulic fluid constituent. Ricin acts as a poison in animals (including humans) by inhibiting protein synthesis causing cell death. The Centers for Disease Control and Prevention (CDC) classifies it as a category B agent of bioterrorism because it is moderately easy to disseminate (can be ingested, inhaled, or injected), results in moderate morbidity rates and low mortality rates, and requires enhanced diagnostic capacity. Clinical manifestations of ricin toxicity vary depending route of exposure. Symptoms of ricin ingestion usually present within 6 hours and may include perioral/oral burning, abdominal pain, vomiting or hematemesis, and bloody diarrhea. Inhalational ricin poisoning may cause respiratory distress, fever, cough and pulmonary edema. If medical attention is delayed, patients usually develop severe dehydration, shock, and multi-organ failure. Death occurs within 3-5 days. There is no antidote available for ricin. Treatment is supportive with intravenous fluids, airway support, and decontamination. Gastric lavage and activated charcoal may be considered depending on time of ingestion. Prognosis is usually good with prompt initiation of supportive care. Answer A: Anthrax is a category A agent of bioterrorism derived from the spore-forming bacteria bacillus anthracis. Clinical manifestations of anthrax vary depending on route of exposure but it may present with gastrointestinal, inhalational, or cutaneous symptoms. Gastrointestinal anthrax is rare and usually caused by ingestion of contaminated meat. Anthrax ingestion usually is not the result of a bioterrorism event. Answer B: Ebola virus is a hemorrhagic fever virus that is recognized as a category A agent of bioterrorism, however, there is no evidence it has ever been used in a biologic attack. Ebola virus can be contracted by infected body fluid or air particles. Clinical manifestations include fever, myalgia, disseminate intravascular coagulation, purpuric rash, epistaxis, hematemesis, and hemoptysis. Answer D: Smallpox is a category A agent of bioterrorism caused by variola major or variola minor virus. Strict vaccination against these viruses led to global eradication of smallpox in the 1970's. Smallpox can be spread by aerosolized route or direct contact with associated cutaneous lesions. Symptoms usually present 12-14 days after exposure and include high fever, malaise, vomiting, headache, backache and a maculopapular rash that begins peripherally and moves to the trunk. Answer E: Tularemia has been recognized as a category A agent of bioterrorism since the mid-twentieth century. The bacteria francisella tularensis can survive for weeks in the environment and can be spread either from inhalation or ingestion of contaminated food or water/ water droplets. Symptoms include fever, headache, fatigue, pharyngitis, lymphadenopathy, pleuritis, and/or bronchopneumonia. Bottom Line: Ricin is a natural poison derived by caster beans. It is classified as a category B agent of bioterrorism due to its ability to be easily disseminated and cause significant morbidity. Patients may initially present with gastrointestinal or pulmonary symptoms depending on route of administration. Without treatment, ricin poisoning leads to shock, multi-organ failure and death within days. There is no known antidote available for ricin, however, prompt diagnosis and early supportive care usually results in full recovery.

A 66-year-old male with chronic kidney disease and a chronic duodenal ulcer presents with generalized weakness and fatigue. Laboratory studies are obtained and reveal the following: White blood cell count 6,500/mm3 Hemoglobin 10.1 g/dL Platelet count 200,000/mm3 Mean corpuscular volume 76 fL BUN 44 mg/dL Creatinine 2.4 mg/dL The most likely cause of this patient's anemia is A. chronic myelogenous leukemia B. cobalamin deficiency C. iron deficiency D. polycythemia vera E. pyridoxine deficiency

The correct answer is: C This patient is presenting with fatigue, microcytic anemia, and a source of bleeding due to the duodenal ulcer. This is consistent with iron deficiency anemia. The most common cause of iron deficiency anemia is blood loss, either overt or occult. Overt blood loss includes traumatic hemorrhage, hematemesis, melena, hemoptysis, menorrhagia, and gross hematuria. Occult blood loss is due to microscopic bleeding in the stool caused by such things such as bleeding ulcers or gastrointestinal tract tumors like colon cancer. The usual presenting symptoms in adults with iron deficiency are weakness, headache, irritability, fatigue and exercise intolerance. Laboratory findings will show a microcytic, hypochromic anemia with low serum iron, ferritin, and transferrin saturation levels. Microcytic anemia is defined as a decrease in mean corpuscular volume (MCV) to < 80 fL. Hypochromia is defined as a decreased mean corpuscular hemoglobin concentration (MCHC) < 33 g/dL. Answer A: Chronic myelogenous leukemia is due to a chromosomal translocation of t(9;22), which is known as the Philadelphia chromosome. It results in the formation of the gene product BCR-ABL1, which is a constitutively active tyrosine kinase. Chronic myelogenous leukemia is most common in middle-aged individuals, and patients will present systemic complaints of fatigue, malaise, sweating, weight loss, bleeding, and early satiety. Physical exam will demonstrate hepatosplenomegaly. Associated laboratory include an elevated white blood cell count, thrombocytosis, and a normocytic, normochromic anemia. The gold standard for the diagnosis of CML is either the demonstration of the Philadelphia chromosome t(9;22) translocation. Treatment is with imatinib. Answer B: A deficiency in cobalamin (vitamin B12) can lead to a megaloblastic anemia. Megaloblastic anemia is macrocytic, and thus will show an increase in the mean corpuscular volume (MCV) greater than 100 fL. Vitamin B12 is required for DNA synthesis, and a deficiency leads to cell cycle inhibition in the G2 growth phase. This leads to continued cell growth without division, and results in the characteristic megaloblasts. Common causes of vitamin B12 deficiency include vegan diet, pernicious anemia, and Diphyllobothrium latum infection. Vitamin B12 deficiency results in subacute combined degeneration of the dorsal and lateral spinal columns due to a defect in myelin formation. Peripheral neuropathy can result, which is symmetrical in nature and affects the legs more than the arms. It manifests with paresthesias, ataxia, and loss of vibration and position sense. Answer D: Polycythemia vera is a stem cell disorder in which there is uncontrolled production of all lines of myeloid progenitor cells. It results in leukocytosis, thrombocytosis, and increased absolute red blood cell mass. Patients can present with generalized pruritus, thrombotic events, erythromelalgia, and acral dysesthesias. Pruritus occurs following a warm bath or shower due to the increase in number of mast cells and resultant degranulation and release of histamine. Erythromelalgia presents as a burning pain in the feet or hands accompanied by erythema, pallor, or cyanosis. It is due to the microvascular thrombotic events that occur. Venous and arterial thrombosis is common due to the increase in blood viscosity from the elevated red blood cell mass, platelets, and leukocytes. Laboratory findings in polycythemia vera include an elevated hematocrit and red blood cell mass, thrombocytosis, and leukocytosis. Patients with polycythemia vera have low serum erythropoietin concentrations due to a negative feedback effect. Polycythemia vera is most commonly caused by a mutation in the tyrosine kinase JAK 2. Answer E: Vitamin B6 (pyridoxine) deficiency can result in a sideroblastic anemia. Sideroblasts are nucleated erythroblasts with one or more iron-containing granules in the cytoplasm. The granules may completely surround the nucleus and form the diagnostic feature ringed sideroblasts. Sideroblastic anemia is a microcytic, hypochromic anemia. The hypochromic red cells have coarse basophilic granules that stain positive for iron with a Prussian blue stain. Laboratory results can also reveal an increased red cell volume distribution width, increased serum iron, reduced total iron binding capacity, increased serum transferrin saturation, and increased serum ferritin levels. Other causes of sideroblastic anemia include alcohol, isoniazid toxicity, lead toxicity, iron overload, copper deficiency, and zinc toxicity. Bottom Line: Iron deficiency anemia is a microcytic, hypochromic anemia. Thus, the MCV will be < 80 fL and the MCHC will be < 33g/dL. A macrocytic anemia will present with an MCV > 100fL.

A 65-year-old female presents to the emergency department with fatigue. The patient is unable to provide any details about her past medical history due to altered mental status. Physical examination is significant for 2+ peripheral edema over bilateral lower extremities with normal cardiovascular and lung examination. Initial laboratory studies reveal the following: Sodium 135 mEq/L Potassium 6.8 mEq/L Chloride 100 mEq/L Bicarbonate 14 mEq/L BUN 70 mg/dL Creatinine 6.5 mg/dL Her glomerular filtration rate is determined to be approximately 9 mL/min/1.73m2. The most appropriate management to lower the potassium is A. banana bag B. calcium gluconate C. hemodialysis D. insulin and dextrose E.sodium bicarbonate

The correct answer is: C This patient with fatigue along with severe hyperkalemia in setting of elevated BUN/Creatinine corresponding to a GFR less than 15 ml/min/1.73 m2 is in acute renal failure. Given the physical exam signs of volume overload like peripheral edema, it is possible the patient had a history of chronic kidney disease and is now presenting with acute on chronic kidney injury. Severe hyperkalemia can be life threatening due to various cardiac arrhythmia's and conduction abnormalities. EKG changes include progression from peaked T waves to prolonged PR with wide QRS, loss of p waves and ultimately sine waves (bye-bye waves as patient can die). Although, there is no EKG provided in this question stem, severe hyperkalemia with K 6.8 mEq/L warrants urgent treatment to prevent cardiac conduction abnormalities and death. Medical therapy will only result in transient shift of potassium into cells; however, given signs of uremia and severe hyperkalemia (K > 6.5 mEq/L) with a low GFR, urgent hemodialysis is the most appropriate therapy for this patient. Other answer choices (sodium bicarbonate, insulin/dextrose) can be performed as the next step while setting up for dialysis to help with transient intracellular shift; however, it is not the most appropriate therapy for this patient with uremia and severe hyperkalemia. Other Indications for urgent hemodialysis besides severe hyperkalemia include acidosis, toxins, volume overload, uremia, and pericarditis. A good mnemonic used in first aid is "C-BIG-K-DI" ( If you see a big K, the patient could die!). Calcium Gluconate Bicarbonate or Beta-2 agonists (nebulized albuterol): transient intracellular potassium shift IV insulin + Dextrose (glucose): insulin shifts potassium intracellularly while glucose prevents hypoglycemia from insulin Kayexalate: binds potassium in the gut and excreted via feces Dialysis or Diuretics (furosemide): eliminates potassium from the body Answer A: A banana bag consists of sodium chloride with thiamine, folic acid and magnesium to prevent Wernicke-korsakoff syndrome in chronic alcoholics. However, it is not indicated for patients with hyperkalemia and acute renal failure. Answer B: IV Calcium gluconate or chloride therapy is only indicated in patients with EKG changes (widening of the QRS or loss of p waves) for stabilization of the cardiac membrane. There is no mention of EKG changes in this patient; therefore, calcium therapy is not the correct answer choice for this patient. Answer D: A bolus of 10 units of IV regular insulin with 50 ml of a 50 percent glucose solution helps with transient shift of potassium into the cells, but does not eliminate excess potassium from the body. Although it can be administered while awaiting set up for hemodialysis, it is not the most appropriate management in the setting of severe hyperkalemia with renal failure. Answer E: Sodium bicarbonate can be administered as 150 meq in one liter of 5 percent dextrose in water at 250 mL/hour which helps with transient shift of potassium into the cells. However, dialysis is the most appropriate management for this patient.

A 50-year-old male presents with new skin lesions that started earlier this week. History reveals that he recently started taking warfarin for atrial fibrillation. Past medical history is pertinent for two deep vein thromboses. Physical examination reveals large areas of necrotic skin across his chest, bilateral buttocks, and thighs. You also note paravertebral hypertonicity and fullness from T1-4 with decrease range of motion. A history of which of the following conditions would be most consistent with his current presentation? A. factor IX deficiency B. factor VIII deficiency C. protein C deficiency D.vitamin C deficiency E. von Willebrand disease

The correct answer is: C Warfarin therapy is one of the most commonly used regimens for anticoagulation due to its ability to be monitored and readily reversed. It interferes with the gamma carboxylation of the vitamin K dependent clotting factors such as II, VII, IX, X, protein C, and protein S. Other than bleeding, one of the known major complications of warfarin therapy is full thickness skin necrosis and sloughing over fatty areas, such as the buttocks, the breast, and the thigh. This phenomenon is known to occur more frequently in individuals diagnosed with protein C deficiency. This can usually be avoided by using a heparin or low molecular weight heparin (LMWH) "bridge" until the warfarin level is therapeutic. Although you are sometimes provided with osteopathic findings that are not needed to answer the question, it is important to understand why they are present and how they can impact management. In this example, the TART changes in the thoracic spine are due to the underlying cardiac pathology (Afib) and could need treatment if symptomatic. Answers A & B: Hemophilia A is due to factor VIII deficiency. Hemophilia B is due to factor IX deficiency. These are bleeding disorders; whereas this patient is clotting. Answer D: Ascorbic acid is an essential dietary nutrient. Deficiency causes scurvy due to impaired collagen synthesis. Symptoms include bleeding gums, ecchymoses, petechiae, coiled hairs, poor wound healing, and hyperkeratosis. Answer E: Von Willebrand disease (VWD) is the most common inherited bleeding disorder. This patient is clotting, not bleeding Bottom Line: Warfarin induced skin necrosis is more frequently seen in individuals diagnosed with protein C deficiency

A 54 year old with sensory and motor impairments, mostly of the distal extremities would have a deficiency in... A. calcium B. selenium C. vitamin B1 D. vitamin B3 E. zinc

The correct answer is: C. Recognize that in developed countries the most common cause of chronic thiamine (B1) deficiency is alcoholism. Patients can present with horizontal nystagmus (abnormal eye movements), ophthalmoplegia, cerebellar ataxia (lack of co-ordination while walking) and mental impairment (dementia) along with confabulation psychosis termed Wernicke-Korsakoff syndrome. Thiamine deficiency also causes beriberi. Dry beriberi presents as symmetrical peripheral neuropathy with both motor and sensory impairment. Wet beriberi presents with (in addition to neuropathy) signs consistent with CHF including cardiomegaly, lower extremity swelling, and tachycardia.

A 34-year-old female presents to the hospital following a head on automotive collision. The patient was not intubated prior to arrival and is found to have blood oozing from her right ear. Upon further examination you find that in response to painful stimuli she opens her eyes. She is demonstrating decorticate posturing. She is only making incomprehensible sounds. What is this patient's Glasgow Coma Scale score? A. 11 B. 3 C.5 D. 7 E. 9

The correct answer is: D A focused neurologic examination should include the Glasgow Coma Scale (GCS) in all trauma patients. It provides useful information about necessity for intubation. it is an objective way to assess a patient over time and between different health care providers. The following values are used in calculating the GCS: Eye opening, Verbal Response, Motor response. Bottom Line: Glasgow Coma Scale is calculated based on eye opening, motor and verbal responses. It is a quick and objective measure of neurologic status in trauma patients.

A 25-year-old male presents comatose to the emergency department after a motor vehicle accident. A neurologic evaluation has determined the patient to be brain-dead. Which of the following is the most likely additional physical finding? A. intact corneal reflex B. intact oculocephalic reflex C. reactive pupils D. spinal reflexes to pain E.spontaneous respiration

The correct answer is: D Brain death may be diagnosed in the absence of brainstem function. The patient must be free of any pharmacologic and medical conditions that could otherwise explain their decreased brain function. Patients cannot be hypotensive, hypothermic, or have decreased oxygen saturation. Findings consistent with brain death include: non-reactive pupils, lack of corneal reflex, lack of oculocephalic (doll's eyes) reflex, loss of respiratory drive even with elevated partial pressure of carbon dioxide. However, movements originating from the spinal cord or peripheral nerves may persist. For example facial nerve twitching or fasciculations of trunk and extremities. Therefore, spinal reflexes may persist despite brain death making it the correct answer choice. Answer A: Corneal reflex can be demonstrated by touching a piece of paper or tissue to the cornea. Lack of closing of the eye lids as a mechanism to protect the cornea is consistent with absent corneal reflex as seen in brain death. Therefore, intact corneal reflex is an incorrect answer choice. Answer B: Oculocephalic reflex is also known as vestibuloocular reflex. It refers to reflex eye movement in the direction opposite to head movement to preserve the image on the center of the visual field. However, this reflex is also absent in brain death. Answer C: Brain death is characterized with absent pupillary light reflex. Pupils are fixed, midposition, and dilated about 4 to 9 mm. Therefore, reactive pupils is not consistent with brain death. Answer E: Respiratory mechanism is also controlled by the brain stem therefore spontaneous respiration is also absent in brain death. Bottom Line: Spinal reflexes may be present in a patient with brain death.

A 70-year-old male presents with functional decline secondary to poor appetite, flank pain, low urine output, paravertebral muscle spasms from T6-9 bilaterally, and a tender nodule one inch lateral and one inch superior to the umbilicus. He is diagnosed with acute renal failure and started on total parenteral nutrition. Which of the following components should be limited in order to prevent azotemia? A. carbohydrates B. fat soluble vitamins C. lipids D. protein E. trace elements

The correct answer is: D Excessive protein administration in TPN should be avoided to prevent worsening azotemia. During the breakdown of protein, nitrogen is formed. Nitrogen is unable to be cleared from the body by a damaged kidney. Patients with renal failure should have an adjusted range of protein in the solution of 1.2-1.3 g/kg/day. Protein requirements are also decreased in patients with hepatic encephalopathy (ammonia) while increased in patients with burn injury and high stress level. In patients with CKD, dietary protein restriction has been shown to decrease the rate of progression of CKD by hemodynamically mediated reductions in intraglomerular pressure. The remainder of the calories provided should be from carbohydrates (dextrose) and lipids. Trace elements and fat soluble vitamins are important to consider as some of these can be dialyzed out during hemodialysis and should be replaced to prevent deficiencies. As with any renal pathology, you may see TART changes from T6-9 representative of viscerosomatic reflex from the underlying kidney pathology. Additionally you may see Chapman points associated with the kidney pathology, the anterior point displayed above one inch lateral and superior to the umbilicus. Answer A: Carbohydrates are mainly dextrose with minimum requirement of 5 g/kg/day with minimum 400 calories/day and supplies 40-50% of the caloric intake. Dextrose is contraindicated in patients with allergies to corn or corn products; however, there is no limitation for renal failure patients. Answer B: Fat soluble vitamins consist of vitamin A, D, E and K that are also essential for various body functions. However, they do not cause worsening azotemia making it an incorrect answer choice. Answer C: Lipid-containing formulations are contraindicated in patient who have allergies to egg, soybean or fat emulsions. However, triglycerides must be monitored in patients receiving IV lipid formulations. But there is no limitation for patients with acute renal failure. Answer E: Trace elements including various vitamins and minerals that are essential in small amounts for the body are easily filtered during dialysis and must be replaced. However, they do not cause worsening azotemia.

A 62-year-old male with a history of alcoholism presents with abdominal muscle cramps and difficulty breathing. Physical examination reveals ipsilateral contraction of the facial muscles when the physician taps 2 cm anterior to the external auditory meatus. Laboratory tests reveal a calcium level of 6.7 mg/dL and albumin level of 3.7 mg/dL. Replacement therapy is initiated with intravenous calcium gluconate. Repeat calcium level is unchanged at 6.7 mg/dL. The most likely additional finding upon laboratory analysis is A. hyperalbuminemia B. hypernatremia C. hypokalemia D. hypomagnesemia E. hypophosphatemia

The correct answer is: D Hypocalcemia is defined as a serum calcium < 8.5 mg/dL. Causes include hypoparathyroidism, malnutrition, hypomagnesemia, acute pancreatitis, vitamin D deficiency, and pseudohypoparathyroidism. Alcoholism is the most common cause of hypomagnesemia. The most classical sign of severe hypomagnesemia (less than 1.0 mEq/L or 1.2 mg/dL) is hypocalcemia. Although the exact mechanism is unknown, it is hypothesized that magnesium interferes with PTH secretion and bone activity, resulting in decreased calcium absorption and reabsorption. Therefore, the serum magnesium level should always be checked in a patient with hypocalcemia of unknown etiology to determine its potential contribution to the hypocalcemia. Patients will not respond to calcium replacement therapy without proper magnesium replacement. Answer A: Most calcium ions are inactive and bound to albumin. The physiologically active form (free ionized calcium) are unbound ions that are hormonally regulated. Total calcium concentration is a measure of free, ionized calcium plus bound, non-active calcium in serum. Total calcium levels will fluctuate with albumin concentration. For each 1 g/dL reduction in albumin levels there is a 0.8 mg/dL decrease in total calcium concentration. Therefore, hypoalbuminemia (not hyperalbuminemia) will result in low total calcium. However, ionized calcium levels will remain the same and thus will not cause clinical hypocalcemia. Active/ free ionized calcium values can be estimated using this formula: total calcium - (serum albumin x 0.8). Answer B: There is no direct relationship between sodium and calcium serum levels. Hypernatremia is not a common finding in patients with hypocalcemia. Answer C: There is no direct relationship between serum calcium and potassium levels. However, hypokalemia is also a common finding in severe cases of hypomagnesemia due to decreased renal reabsorption. Answer E: Phosphate levels vary depending on the etiology of hypocalcemia. Phosphate levels will be elevated in patients with renal failure or hypoparathyroidism. Phosphate will be low in patients with vitamin D deficiency and hungry bone disease. Bottom Line: A common lab finding seen in patients with hypocalcemia is hypomagnesemia. This is because significantly low magnesium levels inhibit secretion and function of PTH, resulting in decreased calcium absorption and reabsorption.

A 72-year-old male presents with complaints of insidious onset of low back pain that radiates into his buttocks and thighs bilaterally. The pain is worse upon standing for long periods of time, and relieved with sitting and leaning forwards while walking. Physical examination reveals 4+ patellar deep tendon reflexes and 2+ Achilles deep tendon reflexes bilaterally. Based on these findings, the most likely diagnosis is A. amyotrophic lateral sclerosis B. cauda equina syndrome C. herniated lumbar intervertebral disc D. lumbar spinal stenosis E. sciatica

The correct answer is: D Lumbar spinal stenosis occurs in elderly patients and is due to narrowing of the spinal canal or intervertebral foramen which compresses the spinal cord or nerve roots. It usually affects adults older than 50-years of age. It most commonly is degenerative in nature, and referred to as "spondylosis." This can be due to hypertrophy of the ligamentum flavum or facet joints, along with vertebral body osteophytes, or thickening of the posterior longitudinal ligament. It manifests with neurogenic claudication, bilateral lower extremity radicular pain, sensory loss, or weakness affecting the legs. Extension of the lumbar spine results in narrowing of the spinal canal, and thus pain is worsened with standing and walking upright. Flexion of the spinal canal opens up the spinal canal. Thus, patients will get relief of symptoms when sitting or leaning forward while walking. Patients will describe a relief of pain when pushing shopping cart at grocery store as they are leaning forward during this activity. Long-standing compression may result in upper motor neuron (UMN) signs and symptoms. Here the patient has hyperreflexia of the patellar (L2-L4 roots) reflexes bilaterally with normal Achilles (S1 root) reflexes. This information localizes the lesion(s) to the lumbar spine and, with the help of the history, confirms the likely diagnosis of spinal stenosis. Recall that deep tendon reflexes are scored from 0-5 as follows: 0 for absent, 1 for hypoactive, 2 for normal, 3 for brisk, 4 for brisk with unsustained clonus, and 5 for sustained clonus. Answer A: Amyotrophic lateral sclerosis is a motor neuron disease that affects both upper and lower motor neurons. Patients present with a gradual onset of asymmetric weakness of the distal extremities. Physical examination will reveal the upper motor neuron findings of hyperreflexia and spasticity. The lower motor neuron findings include weakness, atrophy, and fasciculations. Answer B: This syndrome is due to compression of the lumbosacral nerves below the level of the conus medullaris. Clinical manifestations include saddle anesthesia, urinary or fecal incontinence, and sensory and motor changes to the lower extremity. Answer C: A lumbar herniated disc results in a sharp lancinating pain that radiates posteriorly down the leg in a dermatomal distribution. It is unilateral in nature, as opposed to the bilateral involvement of lumbar spinal stenosis. Most disk herniations occur at the L4-L5 or L5-S1 levels. An intervertebral disc herniation will affect the lower nerve root in the involved segments. Thus, a herniation at the L4-5 level will cause impingement upon the L5 nerve root. These nerve roots are assessed by checking for the absence of the patellar reflex (L4), great toe dorsiflexion (L5), or the Achilles reflex (S1). Pain will be reproduced on physical examination with unilateral straight leg raise. Answer E: Sciatica is caused by irritation of the sciatic nerve, which consists of the nerve roots L4-S1. It leads to an intractable radiating pain below the knee. Pain radiates down the posterior aspect of the thigh and leg. The sciatic nerve can become irritated with a tight piriformis muscle. It is commonly seen in people who sit for extended hours of the day. Pain will be reproducible upon palpation of the piriformis muscle. Bottom Line: Lumbar spinal stenosis is manifested by neurogenic claudication, bilateral radicular pain, sensory loss, and weakness affecting the legs. The symptoms are exacerbated by walking or prolonged standing, and relieved by forward flexion of the lumbar spine.

A 41-year-old male presents to the emergency department with significant pain and swelling of his left lower extremity. He had bumped his leg and developed a bruise on his left calf above his ankle 3 days ago. He thought nothing of the minor injury until this morning when he woke up with excruciating, sharp pain at the site. Swelling and erythema then started at the site and has dramatically spread to involve almost the entire calf. His past medical history is significant for type 2 diabetes mellitus and hypertension. Vitals reveal the following: Temperature 39.1°C (102.4°F) Blood pressure 164/78 mmHg Heart rate 104/min Respiratory rate 16/min Observation reveals the affected extremity shown in the exhibit. Physical examination reveals the site to be exquisitely tender to palpation with an associated decreased sensation to temperature and light touch. A random blood glucose is obtained and read as 336 mg/dL. Computed tomography reveals asymmetric deep fascial thickening. Punch biopsy and culture of the tissue reveals gram-positive diplococci. The most likely diagnosis is A. cellulitis B. compartment syndrome C. erysipelas D. necrotizing fasciitis E. pyomyositis

The correct answer is: D Necrotizing fasciitis is a rapidly progressive infection that travels along the fascial plane and invades local vasculature, leading to ischemia and necrosis of overlying subcutaneous tissue. It can be separated into three separate types based on the causative bacteria: polymicrobial (Type I, usually includes S. aureus, E.coli, and Clostridium perfringens), S. pyogenes (Type II), and clostridial myonecrosis (Type III, gas gangrene). Although necrotizing fasciitis can occur in a young, healthy adult, most cases are seen in immunosuppressed patients, specifically diabetics (make up 20-40% of cases). The infection usually begins at a site of trauma. The site may be as minor as a bug bite or more serious, such as the incision site of a recent surgery. Necrotizing fasciitis presents initially with sudden onset of intense pain at the trauma site that is usually out of proportion of clinical findings and then gradually progresses to anesthesia. Swelling and erythema then develop and quickly spread over hours to days. As the infection extends into the deep fascia it causes thrombosis in the blood vessels of the dermal papillae. This causes the skin to become friable and take on a bluish, maroon discoloration. If left untreated, the pathogen can invade the venous channels and lymphatic system, leading to septic shock and multisystem organ failure. High clinical suspicion is key to diagnosing necrotizing fasciitis. Important signs to be aware of include tissue necrosis, putrid discharge, bullae, severe pain, gas production, rapid burrowing through fascial planes, and lack of classical tissue inflammatory signs. The infection can be definitively diagnosed by tissue biopsy and culture (performed on deep tissue removed during debridement). Doppler ultrasound and/or CT can be used to determine the extent of tissue involvement and the presence or absence of gas. Gas is commonly present in Type I and Type III necrotizing fasciitis, however, gas usually is not present when the cause is S. pyogenes or MRSA. Necrotizing fasciitis is a surgical emergency that requires prompt surgical exploration and debridement of necrotic tissue. Antimicrobial therapy should be started empirically and include coverage for gram-positive, gram-negative, and anaerobic organisms. A common regimen used includes penicillin G, third-generation cephalosporin, and clindamycin. Answer A: Cellulitis is a local infection of the connective and subcutaneous tissue that also presents with pain, erythema, and swelling. It is also commonly associated with fever and chills. Common pathogens include S. pyogenes and S. aureus. Early stages of necrotizing fasciitis may be mistaken for a cellulitis. Therefore, necrotizing fasciitis must be ruled out in all patients with suspected cellulitis. Cellulitis is not associated with skin coloration and necrosis, as seen in this patient. Also, this patient's CT scan showed involvement of the deep fascial layers, which are not involved in cellulitis Answer B: Compartment syndrome is a serious condition caused by pressure buildup within the fascial planes of an extremity, which can lead to rapid vascular and neurological damage. The precipitating event is commonly a major injury to the extremity such as a fracture, crush injury, or constricting bandage on swelling tissue. Patients will present with pain out of proportion of clinical findings, paresthesias, and muscle tightness. Skin changes are not commonly present. Answer C: Erysipelas is a streptococcal infection (S. pyogenes) of the upper dermis and superficial lymphatics most commonly seen in young children and the elderly. It presents with abrupt onset facial swelling and well-defined, indurated lesions that expand along the nasolabial folds and extend to involve the entire face and upper extremities. The red lesions then progress to flaccid bullae during the second and third day. Finally, desquamation of the involved skin occurs 5-10 days later. Answer E: Pyomyositis is characterized by a pus-filled abscess that forms within the skeletal muscle. It is most commonly seen in children aged 2-5 who live in tropical regions. It is caused by S. aureus. The infection most commonly affects large muscle groups such as the quadriceps or gluteal muscles and usually remains localized. However, if left untreated, patients may present in shock due to exotoxins (e.g., toxic shock toxin 1) or enterotoxins produced by the organism. This patient's infection was caused by gram-positive diplococci (i.e. S. pyogenes) and was rapidly spreading. Bottom Line: Necrotizing fasciitis is a deep infection along a fascial plane that causes severe pain followed by rapidly progressive erythema, swelling and subcutaneous tissue necrosis. Diabetes is a major risk factor. Early surgical intervention is the key to management to prevent local and systemic spread of the infection. Treatment includes surgical debridement and broad-spectrum antibiotic therapy.

A 45-year-old male presents with diarrhea and a pruritic rash to his neck and the dorsum of his hands. History reveals that he is a farmer and has primarily been eating corn for the past couple years due to his financial situation. This patient is most likely deficient in A. vitamin B1 B. vitamin B12 C. vitamin B2 D. vitamin B3 E.vitamin B6

The correct answer is: D Pellagra, meaning "raw skin," is due to a deficiency of vitamin B3 (niacin). Pellagra is characterized by a photosensitive pigmented dermatitis in sun exposed areas, diarrhea, and dementia. Pellagra is a rare entity in the United States, but is still a common manifestation of niacin deficiency in poorer countries where the local diet consists of cereal, corn, or sorghum. Pellagra can also occur in alcoholics, anorexia nervosa, and carcinoid syndrome. Carcinoid syndrome results in an increased metabolism of tryptophan into serotonin, which leads to a deficiency of niacin. The most characteristic finding in pellagra is the presence of a symmetric hyperpigmented rash that is present in the exposed areas of skin. Other clinical findings are a red tongue, diarrhea, and vomiting. Neurologic symptoms include disorientation, delusions, dementia, and encephalopathy. Answer A: A deficiency in vitamin B1 (thiamine) can result in Wernicke-Korsakoff syndrome. Wernicke's encephalopathy is manifested by necrosis of the mammillary bodies in the periventricular region of the brain. Symptoms include progressive dementia, confusion, ataxia, and paralysis of the extraocular muscles. This causes ophthalmoplegia of the bilateral lateral rectus muscles, which results in a sixth nerve palsy. Korsakoff psychosis is a thought disorder that results in retrograde memory failure and confabulation. The most common causes of thiamine deficiency include poor diet and chronic alcoholism. Answer B: A deficiency in vitamin B12 (cobalamin) can lead to subacute combined degeneration of the dorsal and lateral spinal columns due to a defect in myelin formation. Peripheral neuropathy can result, which is symmetrical in nature and affects the legs more than the arms. It manifests with paresthesias, ataxia, and loss of vibration and position sense. The animal products of meat and dairy products provide the only dietary source of vitamin B12 for humans. Thus, a strict vegan diet can result in vitamin B12 deficiency. Vitamin B12 deficiency can also be due to inadequate absorption associated with pernicious anemia or Crohn's disease. Answer C: A deficiency of vitamin B2 (riboflavin) is characterized by cheilosis or angular stomatitis (scaling lips with fissures), and glossitis. This can be seen on physical examination as cracked and red lips, inflammation of the tongue, mouth ulcers, cracks at the corners of the mouth, and a sore throat. A deficiency of vitamin B2 may also cause dry and scaling skin, fluid accumulation in the mucous membranes of the mouth, and an iron-deficiency anemia. The eyes may also become sensitive to bright light. Additionally, patients may develop seborrheic dermatitis on the face or genitalia. Vitamin B2 deficiency may be noted in patients with anorexia nervosa or with malabsorptive syndromes such as celiac sprue. Answer E: A deficiency in vitamin B6 (pyridoxine) results in stomatitis, glossitis, cheilosis, irritability, confusion, and depression. Physical examination would reveal inflammation and fissuring of the lips, cheilosis occurring at the corners of the mouth, atrophy of the mucosa of the tongue, and seborrheic dermatitis. Pyridoxine deficiency can also lead to polyneuropathy in adults. Pyridoxal phosphate is necessary for the synthesis of heme, as it serves as a coenzyme for the enzyme ALA synthase. Thus, vitamin B6 deficiency can result in a microcytic, hypochromic sideroblastic anemia with the presence of basophilic stippling seen on a peripheral blood smear. Pyridoxine deficiency may result from pregnancy or therapy with certain medications, such as the anti-tuberculosis drug isoniazid. Another common cause of vitamin B6 deficiency is chronic alcoholism. Bottom Line: Pellagra is due to a deficiency in vitamin B3 (niacin). Manifestations include the 4 D's: dermatitis, diarrhea, dementia, and death.

A 45-year-old male presents for routine pre-employment physical examination. History reveals the patient's only complaints are itching after hot showers or hot baths and intermittent headaches. Vital signs reveal a temperature of 36.8oC (98.2oF), blood pressure of 154/89 mmHg, heart rate 72/min, respiratory rate of 16/min, and oxygen saturation of 98%. Physical examination is unremarkable. A routine basic metabolic panel is obtained and reveals: Sodium 138 mEq/L Potassium 4.2 mEq/L Chloride 104 mEq/L CO2 24 mmol/L BUN 18 mg/dL Creatinine 0.8 mg/dL Glucose 89 mg/dL A complete blood count is also obtained and reveals: Leukocyte 13,000/µL Hemoglobin 20 g/dL Hematocrit 60% Platelets 600,000/µL MCV 88.5 fL The most likely cause of these abnormal laboratory values is A. chronic myelogenous leukemia B. essential thrombocytosis C. hemoconcentration secondary to dehydration D. polycythemia vera E. primary myelofibrosis

The correct answer is: D Polycythemia vera is usually unrecognized and found incidentally. Common complaints may be itching after being in hot water and hyperviscosity symptoms (headaches, visual disturbances, vertigo, and TIAs). Patients may have elevated blood pressure secondary to red cell mass elevation. If the hemoglobin is less than 20 other diagnoses have to be considered (hypoxia from high altitude, smoking, pulmonary disease and tumors). However, with the hemoglobin at 20 with leukocytosis and thrombocytosis the diagnosis is apparent. Further testing includes an assay for JAK2 V617F. Treatment consists of phlebotomy to a goal hemoglobin of ≤14 g/dL and hematocrit ≤45% in males. In females the goals are ≤12 g/dL hemoglobin and a hematocrit of ≤42%. Answer A: The CBC on a patient with CML would have an elevated WBC with ranges of 20,000-60,000/µL, have a normocytic anemia that is mild to moderate, platelets may be low, normal, or high. Answer B: With essential thrombocytosis there would not be an elevated hemoglobin or hematocrit. The remaining CBC besides the platelets are usually within normal ranges. A hyperkalemia may be present due to laboratory artifact. This is due to the large number of platelets releasing potassium during blood clotting; therefore, the potassium is falsely elevated. Answer C: While hemoconcentration due to dehydration may cause elevated hemoglobin and hematocrit levels, there is no evidence in the question stem to suggest dehydration. Also, it would be very unlikely to cause a hemoglobin level that high due to dehydration. This would also be noted on the basic metabolic panel with elevated sodium and chloride. A pre-renal ratio may be seen with a BUN/creatinine ratio greater than 20:1 with dehydration. Answer E: Primary myelofibrosis has common symptoms of night sweats, fatigue, and weight loss. A blood smear would show teardrop cells. Usually has a mild anemia with normal to high leukocyte and platelet counts. The bone marrow is extremely hard to aspirate due to the fibrosis. Radiographic images display osteosclerosis. Bottom Line: Polycythemia vera has non-specific complaints and usually found incidentally on laboratory studies. If the patient has symptoms, think of hyperviscosity like symptoms. This is due to the increased amount of red blood cells in the circulation. Laboratory studies will have elevated levels of hemoglobin, hematocrit, leukocyte count, and platelet count.

A 30-year-old female presents with abrupt onset of nausea, vomiting, wheezing, and lethargy. Vitals are obtained and reveal the following: Heart rate 120/min Blood pressure 150/90 mmHg Respiratory rate 25/min Pulse oximetry 87% on room air Physical examination reveals an obtunded female with pinpoint pupils, rhinorrhea, and facial twitching. Auscultation of the lungs reveals diffuse expiratory wheezes. The most likely etiologic agent of terrorism is A. anthrax B. phosgene C. ricin toxin D. sarin E. sulfur mustard

The correct answer is: D Sarin is a high volatile substance which can quickly vaporize and immediately penetrate the skin. Sarin is a potent inhibitor of acetylcholinesterase which results in toxic accumulation of acetylcholine at the post synaptic cleft, causing miosis, rhinorrhea, bronchospasm, diaphoresis, nausea, vomiting and urinary incontinence. If not quickly treated the patient will rapidly deteriorate resulting in apnea, coma, convulsions or death. At non-lethal doses, permanent neurological damage may result if untreated. Answer A: Anthrax is an acute disease caused by Bacillus anthracis, a gram positive rod-shaped aerobic bacterium which can form dormant endospores that survive for decades and up to centuries. Anthrax commonly infects herbivorous mammal while grazing. Disease is spread by direct contact through broken skin or by consuming meat contaminated with spores. Inhalational infection presents with flu-like symptoms followed by pneumonia and severe respiratory collapse. Consuming infected meat results in hematemesis, severe diarrhea and gastrointestinal inflammation. Cutaneous exposure results in a boil-like lesion that forms a black eschar with a painless necrotic ulcer. Answer B: Phosgene- is a chemical weapon used during WWI and in dye manufacturing in the 19th century. Cobalt II Chloride converts into phosgene when exposed to UV light, creating a free radical which results in laryngospasm with inspiratory stridor and partial airway obstruction and irritation of the eyes, nose and throat. Answer C: Ricin toxin is a highly toxic lectin produced from the seeds of the castor oil plant. Exposure is inhaled, injected or ingested. Ricin causes severe nausea, diarrhea, seizure, tachycardia and hypotension followed by cardiovascular shock and death within 3-5 days of exposure. Answer E: Sulfur mustard is a highly reactive intermediate compound that alkylates the guanine nucleotide in DNA stands which leads to programmed cell death. It is lipophilic and strongly mutagenic and carcinogenic due to alkylating properties. Exposure to mustard gas rarely causes immediate symptoms. Within 24 hours of exposure, victims experience skin irritation followed by fluid filled blisters in the areas of contact or first and second degree burns. Eye exposure results in miosis and conjunctivitis which can result in temporary blindness. If inhaled in very high concentrations, damage to the mucous membranes causing bleeding, blistering and pulmonary edema. Bottom Line: Sarin is a potent inhibitor of acetylcholinesterase which results in toxic accumulation of acetylcholine at the post synaptic cleft, causing miosis, rhinorrhea, bronchospasm, diaphoresis, nausea, vomiting and urinary incontinence. If not quickly treated the patient will rapidly deteriorate resulting in apnea, coma, convulsions or death. At non-lethal doses, permanent neurological damage may result if untreated.

A 42-year-old homeless man presents with a non-healing wound to his right leg. The patient states the initial injury occurred six months prior. Physical examination reveals scattered ecchymosis throughout his body, along with swollen gums. The most likely deficient substance is A. biotin B. copper C. selenium D. vitamin C E.zinc

The correct answer is: D Scurvy is a clinical syndrome seen with ascorbic acid deficiency which results in impaired collagen synthesis with disordered connective tissue. Symptoms occur as early as three months after deficient intake. Collagen synthesis requires vitamin C as a cofactor to facilitate cross-linking of the alpha peptides. Failure of this step in collagen synthesis results in the symptoms seen in scurvy. In scurvy, the lack of hydroxylation of proline and lysine causes a looser triple helix formation. This includes impaired wound healing, defective tooth formation, ecchymoses, bleeding gums, petechiae, coiled hairs, hyperkeratosis, arthralgias, and impaired wound healing. Radiographic findings in scurvy include periosteal elevation and epiphyseal separation. The epiphyses and periosteum also become easily detachable because of hemorrhaging that occurs below the periosteum. Vitamin C deficiency results in cortical thinning and decreased trabeculae of bone. This produces a decrease in radiopacity on plain film radiographs, resulting in an appearance that is similar to ground glass. Generalized systemic symptoms are weakness, malaise, joint swelling, arthralgias, edema, depression, neuropathy, and vasomotor instability. Answer A: Biotin is an important cofactor for enzymes involved in carboxylation reactions, an example of which is the synthesis and oxidation of fatty acids. A deficiency of biotin can lead to depression, hallucinations, muscle pain, brittle hair, alopecia, dermatitis, and an erythematous perioral macular rash. Biotin is present in dietary food, and is also produced by intestinal bacteria. Deficiencies of biotin occur in people who consume massive amounts of raw eggs. This is because egg white contains a protein, avidin, which combines very tightly with biotin and prevents its absorption. Answer B: Copper deficiency results in Menkes kinky hair syndrome. This presents as fine fragile silvery hair, depigmentation of the skin, muscle weakness, neurological abnormalities, and hepatosplenomegaly. A deficiency in copper results in osteoporosis due to defective collagen. It is also associated with skin depigmentation due to dysfunctional melanin synthesis from the copper dependent enzyme tyrosinase. A copper deficiency also results in a microcytic, hypochromic anemia and neutropenia. Answer C: Selenium is an antioxidant that is part of the enzyme glutathione peroxidase, which is found in red and white blood cells. As such, selenium acts to prevent oxidative damage to both these cell lines. Selenium deficiency can occur in patients with severely compromised intestinal function and those receiving total parenteral nutrition. A deficiency of selenium can lead to the development of heart related problems, most commonly a dilated cardiomyopathy. Answer E: A deficiency in zinc is associated with growth retardation, hypogonadism, infertility, dysgeusia, diarrhea, poor wound healing, and impaired cellular immunity. Zinc absorption may be impaired in pancreatic disease, malnutrition, malabsorption syndromes, and those receiving chronic total parenteral nutrition. Acrodermatitis enteropathica is an autosomal recessive disease in which zinc absorption is impaired. It is characterized by the signs of zinc deficiency including diarrhea, dermatitis alopecia, poor growth, and impaired cellular immunity. Bottom Line: Scurvy is due to a deficiency in vitamin C. It can lead to symptoms associated with impaired collagen synthesis, which includes ecchymoses, bleeding gums, petechiae, coiled hairs, hyperkeratosis, arthralgias, and impaired wound healing.

A 48-year-old male presents to the emergency department complaining of fevers, chills, and rigors for the last two days. History reveals the patient had an emergent laparotomy and splenectomy secondary to a motor vehicle accident last year. The patient was subsequently lost to follow-up after the procedure. Vitals and a physical examination reveal the patient to be in respiratory distress with hypotension. The most likely etiologic organism is A. Capnocytophaga canimorsus B. Haemophilus influenza C. Neisseria meningitidis D. Streptococcus pneumoniae E. Streptococcus pyogenes

The correct answer is: D Spleen plays a very important role in fighting many infections especially encapsulated organisms. Some of the host spleen mechanisms to fight these organisms include antibody production, activation of the alternative complement pathway, and phagocytosis of unopsonized particulate matter. Therefore, after a splenectomy, these patients are at risk for Overwhelming Post-splenectomy Infection (OPSI). These patients are vulnerable mostly to encapsulated organisms, which include Streptococcus pneumoniae, Neisseria meningitidis, and Haemophilus influenza. The most common of these organisms is S. pneumoniae, causing up to 90% of overall cases. It is recommended to receive vaccines for these organisms including pneumococcal vaccine, meningococcal vaccine, and Haemophilus influenzae vaccines two weeks prior to an elective splenectomy, or two weeks post-op an emergent splenectomy. Answer A: Capnocytophaga canimorsus is a gram-negative rod that is associated with dog or cat bites. It can cause bacteremia and fatal sepsis in patients with hepatic disease or asplenia. However, it is not the most common etiology of infection in asplenic patients unless there is a history of dog bite. Answer B: H. influenzae is a gram-negative oxidase-positive rod. There are several serotypes depending on the presence or absence of a polysaccharide capsule. Risk factors for H. influenzae infection include asplenia, sickle-cell anemia, complement deficiency and other immunocompromising conditions such as malignancy and HIV. However, it is not the most common cause of sepsis in patients post-splenectomy making it an incorrect answer choice. Answer C: N. meningitidis is a gram-negative diplococci that can be subdivided into various serogroups based upon distinct polysaccharide capsules. It can cause a variety of infections ranging from fever to bacteremia and meningitis which can fatal in especially in patients with immunocompromised state or post-splenectomy. Answer E: Streptococcus pyogenes or Group A Streptococcus is an aerobic gram-positive cocci. It typically causes pharyngitis, cutaneous soft tissue infections (pyoderma, erysipelas, cellulitis), acute rheumatic fever, acute glomerulonephritis, and toxic shock syndrome (TSS). Although it can cause fatal septic infections in immunocompromised and asplenic patients, it is not most common etiology for sepsis in post-splenectomy patients.

A 22-year-old college student presents to the emergency department unresponsive. History obtained from his dormitory roommate reveals he was complains of a headache and neck pain for two days' duration. Vital signs reveal a blood pressure of 80/60 mmHg and a temperature of 38.5ºC (101.3ºF). Physical examination reveals a petechial rash on his chest and back. Laboratory analysis reveals hyponatremia and pancytopenia. Which of the following is most likely associated with his condition? A. bloody lumbar puncture B. elevated creatinine C. myelofibrosis D. thrombosis E. wide anion gap

The correct answer is: D The most likely diagnosis is Waterhouse-Friderichsen syndrome, which is characterized by massive hemorrhage of the adrenal glands and septic shock from Neisseria meningitidis bacteremia. The petechial rash is a classic clue for this bug. This syndrome, in addition to adrenal hemorrhage, can lead to disseminated intravascular coagulation (DIC) with widespread thrombosis. Answer A: Bloody lumbar puncture is associated with herpes encephalopathy and subarachnoid hemorrhage, neither of which is suspected in this case. Answer B: Elevated creatinine suggests renal failure which is possible but not as probable as DIC in patients with Waterhouse friderichsen syndrome. Answer C: Myelofibrosis is replacement of the bone marrow by fibrotic tissue. It can account for the pancytopenia, but not the other symptoms. Answer E: Adrenal failure is associated with a non-anion gap metabolic acidosis due to lack of aldosterone. Bottom Line: Waterhouse-Friderichsen syndrome is massive hemorrhage of the adrenal glands and septic shock from Neisseria meningitidis bacteremia. DIC and a petechial rash are commonly associated.

A patient with chronic renal failure has remained in critical condition in the intensive care unit after a total abdominal colectomy. The patient has diffuse microvascular bleeding from several puncture sites. Laboratory values show a normal PT/INR, aPTT, and platelets. What is the most likely etiology of this patient's bleeding? A. lack of erythropoietin production B. nitric oxide deficiency C. over-stimulation of urokinase D. poor platelet adhesion E. shock liver

The correct answer is: D The patient has chronic renal disease and is coagulopathic from uremia. Impaired platelet function is a major determinant of uremic bleeding. Uremia predisposes toward bleeding due to platelet dysfunction with an inability to aggregate and adhere to the endothelium. Laboratory values will have a normal coagulation profile, normal platelets, but an abnormal bleeding time. Uremic patients that are bleeding can be treated with desmopressin which increases plasma levels of von Willebrand factor, factor VIII, and t-PA contributing to a shortened activated partial thromboplastin time (aPTT) and bleeding time. Answer A: Erythropoietin is a true endocrine hormone produced in the kidney by cells that sense the adequacy of tissue oxygenation relative to the individual's metabolic activity. Lack of erythropoietin may cause anemia and is typically due to chronic kidney disease. Answer B: NO is an inhibitor of platelet aggregation that is produced by endothelial cells and platelets. NO is increased in uremic patients, and NO synthesis inhibitors have been shown to normalize bleeding time in research studies. Answer C: Urokinase is a plasminogen activator that is normally present in the urine. It is the major activator of fibrinolysis in the extravascular compartment, in contrast to TPA which is largely responsible for initiating intravascular fibrinolysis. Answer E: Any cause of shock or hemodynamic instability can cause ischemic injury to the liver, known as ischemic hepatitis, shock liver, or hypoxic hepatitis. Hepatic synthetic function usually remains normal or is only mildly impaired; the prothrombin time is infrequently prolonged by more than three seconds. Bottom Line: Uremic patients have poor platelet function which causes increased bleeding.

A 23-year-old male presents with left-sided flank pain which radiates to the ipsilateral groin, nausea and vomiting. History reveals recurrent flank pain, which woke him from sleep this morning. Physical examination reveals an uncomfortable male in a moderate amount of distress. He appears to be in considerable pain and cannot seem to rest comfortably whatsoever on the exam table. Palpation of the abdomen reveals diffuse tenderness, which is more focal in the left lower quadrant, but no involuntary guarding is appreciated. Structural examination reveals tissue texture changes from T9-L1 on the left. A urine sample is obtained and is distinctly bloody. The urine is filtered and a kidney stone is discovered. The patient is educated that this particular type of kidney stone is prevented with acidification of the urine by drinking cranberry juice. The type of stone affecting this patient was most likely composed of A. ammonium uric acid B. calcium ammonium phosphate C. calcium oxalate D. calcium phosphate E.sodium uric acid

The correct answer is: D The patient most likely has a calcium phosphate kidney stone. Calcium phosphate stones account for 15% (2nd most common), and can be present with calcium oxalate or struvite stones. Calcium phosphate crystals are pH sensitive, forming in alkaline urine. They share this quality with struvite stones. They will not be found in conjunction with urate crystals since uric acid crystals form in acidic pH levels. Struvite stones are often considered with infections of urea-splitting bacteria (i.e. Proteus sp.). Calcium phosphate and oxalate, struvite, and cystine stones are radiopaque and can be seen on plain film X-ray. However, helical CT is preferred imaging, as small stones or stones overlying bony structures can be missed with plain film radiography. Answers A & E: Sodium and ammonium uric acid stones account for approximately 8% of analyzed stone samples. Sodium urate is also found in the joint fluid of patients with gouty arthritis. Similar to other kidney stones, uric acid is more common in an acidic environment, with urine pH 5.5 or lower. Risk factors also include low urine volume and high uric acid excretion. Thus the patient should avoid acidifying the urine, if he were affected by these types. Answer B: Struvite stones, which are made of magnesium ammonium phosphate (not calcium ammonium phosphate), form in alkaline urine. This is similar to the calcium phosphate stone afflicting this patient. Struvite stones are often correlated with urea-splitting bacteria such as Proteus species. Answer C: Calcium oxalate is the most common type of kidney stone, accounting for 70-80% all cases. The monohydrate form of these stones is shaped like either a dumbbell or needle-shaped, whereas the dihydrate crystals are envelope-shaped. They form in acidic urine and are more common when urine volumes are low. Bottom Line: Calcium phosphate stones are the second most common and, like struvite stones, form in alkaline urine. Prevention focuses on adequate hydration with correction of low urine volumes and acidification of the urine.

A 68-year-old male with past medical history of hypertension and dyslipidemia presents to the emergency department after an episode of chest discomfort. His episode lasted about 2 minutes while he was walking up a flight of stairs and resolved as he stopped to rest. Vitals reveal the following: Blood pressure: 150/88 mmHg Heart rate: 100/min Respiratory rate: 20/min Oxygen saturation: 94% on 2L nasal cannula Body mass index: 26 kg/m2 Physical examination reveals a normal S1 and S2 with a mid-systolic ejection murmur over the second right intercostal space. The neck is absent for jugular venous distention, but a systolic ejection murmur is appreciated at the carotid posts. Auscultation of the chest is clear without rales or rhonchi, and his chest pain is not reproducible upon palpation of the anterior chest wall. An electrocardiogram is obtained revealing left ventricular hypertrophy with normal ST segments. A plain chest radiograph is also obtained and read as unremarkable. The most likely mechanism causing chest discomfort is A. calcified aortic valve B. inflammation of costochondral junctions C.papillary muscle rupture D. reduced coronary artery perfusion E. thromboembolism of lung vasculature

The correct answer is: D This is a classic presentation of a patient with stable angina. Chest pain/discomfort present upon exertion with activities like walking and running which is relieved with rest is defined as stable angina. Although, unstable angina can also present as chest discomfort/pain, it is present at rest; whereas in this case patient had chest pain during exertion only. Coronary artery disease is a manifestation of reduced coronary artery perfusion and a supply-demand mismatch. A low supply state refers to a decreased blood flow to the coronary arteries secondary to calcification of the coronary arteries, vasospasm, anemia or shock. A high demand state refers to high oxygen demand states like those seen during exercise and left ventricular hypertrophy. In this scenario, the patient has a murmur of aortic stenosis (systolic ejection murmur with radiation to the neck) which creates a high demand state resulting in reduced coronary artery perfusion. As well as a low supply state secondary to coronary artery disease based on patient's risk factors including male with age greater than 55 years, obesity, history of dyslipidemia and hypertension. Answer A: Aortic stenosis results in a mid-systolic ejection murmur heard best over the second right intercostal space with radiation to the carotids, as present in this question stem. Calcified aortic valve is one of the two most common causes of aortic stenosis besides bicuspid aortic valve. Common clinical manifestations of aortic stenosis include syncope, angina and dyspnea. Although this patient is presenting with an angina episode which could be secondary to aortic stenosis, given this patient's age and other risk factors like dyslipidemia and hypertension, coronary artery disease with reduced perfusion is a more likely explanation of this patient's chest discomfort. Answer B: Although this patient is complaining of chest discomfort while walking, his pain is not reproducible upon palpation. Costochondritis typically is pain at the costochondral junction with the sternum secondary to inflammation. Pain is reproducible upon palpation and treated with anti-inflammatory medications such as NSAIDs. Answer C: Papillary muscle rupture after an acute myocardial infarction can result in mitral regurgitation. The murmur of mitral regurgitation is a high pitched holosystolic murmur with radiation to the apex instead of the carotids. However, aortic stenosis is heard louder over the second right intercostal space with radiation to the vessels in the neck such as carotid artery instead of the apex. Thus, papillary muscle rupture is not consistent with this patient's physical examination and presentation. Answer E: Venous thromboembolism in the lung vascular refers to pulmonary embolism. Pulmonary embolus can also present with chest pain, tachycardia and exertional dyspnea. Although most common presentation of pulmonary embolism on ECG is sinus tachycardia, a S1Q3T3 (S wave in lead I, lead III with a Q wave and inverted T wave) pattern can also be noted as well. Even though the patient has tachycardia, there is no evidence of hypoxemia or pleuritic chest pain. Also, this question does not give enough background information suggestive of risk factors associated with increased risk of thromboembolism such as prolonged immobilization, recent fracture, history of prior DVT and sedentary lifestyle. Bottom Line: Chest pain upon exertion is a classic presentation of stable angina which results due to decreased coronary artery perfusion during any activity that creates a high oxygen demand state.

A 65-year-old male who recently immigrated from Mexico presents with a cough and bloody sputum of 1 weeks' duration. History reveals that he feels fatigued throughout the day despite adequate sleep and can no longer perform his activities of daily living. He reports a 7 kg (15 lb 8 oz) weight loss over the last 2 months and decreased appetite. Social history is significant for a 40 pack-year history of smoking. Vitals reveal his temperature to be 37.5°C (99.5°F), pulse is 85/min, respirations are 25/min, and blood pressure is 155/85 mmHg. Physical exam reveals left supraclavicular fullness, rib 1 inhalational dysfunction on the left and paravertebral hypertonicity and tenderness from T2-6. A basic metabolic panel is obtained as follows: Sodium 144 mEq/L Potassium 4.6 mEq/L Chloride 100 mEq/L Bicarbonate 23 mEq/L BUN 30 mg/dL Creatinine 0.9 mg/dL Calcium 13.5 mg/dL A plain film radiograph of the chest is obtained as shown in the exhibit. Which of the following is the most likely diagnosis in this patient? A. adenocarcinoma of the lung B. sarcoidosis C. small cell carcinoma of the lung D. squamous cell carcinoma of the lung E. Tuberculosis

The correct answer is: D This patient has a large left hilar mass on chest x-ray, which is most likely squamous cell carcinoma given his extensive smoking history. He describes a clinical history of cough and hemoptysis consistent with lung cancer. His past social history of 40 pack-year smoking puts him at high risk for lung cancer. Hypercalcemia, which is seen on this patient's laboratory work-up, is very common with squamous cell carcinoma of the lungs. His hypercalcemia may be a result of bone metastasis or from the release of Parathyroid Hormone related peptide (PTHrP) from the squamous cell carcinoma. The PTHrP acts similar to PTH and causes an increase in calcium reabsorption in the distal tubules and promotes bone resorption by activating osteoclasts indirectly. Lung pathology can result in rib somatic dysfunction adjacent to the pathological lung segment. Additionally, you may see TART changes from T2-6 as a result of viscerosomatic reflexes from the underlying lung pathology. Answer A: Adenocarcinoma of the lung is more likely located on the lung periphery, more common in women, and less commonly associated with smoking and hypercalcemia. Answer B: Sarcoidosis can also cause hypercalcemia but the lesion is usually bilateral with hilar adenopathy. Patients can have other manifestations such as erythema nodosum on the shins. Hemoptysis is not characteristic. Granulomatous disease increases the concentration of calcitriol (1,25-dihydroxyvitamin D) instead of PTHrP. Answer C: Small cell carcinoma of the lung can cause Lambert-Eaton syndrome by producing antibodies against the pre-synaptic calcium channels but would not produce hypercalcemia. Answer E: Tuberculosis would most likely cause apical cavitary lesions not hilar masses and would not cause hypercalcemia. Bottom Line: Squamous cell carcinoma classically is found in patients with a unilateral hilar mass with hypercalcemia and a smoking history.

A 40-year-old obese female with a sedentary lifestyle presents with symptoms of never being able to quench her thirst and urinating excessively. Which of the following would confirm the most likely diagnosis in this patient? A. 2-hour plasma glucose of 150 mg/dL B. fasting glucose of 125 mg/dL C. hemoglobin A1C of 6.4% D. random glucose of 200 mg/dL E.urine glucose of 4+

The correct answer is: D This patient has symptoms and risk factors for diabetes mellitus. Patients presenting with these symptoms (polyuria and polydipsia) should be screened for diabetes mellitus. Screening for diabetes should begin at age 45-years-old every 3 years. However, screening should begin earlier at age 30-years-old if they have a 1st degree relative with diabetes mellitus, cardiovascular disease, sedentary lifestyle, non-Caucasian race, acanthosis nigricans and is overweight (BMI ≥ 25kg/m2) or obese, history of impaired glucose tolerance, hypertension, hyperlipidemia, history of gestational diabetes, history of a delivery of a baby > 4kg (9 lbs), polycystic ovaries, and/or psychiatric disease (schizophrenia). Testing for diabetes mellitus consists of performing one of the following tests and repeating the same test a second time to confirm the diagnosis: Fasting plasma glucose ≥ 126 mg/dL Random plasma glucose ≥ 200mg/dL with classic symptoms of diabetes (polyuria and polydipsia) Hemoglobin A1C ≥ 6.5% - average of glucose levels over approximately 90-120 days (life of a red blood cell). 2-hour plasma glucose ≥ 200mg/dL after 75 gram oral glucose tolerance test Answer A: A 2-hour plasma glucose ≥ 200mg/dL after 75 gram oral glucose tolerance test would confirm the diagnosis of diabetes. Answer B: A fasting glucose of ≥ 126 mg/dL is required for the diagnosis of diabetes mellitus. Answer C: The patient would need a hemoglobin A1C level of ≥ 6.5% to confirm the diagnosis. Hemoglobin A1C levels are an average of the plasma glucose levels on hemoglobin of approximately 90-120 days, which is the average lifespan of a red blood cell. Answer E: Urine glucose is not used to screen or diagnose diabetes mellitus because it is an insensitive test. Glucose can show up in the urine for many more reasons than diabetes: familial glucosuria, renal tubular acidosis, etc. On the other hand, screening for microalbuminuria is recommended in all patients with diabetes with a result greater than 30 mg/g indicating albuminuria. Microalbuminuria is a common finding in type 2 diabetes and is a risk factor for coronary artery disease. Microalbuminuria is an indicator of kidney disease in those with type 1 diabetes. Bottom Line: The following screening tests are used to diagnose diabetes mellitus: Fasting plasma glucose ≥ 126 mg/dL, random plasma glucose ≥ 200 mg/dL with symptoms (polyuria and polydipsia), hemoglobin A1C ≥ 6.5%, or a 2-hour plasma glucose ≥ 200 mg/dL after 75 gram oral glucose tolerance test. These same test should be repeated to confirm the diagnosis unless signs of metabolic decompensation (diabetic ketoacidosis or hyperosmolar hyperglycemic state). The best test for overt type II diabetes mellitus is fasting plasma glucose, as hemoglobin A1C can have approximately a 20% false negative in the screening setting.

A 30-year-old female with hypothyroidism and type 1 diabetes mellitus is brought to the emergency department after being found minimally responsive at home. Her husband reports that she has been complaining of weakness and fatigue over the past couple months and recently had mild diarrhea for the past few days due to a "stomach bug" going around her work. However, this morning she started having nausea, severe abdominal pain, and recurrent vomiting. Her current medications include levothyroxine and NPH insulin. Vital signs reveal the following: Temperature 39.1°C (102.4°F) Blood pressure 82/52 mmHg Heart rate 127/min Respiratory rate 22/min The patient appears confused, lethargic, and is only oriented to person. Physical examination reveals pallor, generalized weakness, and a soft, non-distended, diffusely tender abdomen. Laboratory studies reveal the following: Sodium 125 mEq/L Potassium 6.1 mEq/L Chloride 99 mEq/L Bicarbonate 19 mEq/L BUN 55 mg/dL Creatinine 4.3 mg/dL Glucose 57 mg/dL Cortisol (AM) 2.8 mcg/dL Which of the following most likely precipitated this event? A. adrenal adenoma B. dehydration C. excess insulin administration D. gastrointestinal upset E. pregnancy

The correct answer is: D This patient is experiencing adrenal crisis secondary to underlying chronic primary adrenal insufficiency, also known as Addison's disease. Addison's disease results from progressive destruction of the adrenal cortices leading to decreased synthesis of cortisol and mineralocorticoids. The most common cause in the United States is idiopathic/ autoimmune, however, tuberculosis is the most common cause worldwide. Other causes of primary adrenal insufficiency include CMV, Cryptococcus, toxoplasmosis, bilateral adrenalectomy, and adrenal metastatic disease. This patient's medical history is suggestive for polyglandular autoimmune syndrome type II. This term is used to describe an inherited disorder that leads to multiple autoimmune diseases, specifically Hashimoto thyroiditis, type 1 diabetes mellitus, and Addison's disease. The onset of each disease can vary but commonly manifests in early adulthood and is more common in females. Clinical symptoms of Addison's disease usually are not present until >90% of the adrenal glands have been destroyed. Symptoms include insidious onset of weakness, fatigue, anorexia, and hyperpigmentation. However, approximately 25% of patients with primary adrenal insufficiency will present initially with an adrenal crisis. This patient was experiencing mild symptoms of adrenal insufficiency (e.g., fatigue, weakness) over several months. Then, this recent viral gastroenteritis caused an acute increase in cortisol and aldosterone demand. Recent studies have found that gastrointestinal upset and fever are the two most common precipitating factors of crisis. Other triggers include trauma, surgery, infection/ sepsis, and severe emotional stress. Adrenal crisis usually presents with intractable nausea, vomiting, and abdominal pain. The abdominal pain can be so severe that it can be mistaken for an acute abdomen. Fever may be severe or absent. Patients quickly become intravascularly volume depleted, which progresses to cardiovascular collapse and hypovolemic shock. Classic laboratory values suggestive of adrenal crisis/ primary adrenal insufficiency include hyponatremia (due to decreased aldosterone), hyperkalemia (due to decreased aldosterone), hypoglycemia (due to decreased cortisol), and decreased cortisol (AM cortisol < 3 mcg/dL is diagnostic of adrenal insufficiency). ACTH would be high in Addison's disease due to lack of negative feedback from cortisol. The diagnosis of Addison's disease is made with an ACTH stimulation test. The administration of exogenous ACTH will fail to cause the expected increase in cortisol levels. Of note, this patient's increased BUN and creatinine indicated acute renal failure secondary to decreased kidney perfusion as a result of hypovolemic shock. Answer A: Adrenal adenoma is a common cause of Cushing's syndrome, a clinical diagnosis associated with excess glucocorticoid production. Patients with Cushing's syndrome may present with diabetes, hypertension, hypogonadism, proximal muscle wasting, osteoporosis, and/ or psychiatric disturbances. morning cortisol levels with be elevated (> 5.0) and ACTH levels will be low. This patient's presentation is more consistent with adrenal crisis secondary to primary adrenal insufficiency. Answer B: Dehydration refers to loss of total body water. Patients experiencing adrenal crisis are usually dehydrated due to excessive vomiting and poor oral intake. More importantly, they are at an increased risk of developing rapid, severe hypovolemia (decreased blood volume). This is due to decreased aldosterone, which prevents sodium, and therefore water, reabsorption. Decreased intravascular volume leads to hypotension, decreased cardiac output, decreased renal perfusion and shock. Dehydration is not a precipitating factor for adrenal crisis. Answer C: Excess insulin administration can result in hypoglycemia, which presents similarly with nausea, vomiting, abdominal pain and pallor. Other symptoms include palpitations, sweating, tachycardia, altered mental status and seizure. Hypoglycemia is not correct because it is not associated with a decreased cortisol level or the electrolyte abnormalities seen in this patient. It is important to note that patients with adrenal insufficiency can have hypoglycemia because decreased cortisol leads to decreased gluconeogenesis. Therefore, the development of Addison's disease in patients with pre-existing diabetes can lead to decreased insulin requirements due to this alteration in glucose regulation. Answer E: Adrenal crisis can be seen as a rare complication of pregnancy and occurs as a result of bilateral spontaneous adrenal hemorrhage. It is most commonly seen in pregnancies complicated by toxemia of pregnancy, spontaneous abortion, antepartum or postpartum hemorrhage, twisted ovarian cyst (in pregnancy), and primary antiphospholipid antibody syndrome. This patient's history and physical examination do not suggest pregnancy and are more consistent with underlying chronic adrenal insufficiency. Bottom Line: Addison's disease results from destruction of the adrenal glands. In the United States, it is most commonly caused by an autoimmune process and patients with other autoimmune diseases are at increased risk for developing primary adrenal insufficiency. Approximately 25% of patients with Addison's disease may initially present with adrenal crisis due to an impaired adrenocortical hormone response to severe physiological stress (e.g., trauma, infection, gastrointestinal upset, surgery). Clinical manifestations of adrenal crisis include nausea, vomiting, abdominal pain, cardiovascular collapse, and/or hypovolemic shock.

A 45-year-old female with history of alcoholism and depression presents to the emergency department complaining of epigastric pain and vomiting that began 2 days ago. She has also noticed a tingling sensation around her mouth and fingers that began this morning. Physical examination reveals an absence of bowel sounds, severe epigastric tenderness with guarding, prolonged painful muscle contractions, and 4+ deep tendon reflexes in both lower extremities. Laboratory studies reveal a lipase of 18,756 U/L and amylase of 2,486 U/L. ECG shows prolongation of the QT interval. The most likely additional finding upon laboratory analysis is A. hypercalcemia B. hyperkalemia C. hypermagnesmia D. hypocalcemia E. hypokalemia

The correct answer is: D This patient is presenting with acute pancreatitis likely secondary to alcohol abuse. This is evidenced by her abdominal examination findings and elevated lipase and amylase. The other laboratory finding that is most likely present in this patient is hypocalcemia. The cause of hypocalcemia in the setting of acute pancreatitis is multifactorial. Damage to the pancreas leads to the excess free fatty acid production due to increased activity of pancreatic lipase. Free fatty acids then bind insoluble calcium salts in the pancreas, forming calcium deposits that are distributed in the retroperitoneum. Other contributing factors include concomitant hypoalbuminemia and impaired parathyroid hormone secretion (due to glucagon-stimulated calcitonin release). Clinical manifestations of hypocalcemia are primarily caused by neuromuscular irritability and include parathesias, numbness, spontaneous tonic muscular contractions (tetany), and hyperreflexia. Classic physical exam findings include contraction of the facial muscles in response to palpation or tapping of the 7th cranial nerve (Chvostek's sign) and carpal spasm in response to BP cuff inflation (Trousseau' sign). Hypocalcemia can cause QT prolongation, which can lead to deadly arrhythmias. Management of acute, symptomatic hypocalcemia includes IV calcium gluconate and treatment of the underlying cause. Serum magnesium levels should be checked and, if low, corrected to allow for appropriate calcium repletion (magnesium is needed for adequate PTH secretion). Answer A: Hypercalcemia may present with personality changes, depression, peptic ulcer disease, nephrolithiasis, bone pain, constipation, lethargy, stupor and/or coma depending on severity and duration. Hypercalcemia does not typically present with neuromuscular excitability as presented in the case. Also, hypercalcemia is associated with shortening of the QT interval, not prolongation. Answer B: Hyperkalemia is also associated with QT prolongation (due to widening of the QRS complex). Unlike hypocalcemia, hyperkalemia causes muscle weakness, flaccid paralysis, and decreased deep tendon reflexes. Also, hyperkalemia is not a common electrolyte abnormality associated with acute pancreatitis. Answer C: Hypermagnesemia may present with weakness, nausea, vomiting, bradycardia, and decreased or absent deep tendon reflexes. It does not result in neuromuscular excitability. ECG changes associated with hypermagnesemia are similar to those seen in hyperkalemia and include PR prolongation, widening of QRS, and T wave elevation. Also, hypomagnesemia, not hypermagnesemia, is commonly seen in acute pancreatitis due to redistribution of magnesium from the extracellular to the intracellular space. Answer E: Hypokalemia may present with nausea, vomiting, muscle weakness, cramps, and decreased deep tendon reflexes. ECG changes include T wave flattening, T wave inversion, ST segment depression, and U wave formation. Hypokalemia is not associated with neuromuscular excitation or QT prolongation. Bottom Line: Hypocalcemia is a serious complication of severe acute pancreatitis. Patients may present with symptoms of neuromuscular excitability including involuntary contractions, parasthesias, and rarely tetany. The classic ECG finding associated with hypocalcemia is prolongation of the QT interval. Treatment requires immediate administration of IV calcium gluconate and adequate treatment of the underlying cause.

A 42-year-old male patient presents to the office with the complaint of excruciating pain in his right great toe. History reveals that he awoke this morning with an exquisitely tender and warm toe. He has a past medical history of obesity, alcohol abuse, and hypercholesterolemia. At his most recent appointment an additional medication to treat his cholesterol was added, but he cannot recall the name. Physical examination today reveals an erythematous, warm, and tender right great toe. The medication he was likely started on that resulted in his current condition is A. cholestyramine B. ezetimibe C. gemfibrozil D. niacin E.simvastatin

The correct answer is: D This patient is presenting with an acute gouty attack as evidenced by the erythematus, warm, tender right great toe. Niacin was added to his statin therapy at his last appointment. Niacin is a component of two co-enzymes necessary for lipid metabolism, tissue respiration, glycogenolysis, and inhibition of very low-density lipoprotein (VLDL) synthesis. It is associated with glucose intolerance, pruritis from a transient histamine release, and elevation in uric acid levels. It is not a first-line therapy for hyperlipidemia, however it is often added to statin therapy if statins alone are not controlling the hypercholesterolemia. Niacin is also known to help raise HDL even more than statins, exercise, and tobacco cessation. Answer A: Cholestyramine inhibits enterohepatic re-uptake of intestinal bile salts thus increasing fecal loss of bile salt-bound LDL and reducing serum cholesterol. Adverse effects include flatus and abdominal cramping. It also has significant drug-drug interactions due to its ability to block absorption. Answer B: Ezetimibe inhibits a sterol transporter at brush border thus inhibiting intestinal absorption of cholesterol, decreasing delivery of cholesterol to the liver, and reducing hepatic cholesterol stores. It is effective at lowering LDL levels, however is not better than placebo in regards to clinical endpoints such as stroke, myocardial infarction, or death. It is well tolerated with minimal side effects. Answer C: Gemfibrozil inhibits peripheral lipolysis, decreases hepatic uptake of free fatty acids leading to inhibition of VLDL secretion, and may increase HDL Cholesterol by an unknown mechanism. It leads to a greater triglyceride lowering potential than statins, however has not been shown to be associated with a mortality benefit equal to that of statin therapy. There is an increased risk of myositis when used in combination with statins. Answer E: Statin therapy is the first-line therapy for hypercholesterolemia. They work by inhibiting HMG-CoA reductase which catalyses the rate-limiting step in cholesterol biosynthesis. Their main side effects include an elevation in liver transaminases and myositis. Bottom Line: Niacin adverse effects include glucose intolerance, pruritus, and an elevation in uric acid levels.

A 25-year-old male is being transferred to your hospital from an outside facility after a head on motor vehicle collision. A CT scan displays a cervical vertebral burst fracture with compression of the left half of his spinal cord. Brown-Sequard syndrome is suspected. Which of the following neurological findings is most consistent with this diagnosis? A. bilateral loss of motor control and bilateral loss of pain and temperature sensation B. contralateral loss of motor control and contralateral loss of pain and temperature sensation C. contralateral loss of motor control and ipsilateral loss of pain and temperature sensation D. ipsilateral loss of motor control and contralateral loss of pain and temperature sensation E. ipsilateral loss of motor control and ipsilateral loss of pain and temperature sensation

The correct answer is: D True Brown-Sequard syndrome may occur after injury to one half of the spinal cord. This lateral hemisection syndrome results in injury to corticospinal tract, spinothalamic tract and the dorsal column unilaterally. Symptoms include ipsilateral weakness (corticospinal tract), ipsilateral loss of vibration and proprioception (dorsal column tract) along with contralateral loss of pain and temperature sensation (spinothalamic tract). Posterior column and spinothalamic loss occur on opposite sides of the body in this syndrome secondary to spinothalamic decussation near the level of the lesion (one or two levels below the injury). This injury may occur due to penetrating trauma (knife or bullet), disc herniation, vasculitis, infarction and demyelination. The prognosis and potential for recovery are variable depending on the injury. Answer A: Posterior column and spinothalamic loss occur on opposite sides of the body in this syndrome, therefore making bilateral loss a wrong answer choice. Answer B: Posterior column and spinothalamic loss occur on opposite sides of the body in this syndrome secondary to spinothalamic decussation near the level of the lesion (below the injury), therefore making contralateral lesions for both a wrong answer choice as well. Answer C: Posterior column and spinothalamic loss occur on opposite sides of the body in this syndrome secondary to spinothalamic decussation near the level of the lesion (below the injury), which is exact opposite of this answer choice, making it an incorrect answer choice as well. Answer E: Posterior column and spinothalamic loss occur on opposite sides of the body in this syndrome secondary to spinothalamic decussation near the level of the lesion (below the injury), therefore making ipsilateral lesions for both a wrong answer choice as well Bottom Line: Brown-Sequard syndrome symptoms include ipsilateral loss of motor control and contralateral loss of pain and temperature sensation.

A 57-year-old male with end-stage renal disease secondary to poorly controlled type I diabetes mellitus presents to you for creation of a dialysis fistula. Which of the following is the most ideal site for creation of an AV fistula? A. brachial artery and axillary vein B. brachial artery and basilic vein C. brachial artery and median antecubital vein D. femoral artery and femoral vein E. radial artery and cephalic vein

The correct answer is: E AV fistulas are typically constructed with an end-to-side vein-to-artery anastomosis between an artery and vein. In creation of dialysis access fistulas, using the most distal vessels of the upper extremity is recommended. Creation of more proximal fistulas prevent potential future placement of distal AV fistulas. Answer A: Fistulas are preferred to grafts given their long-term patency and lower rate of complications. Common graft locations are straight forearm (radial artery to cephalic vein), looped forearm (brachial artery to cephalic vein), straight upper arm (brachial artery to axillary vein), or looped upper arm (axillary artery to axillary vein). Answer B: If neither a radiocephalic fistula nor a brachiocephalic fistula is possible, then a transposed brachiobasilic fistula should be considered. Answer C: Fistulas are less commonly created between the brachial artery and median antecubital vein. Answer D: With a vein transposition fistula, the vein is moved or transposed to a position that is better suited for the construction of a fistula. These are uncommon but sometimes performed at the femoral site if clinically necessary.

A 62-year-old female with a history of hypothyroidism presents with insidious onset of hand weakness that has progressively been getting worse over the past couple months. She reports the presence of paresthesias over her radial three fingers. The most appropriate physical examination technique to support the diagnosis is A. Allen test B. Cozen test C. Finkelstein test D.Murphy test E. Phalen test

The correct answer is: E Carpal tunnel syndrome is most commonly due to an inflamed synovium. Risk factors include obesity, pregnancy, hypothyroidism, rheumatoid arthritis, diabetes mellitus, repetitive motion activities, and advanced age. The median nerve produces sensation to the radial 3 ½ fingers, and patients will initially complain of paresthesias or numbness in this distribution. Other symptoms include clumsiness and pain that awakens the patient at night. It is worsened with repetitive use. In advanced disease, patients may present with thenar atrophy as the median nerve innervated the flexor muscles of the thumb. Physical examination will reveal positive Phalen and Tinel's tests. In the Phalen test, the patient flexes their wrists together and holds it there for one minute. It is positive if the symptoms are reproduced. Tinel's test involved tapping on the median nerve. Treatment is with night splints, activity modification, NSAIDs, and steroid injections. Answer A: The Allen test is used to test the patency of the radial and ulnar arteries of the arm. The patient elevates the hand and proceeds to make a fist for 30 seconds while pressure is applied over both the radial and ulnar arteries in order to blanch the hand. The patient is then asked to open the hand and pressure over the ulnar artery is released. The number of seconds it takes for the hand to return to normal color is recorded. The same procedure is repeated and pressure is released over the radial artery. For the ulnar artery, if normal color does not return within 7 seconds than the Allen test is considered positive and the ulnar artery supply to the hand is not sufficient. It is commonly performed prior to coronary artery bypass grafting because the radial artery is commonly used as a graft. If the Allen test takes longer than 5 seconds for the radial artery, then it is not considered patent, and is thus insufficient to be used for bypass grating. Answer B: The Cozen test is a test for lateral epicondylitis, also known as tennis elbow. Lateral epicondylitis is an overuse syndrome of the extensor tendons. The extensor tendons insert onto the lateral epicondyle of the elbow. Tennis players often suffer lateral epicondylitis as a result of pressure grip strain, or due to backhand shots performed with a leading elbow. Lateral epicondylitis is caused by microtearing of the extensor carpi radialis brevis tendon. Physical exam reveals local tenderness over the origin of the extensor tendons on the lateral epicondyle. The Cozen test is positive if resisted wrist extension triggers pain over the lateral epicondyle over the humerus due to stress placed upon the extensor carpi radialis brevis tendon. Treatment is with activity modification, ice, NSAID's, and a counter-force brace. Answer C: Finkelstein test is used to test for De Quervain's tenosynovitis. De Quervain's is a stenosing tenosynovitis of the 1st dorsal compartment of the hand. This includes the abductor pollicis longus and the extensor pollicis brevis tendons located over the styloid process of the radius. It is most often due to overuse of the wrist and hand, and is also associated with pregnancy, direct trauma, and rheumatoid arthritis. It is most commonly seen in women between 30 and 50 years of age. Patients with de Quervain's tenosynovitis typically note pain at the radial side of the wrist during pinch grasping, or during thumb and wrist movement. The patient may complain of difficulty gripping and radial sided wrist pain. Physical exam will reveal tenderness over the radial styloid. A positive Finkelstein test will also be demonstrated. It is performed by ulnar deviation of the wrist with the thumb clenched in fist. Treatment is with NSAIDS, wrist splints, and steroid injections are best initial treatment. Answer D: Murphy's test is used to asses gallbladder pathology. Murphy's test is performed by having the patient take a deep inspiration and simultaneously exerting pressure over the right upper quadrant. A positive Murphy test will cause abrupt cessation of inspiration due to pain. This test will be positive in cases of acute cholecystitis. Bottom Line: Carpal tunnel syndrome is due to compression of the median nerve at the level of the wrist. Physical examination tests that will reproduce the symptoms include Phalen's test and Tinel's test.

A 46-year-old male with past medical history of congestive heart failure presents to clinic for a routine wellness examination. Home medications consist of metoprolol, lisinopril and furosemide. Review of systems is negative for any chest pain, but does complain of some exertional dyspnea. Vitals are all within normal limits. Cardiovascular examination reveals a normal S1 and S2 with no murmurs. Lung examination reveals bibasilar crackles with no accessory muscle use. Extremities show bilateral peripheral edema of the ankles. Prevention of heart failure exacerbation should include monitoring of which of the following electrolytes? A. bicarbonate B.chloride C. magnesium D. potassium E. sodium

The correct answer is: E Congestive heart failure exacerbation is one of the leading causes of hospital admissions in the country. Therefore, prevention of heart failure exacerbation will significantly help reduce the health care costs. During outpatient visits of patients with history of congestive heart failure, a history and physical examination should include assessment of symptoms, functional capacity and fluid retention. Some of the common symptoms of heart failure include dyspnea on exertion, paroxysmal nocturnal dyspnea, orthopnea, peripheral edema and non-specific abdominal symptoms like nausea and vomiting. Common physical findings in heart failure exacerbation include peripheral edema, pulmonary crackles, elevated jugular venous pressure, hepatojugular reflex, displaced apical impulse, and S3 gallop. Given the pulmonary crackles and peripheral edema with exertional dyspnea, our patient's clinical presentation is consistent with heart failure exacerbation. Prevention of heart failure is primarily focused on treating the underlying lifestyle changes, medication adherence, and aggressive management of co-morbid conditions. Major risk factors and co-morbid conditions requiring early intervention to prevent heart failure include hypertension, dyslipidemia, diabetes, obesity, metabolic syndrome and coronary artery disease. Medication regimen typically focuses on regression or reverse modeling of the left ventricle with beta blocker, diuretics and ACE inhibitor or Angiotension receptor blocker therapy. Lifestyle changes include exercise, weight loss and a low sodium diet. Reduced sodium intake is a key step for prevention in non-pharmacologic management of heart failure. Water always follows sodium; therefore, monitoring sodium intake is key in monitoring plasma volume in patients. Increased sodium intake results in increased fluid retention which further decreases the cardiac output. Sodium restriction is necessary to prevent expansion of the extracellular fluid volume and to prevent peripheral edema to decrease the myocardial wall stress and improve renal flow. Therefore, sodium intake should be reduced to less than 2 g/day to prevent heart failure exacerbation. Answers A & B: Chloride and bicarbonate are not routinely monitored in patients with heart failure except in patients with kidney injury. Therefore, sodium is a better answer choice for prevention of heart failure exacerbation. Answer C: Low magnesium levels in heart failure patients can increase the risk of ventricular arrhythmias like torsades de pointes and ventricular tachycardia. However, sodium is more closely associated with heart failure exacerbation, making magnesium a less likely answer. Answer D: Potassium levels should be monitored closely in patients on any potassium sparing medications like ACE inhibitors and spironolactone. Also, low salt substituents for heart failure patients often contain potassium chloride, which should be used in moderation due to risk of hyperkalemia in patients on potassium sparing medications. However, low sodium intake is more important in prevention of heart failure exacerbation than potassium, making it a less likely answer choice. Bottom Line: Reduced sodium intake is very important in prevention of heart failure exacerbation.

A 18-year-old male with dysgeusia, anosmia, and growth retardation would have a deficiency in.... A. calcium B. selenium C. vitamin B1 D. vitamin B3 E. zinc

The correct answer is: E In Middle Eastern countries, chronic zinc deficiency can result in stunted growth in children with hypopigmented hair. A mild deficiency is associated with decrease taste sensation (hypogeusia), night blindness, and decreased spermatogenesis. Severe deficiency may present with diarrhea, alopecia, pustular dermatitis, and decreased immunity with frequent infections. Some foods rich in zinc include oysters, beef, crabs and cereal.

A 45-year-old male presents with swelling in his lower extremities bilaterally. History reveals he was recently diagnosed with hypertension and just started a new medication. Laboratory studies are obtained and reveal the following: Na 140 mmol/L K 3.9 mmol/L Cl 104 mmol/L CO2 24 mmol/L BUN 16 mg/dL Creatinine 0.7 mg/dL The most likely pharmacologic agent causing these symptoms is A. chlorthalidone B. furosemide C. lisinopril D. metoprolol E. nifedipine

The correct answer is: E Nifedipine is a dihydropyridine calcium channel blocker. The most common adverse effect associated with nifedipine is peripheral edema; followed by headache, nausea, heartburn and transient hypotension. Mechanism of action of nifedipine is by blocking L-type calcium channels (slow channels) in the heart and vascular smooth muscle causing vasodilation. However, the effects of nifedipine are more specific for vascular smooth muscle relaxation vs. myocardium relaxation, therefor, peripheral edema is common due to systemic vasodilation. Alternatively, non-dihydropyridines (verapamil or diltiazem) are more specific for the myocardium affecting contractility. Many anti-hypertensive medications are associated with electrolyte abnormalities, however, nifedipine is not associated with abnormal laboratory values. Given the patient's physical exam finding of peripheral edema in the setting of normal lab values, nifedipine is the most likely pharmacologic agent causing this patient's symptoms. Answer A: Chlorthalidone is a thiazide diuretic. The most common side effects associated with chlorthalidone are GI symptoms such as dyspepsia or poor appetite, hypokalemia and photosensitivity. The patient's renal panel is within normal limits and does not show hypokalemia. It also not associated with peripheral edema. Answer B: Furosemide is a loop diuretic. Most common electrolyte abnormalities associated with furosemide are hypochloremia, hypokalemia, hypomagnesemia, hyponatremia, and hypocalcemia (hypercalciuria). The patient's laboratory result does not show any of the electrolyte abnormalities associated with a loop diuretic. Furosemide is commonly used to treat edema secondary to congestive heart failure, it does not cause peripheral edema. It is important to note that loop diuretics are not effective for edema secondary to calcium channel blockers. Answer C: Lisinopril is an ACE inhibitor. The most common side effects are dry cough, hyperkalemia, acute renal failure in renal artery stenosis, and angioedema. Patient's lab does not show any hyperkalemia, making lisinopril an unlikely answer. It is also not associated with peripheral edema. Answer D: Metoprolol is a beta blocker and is considered second line therapy in lone hypertension. While it is not associated with significant laboratory abnormalities, it is also not associated with peripheral edema. Common adverse effects of a beta blocker includes: bradycardia, hypotension, dizziness, decreased libido. Bottom Line: Calcium channel blockers both dihydropyridines and non-dihydropyridines cause peripheral edema, facial flushing, headaches, GI upset, and constipation. They are not associated with abnormal laboratory values.

A 45-year-old alcoholic male presents with generalized weakness. Physical examination is unremarkable except for bilateral temporal wasting in a thin cachectic male. He is admitted to the general medical floor and provided IV maintenance fluids with thiamine, folate, and a multivitamin in isotonic saline with 5% dextrose at 125 mL/hour and a general diet. Twelve hours later the patient is noted to have an episode of generalized tremors of the entire body with associated confusion. Previous to this he was alert and oriented. Vitals now are significant for a blood pressure of 85/60 mm Hg, respiratory rate of 20/min, and heart rate of 100/min. A complete metabolic panel is obtained and reveals the following: Sodium: 140 mEq/L Potassium: 3.3 mEq/L Chloride: 99 mEq/L Bicarbonate: 20 mEq/L BUN: 30 mg/dL Creatinine: 1.5 mg/dL Calcium: 7.7 mg/dL Phosphorus: 0.9 mmol/L Magnesium: 1.5 mEq/L Albumin: 2.8 g/dL Which of the following abnormalities most likely contributed to this patients clinical deterioration? A. calcium B. creatinine C. glucose D. magnesium E. phosphorous

The correct answer is: E Patients other than those with anorexia nervosa are at risk for the refeeding syndrome. These include malnourished elderly patients, some surgical patients, those on chemotherapy, and homeless or alcoholic patients who have not eaten for many days. This patient's presentation with bilateral temporal wasting and history of daily consumption of alcohol is suggestive of malnutrition and alcoholism, which likely contributed to refeeding syndrome. Phosphate, calcium, potassium, magnesium and thiamine are some of the nutrients often found to be depleted in chronic alcoholics. Although phosphate depletion is common in these patients, the absolute fall in the serum phosphate below 1 mg/dL may not happen until 24 hours after hospitalization due to transcellular shift from iatrogenic interventions. Most alcoholics are initially treated with a "banana bag" which usually contains dextrose and thiamine. The dextrose in the solutions directly stimulates insulin release resulting in increased uptake of phosphate by cells. Additionally, alcohol ketoacidosis presents with metabolic acidosis with compensatory respiratory alkalosis causing further intracellular shift of phosphate as well. Therefore, hypophosphatemia is the most common clinical finding observed in alcohol withdrawal patients in the hospital. Hypophosphatemia can result in myopathy due to phosphate depletion causing significant rhabdomyolysis. Additional rare clinical findings include respiratory failure and severe central nervous system dysfunction including delirium, generalized seizures and coma as present in this patient. Answer A: Hypocalcemia can also present with neuromuscular irritability ranging from mild paresthesias to carpopedal spasm to seizures. Although this patient has a low serum calcium level of 7.7 mg/dL, it should be corrected for the serum albumin level. In a chronic alcoholic with malnutrition, a low albumin level is to be expected. For every 1g decrease in albumin, serum calcium can be increased by 0.8 since albumin binds free calcium making the corrected serum calcium level to be normal at 8.7 mg/dL. Answer B: Although this patient has an elevated creatinine, it is probably a combination of pre-renal azotemia from long-standing dehydration as well as intrinsic tubular injury from rhabdomyolysis. Progressive tubular injury can result in significant reduction in kidney function causing uremia and altered mental status. However, moderate elevation of creatinine does not explain this patient's seizure activity making it an incorrect answer choice. Answer C: In any patient with a seizure or syncopal episode, hypoglycemia must be excluded first even before any imaging is performed. Although this question stem does not give you the absolute glucose value, it does give a background of banana bag administration which normally contains dextrose making hypoglycemia a less likely answer choice. Answer D: Chronic alcoholics with malnutrition are often noted to have low calcium, low phosphate and low magnesium levels. Hypomagnesemia can also present with neuromuscular hyperexcitability like convulsions, delirium and coma. Other common signs include hypocalcemia and hypokalemia due to magnesium being a key component for cellular uptake of calcium and potassium. However, these signs are typically seen with magnesium levels below 1.0 mEq/L, much lower than our patient with a level of 1.5 mEq/L making it an incorrect answer choice. Bottom Line: Hypophosphatemia can present with neuromuscular instability including seizures and coma in alcoholics post hydration and feeding.

A 54-year-old male presents to the emergency department with generalized abdominal pain. An upright chest radiograph demonstrates free air under the diaphragm and the patient is taken to the operating room where he is found to have sigmoid diverticulitis with feculent peritonitis. Due to intraoperative hemodynamic instability, he is washed out, resected with stapled ends, and a temporary abdominal closure device is placed. On postoperative day two his morning labs are significant for a creatinine of 2.0 mg/dL. Which of the following labs will best differentiate his azotemia as prerenal rather than an intrinsic in nature? A. fractional excretion of sodium of 3% B. urine : plasma urea < 3 C. urine creatinine : plasma creatinine ratio of < 20 D. urine osmolality of 285 mOsm E. urine sodium of < 20 mOsm/L

The correct answer is: E Prerenal causes of acute renal failure stem from any pathologic process that steals renal perfusion. Dehydration, hemorrhagic shock, septic shock, hepatorenal syndrome, and abdominal compartment syndrome are all perceived by the kidney as an inadequate blood volume. The end result is a stimulation of renin release by the juxtaglomerular cells. This cascade ultimately produces aldosterone which acts on the principle cells of the collecting ducts to salvage sodium in exchange for either potassium or hydrogen. Water reabsorption is always passive and secondary to solute transport thus blood volume is restored as sodium is reclaimed from the renal filtrate. Understanding this physiologic mechanism allows for a more logical differentiation of prerenal and intrinsic causes of azotemia. Answers A & D: As sodium is resorbed in the collecting ducts, the concentration remaining in the filtrate, and thus, urine (UNa) will decline to a level < 20 mOsm. Therefore, the fraction of filtered sodium that is actually excreted (FeNa) in the urine will be 1% or less. This effort to preserve blood volume leads to a concentration of the urine with a rise in osmolality to > 500 mOsm/L. Answers B & C: These are both consistent with intrinsic sources of renal failure. Causes of intrinsic renal failure include pathology of the vessels, glomeruli, or tubules-interstitium. Bottom Line: Prerenal acute renal failure stimulates release of aldosterone which leads to resorption of sodium from the collecting ducts. This will cause a fall in urine sodium to < 20 mOsm/L and fractional excretion of sodium to 1% or less. The urine becomes more concentrated than plasma.

An 11-year-old patient with sickle cell disease is brought to the emergency department complaining of pain in his left knee for 1-day duration. He denies any recent trauma to the affected knee. His temperature is 38.4°C (101.1 °F), heart rate is 103/min, blood pressure is 118/78 mmHg, and respiratory rate is 15/min. On physical examination the left knee is swollen, tender, warm to touch, and erythematous. Regional range of motion of the left knee is limited due to pain and swelling. Plain radiograph of the knee reveals periarticular soft tissue swelling without periosteal involvement. Which organism is most likely responsible for this patient's condition? A. Campylobacter jejuni B. Eikenella corrodens C. Neisseria gonorrhoeae D. Salmonella enteritidis E. Staphylococcus aureus

The correct answer is: E Sickle cell disease is associated with several musculoskeletal disorders due to repetitive tissue and bone infarction causing necrosis. Areas of infarction are at increased risk for infection, including septic arthritis and osteomyelitis. Septic arthritis presents in sickle cell patients similarly to how it presents in the general population with swelling, erythema, pain and decreased range of motion. It usually involves large joints, most commonly the knee and less commonly the hip, shoulder, or elbow. Plain radiograph will show soft tissue swelling and joint space narrowing without periosteal involvement (if present, indicates osteomyelitis). The most common pathogen associated with septic arthritis is Staphylococcus aureus in all populations, including sickle cell patients. Other microorganisms that can cause septic arthritis in sickle cell disease include Streptococcus, Escherichia coli, and Salmonella. Salmonella infection is much less common in sickle cell patients with septic arthritis compared to osteomyelitis where Salmonella infections (most commonly S. typhimurium) make up more than half of cases. Answer A: Campylobacter jejuni is associated with gastroenteritis. Reiter's syndrome is a rare complication of C. jejuni gastroenteritis and presents with a reactive polyarthritis several weeks after the initial enteric infection. Patients commonly report asymmetric oligoarthritis affecting mainly the knees, ankles and feet. This patient presented with a monoarticular arthritis and no history of gastroenteritis. Answer B: Eikenella corrodens is a common bacteria that makes up normal human oral flora. It can cause septic bacterial arthritis in patients who are bit by another human. Infection is caused by direct introduction of the pathogen into the joint at the site of trauma. This patient denied trauma, making this choice unlikely. Answer C: Neisseria gonorrhoeae is a common cause of arthritis in young adults between 20-40 years of age and is a consequence of bacteremia arising from gonococcal infection. Signs and symptoms of disseminated gonococcal infection including fever, chills, papular rash on trunk and extensor surfaces, and a migratory arthritis involving large joints usually precede true gonococcal septic arthritis. Gonococcal arthritis would be rare in an 11-year-old without mention of sexual activity or abuse. Answer D: Salmonella species cause over half of cases of osteomyelitis in sickle cell patients. They are also a less common but potential cause of septic arthritis in this population. Common isolates include S. typhimurium and S. paratyphi. S. enteritidis is not associated with septic arthritis or osteomyelitis in sickle cell disease. However, it has been associated with reactive arthritis (Reiter's syndrome) following gastroenteritis, most commonly in persons with HLA-B27 histocompatibility antigen. Bottom Line: Sickle cell disease predisposes patients to infections of the joint and bone. Pathogens associated with septic arthritis in sickle cell include S. aureus, Streptococcus, E. coli, and Salmonella. The latter is not seen as frequently in septic arthritis as it is in osteomyelitis.

A 55-year-old female with past medical history of systemic lupus erythematosus presents to the emergency department complaining of dizziness. Patient states two weeks ago she had arthralgias and myalgias which she attributed to a lupus flare up and was treated with a short course of prednisone. Review of systems is also positive for abdominal pain and diarrhea for the past three days. Vital signs include temperature of 37.6ºC (99.8ºF), blood pressure of 90/60 mm Hg, heart rate of 100/min, and respiratory rate of 16/min. Oral mucosa appears dry with no ulcers. Cardiovascular examination is unremarkable. Abdominal examination reveals normal bowel sounds with mild diffuse tenderness to palpation of the abdomen. Laboratory studies reveal: Basic Metabolic Panel: Sodium 148 mEq/L Potassium 4.8 mEq/L Chloride 109 mmol/L Bicarbonate 22 mEq/L Blood urea nitrogen 42 mg/dL Creatinine 1.8 mg/dL Random, urine sodium 14 mEq/L Random, urine creatinine 22 mg/dL Urinalysis: pH 5.0 Color amber Specific gravity 1.018 Protein trace Nitrite negative Bilirubin negative RBC 4 WBC 2 Casts hyaline The most likely etiology of this patient's elevated creatinine is A. acute glomerulonephritis B. acute interstitial nephritis C. acute tubular necrosis D. obstructive uropathy E. pre-renal azotemia

The correct answer is: E The clinical scenario of a patient with diarrhea strongly hints towards possible volume depletion. Additional objective clinical data like dry mucous membranes should further confirm the suspicion for dehydration as well. Laboratory data with an elevated creatinine and blood urea nitrogen is consistent with acute kidney injury. AKI secondary to dehydration can result in both pre-renal azotemia and acute tubular necrosis (ATN). Therefore, other tools are needed to further distinguish between the two. 1. Fractional excretion of sodium (FENa) = 100 X (Serum creatinine/Urine creatinine)/(Serum sodium / urine sodium) FENa = 100 X (1.8/22)/(148/14) = 0.76 % , which is consistent with pre-renal azotemia. FENa is less than 1% in pre-renal azotemia and greater than 2% in ATN and AIN (acute interstitial nephritis). 2. Ratio of blood urea nitrogen/serum creatinine BUN/Cr = 23.3, which is consistent with pre-renal disease BUN/Cr is greater than 20:1 in pre-renal disease and either normal or 10 to 15 : 1 in ATN and AIN. 3. Urine sodium Urine sodium is usually less than 20 mmol/L in pre-renal azotemia whereas greater than 40 mmol/L in acute tubular necrosis and post-renal obstruction. Therefore, pre-renal azotemia is the most likely diagnosis of acute kidney injury in this patient. Also, given patient's recent flare up of lupus, glomerulonephritis and interstitial nephritis are important differential diagnosis; however, given no significant proteinuria, hematuria and casts in the urinalysis they are less likely options. Answer A: Acute glomerulonephritis can be one of the initial presentations of renal disease manifestations in systemic lupus erythematosus (SLE). Although a recent flare up of SLE can be suggestive of a trigger for glomerulonephritis, a bland urine sediment makes this an unlikely diagnosis. Besides an elevated creatinine, the urine sediment in lupus nephritis includes significant proteinuria, red cell casts and hematuria as well, which is not present in this patient's urinalysis. Other serum markers include elevated anti-dsDNA titers and low complement levels. Answer B: Acute interstitial nephritis (AIN) can also present with elevated creatinine. History of lupus and recent infection (diarrhea) in this patient can be suggestive of tubulointerstitial nephritis; however, a bland urine sediment makes it a less likely option. AIN typically presents with white cell, white cell casts and eosinophils in the urinalysis. Answer C: Acute tubular necrosis (ATN) can often be difficult to distinguish from pre-renal azotemia in patients with acute kidney injury. Mainly because dehydration initially results in pre-renal azotemia, but prolonged hypoperfusion of the kidneys can result in development of acute tubular necrosis. Other common etiologies include sepsis, drugs and toxins. Besides FENa and urine sodium as discussed above, another important clue in this question making ATN less likely is the urine sediment. ATN patients usually have epithelial casts or "muddy brown" granular casts; whereas hyaline casts are a very non-specific finding for concentrated urine. Answer D: Obstructive uropathy can also present with elevation of creatinine with a fairly normal urinalysis depending on the etiology of obstruction. Obstruction can be present due to stone, foreign object, benign growth or tumor. Imaging is typically needed to figure out the exact cause of obstruction. However, typically FENa is > 1% and BUN/Cr is normal-high but less than 20:1, making this a less likely diagnosis. Bottom Line: In a patient with acute kidney injury, FENa less than 1% and BUN/Creatinine ratio greater than 20:1 is consistent with pre-renal azotemia.

A 45-year-old female with end stage renal disease presents for a routine examination. Physical exam reveals a tender nodule one inch superior and one inch lateral on the left side of the umbilicus. During the recording of her vital signs the patient displays a positive Trousseau's sign. A review of her most recent laboratory results reveals a serum calcium of 6.7 mg/dL. The most likely physiologic explanation for this patient's abnormal result is A.elevated parathyroid hormone B. inadequate intake C. low serum phosphate D. secondary to dialysis E. vitamin D deficiency

The correct answer is: E The most common cause of an acquired decrease in renal production of 1,25-dihydroxyvitamin D is chronic kidney disease (CKD). It typically does not occur until end stage CKD (G5, eGFR < 15 mL/min or treatment by dialysis). Gut absorption of calcium is decreased when there is a deficiency in renal vitamin D metabolism leading to hypocalcemia. Manifestations of hypocalcemia include Chvostek's sign, Trousseau's sign, and prolonged QT interval. Trousseau's sign is the induction of carpopedal spasm (characterized by adduction of the thumb, flexion of the metacarpophalangeal joints, extension of the interphalangeal joints, and flexion of the wrist) by inflation of a sphygmomanometer above systolic blood pressure for three minutes. Osteopathic findings may be present as with any other renal pathology. In this example you are given the anterior chapman point for the left kidney. Answer A: An increased parathyroid hormone (PTH) level would cause hypercalcemia, not hypocalcemia. Answer B: The patient's hypocalcemia is more likely due to a poor absorption of calcium, not an inadequate intake. Answer C: A decreased phosphate level would be associated with hypercalcemia, not hypocalcemia. In contrast to other forms of hypocalcemia associated with vitamin D deficiency, hypocalcemia in CKD is also due to the occurrence of hyperphosphatemia (due to reduction in the filtered phosphate load and a reduction in the fractional excretion of phosphorus). Answer D: Dialysis is not a physiologic cause of hypocalcemia. Bottom Line: Gut absorption of calcium is decreased when there is a deficiency in renal vitamin D metabolism

A 38-year-old male presents for routine wellness examination. Vital signs reveal a temperature 36.9oC (98.4oF), blood pressure of 156/92 mmHg, heart rate of 82/min, respiratory rate 16/min, and oxygen saturation 97%. History reveals no previous diagnosis of hypertension. Physical exam is unremarkable. Laboratory studies are obtained and reveal the following: Sodium 148 mmol/L Potassium 3.2 mmol/L Chloride 112 mmol/L CO2 24 mmol/L BUN 18 mg/dL Creatinine 0.8 mg/dL Glucose 89 mg/dL Urinalysis is obtained with normal results. The most appropriate pharmacologic therapy is A. aliskiren B. hydrochlorothiazide C.metyrapone D. phenoxybenzamine E. spironolactone

The correct answer is: E The most likely diagnosis is hyperaldosteronism. The elevated aldosterone level is an independent cause outside of the renin-angiotensin system. The elevated levels of aldosterone cause retention of sodium and potassium excretion. This sodium re-absorption causes retention of volume and subsequently causes hypertension. Patients may have mild, moderate, or even severe hypertension. Suspect hyperaldosteronism in patients that are not responsive to antihypertensive medications. Most patients are usually asymptomatic. They may present with muscle fatigue or weakness due to hypokalemia, often seen at levels of less than 3.1 mmol/L. This disease is caused by an adrenal tumor or adrenal hyperplasia. It is responsive to surgery if unilateral; however, if bilateral medical management is the preferential choice. The use of aldosterone antagonists are indicated (spironolactone or eplerenone). Aldosterone antagonists are contraindicated in renal failure and hyperkalemia. Answer A: Aliskiren is a direct renin inhibitor. It is used for hypertension in patients with diabetic nephropathy. It is contraindicated in pregnancy. It may increase the risk of angioedema. Avoid the use of aliskiren with ACE inhibitors and ARBs. Answer B: Thiazide diuretics are first line therapy in benign essential hypertension. Thiazide diuretics mechanism of action is at the distal convoluted tubule in the nephron. It inhibits sodium, chloride, and potassium reabsorption. It has common reactions of hypokalemia, hypercalcemia, and hyperuricemia. Caution is advised in diabetics, hyperuricemia, gout, dyslipidemia, and hypokalemia. Answer C: Metyrapone inhibits cortisol synthesis at the level of 11β-hydroxylase. It is indicated in the use of Cushing's syndrome. It is contraindicated in adrenocortical insufficiency. Caution is advised with the use of phenytoin, pregnancy/lactation, and may cause CNS depression. Answer D: Phenoxybenzamine is used for hypertension in pheochromocytoma. It is a non-selective alpha antagonist. It may cause orthostatic hypotension. Caution is advised in congestive heart failure and renal impairment. Bottom Line: In patients with hypertension and electrolyte abnormalities consistent with elevated sodium and decreased potassium think hyperaldosteronism. Primary aldosteronism is treated by blocking aldosterone with an antagonist such as spironolactone and may require surgery.

A 32-year-old male inmate is transferred from the state prison to the hospital for altered awareness with nausea and vomiting. Upon examination it is noted that the patient is unable to answer questions appropriately and has yellow coloring of the sclera. There is also significant tenderness in the right upper quadrant of the abdomen. Laboratory studies reveal the following: Aspartate aminotransferase 1543 IU/mL Alanine aminotransferase 1125 IU/mL Alkaline phosphatase 300 IU/mL Total bilirubin 6 mg/dL INR 1.4 HBsAg positive Anti-HBs negative Anti-HBc IgM positive HBeAg positive Anti-HBe negative Which of the following is the most appropriate next step in management? A. liver biopsy B. observation with liver enzyme monitoring C. pegylated interferon D. ribavirin E. tenofovir

The correct answer is: E The patient has an acute hepatitis B infection noted by the positive HBsAg and Anti-HBC IgM that is highly infective based on the positive HBeAg. Notice the detail of the inmate, this would help indicate high risk behavior for hepatitis B, sexual intercourse with multiple partners and intravenous drug abuse. The patient has rapid progression of symptoms with elevated liver enzymes in the thousands. This is concerning for fulminant hepatic failure. Clinical signs and symptoms are altered mental status and scleral icterus. The patient was displaying signs of serum sickness that started with nausea and vomiting that progressed to altered mental status. Laboratory data of elevated bilirubin levels and elevated INR are showing signs of liver failure. The patient needs to be admitted to the ICU for fulminant liver failure and started on tenofovir disoproxil fumarate for the acute hepatitis B to lower the viral count. If the patient does not show signs of improvement the patient would require a liver transplant. These patients need to be first placed on oral therapy such as tenofovir disoproxil fumarate which is a reverse transcriptase inhibitor. It may be used as retroviral therapy for HIV patients as well. Tenofovir inhibits replication of hepatitis B virus by inhibiting hepatitis b virus polymerase. Answer A: Liver biopsy is rarely indicated in acute viral hepatitis. Serologic tests would be indicated during the acute phase which are listed in the lab section in the question. A liver biopsy would be indicated if the diagnosis is uncertain or when the clinical evidence points towards a diagnosis of chronic hepatitis. Answer B: The patient has rapid progression of symptoms with elevated liver enzymes in the thousands. This is concerning for fulminant hepatic failure and should be treated. Hepatitis C on the other hand is typically monitored for 6 months for transition into the chronic state, for a good portion of acutely infected patients may spontaneously resolve without therapy. Answers C & D: Peginterferon is used to treat chronic hepatitis C in conjunction with ribavirin. This is done after an intense therapy course done with antiviral therapy with interferon alfa for an average of 3 months. Bottom Line: With acute hepatitis B infection start antiviral therapy with tenofovir disoproxil fumarate.

A 55-year-old male is brought to the emergency department for altered mental status after being found wandering the streets. History via the patient is unreliable as the patient does not follow directions and is displaying a disorganized thought process. A family member relates a long-standing history of alcohol abuse. Physical examination reveals that the patient is alert and oriented to person only. An eye examination reveals inability to abduct the eye bilaterally. The most likely deficient substance is A. cobalamin B. niacin C. pyridoxine D.riboflavin E. thiamine

The correct answer is: E This patient is presenting with confusion, ataxia, and a history of alcohol abuse, which is seen in Wernicke-Korsakoff syndrome. It results from a deficiency in thiamine (vitamin B1). Wernicke's encephalopathy results in necrosis of the mammillary bodies in the periventricular region of the brain. Symptoms include progressive dementia, confusion, ataxia, and paralysis of the extraocular muscles. This causes ophthalmoplegia of the bilateral lateral rectus muscles, which results in a sixth nerve palsy. Korsakoff psychosis is a thought disorder that results in retrograde memory failure and confabulation. The most common causes of thiamine deficiency include poor diet and chronic alcoholism. Answer A: Cobalamin, also known as vitamin B12, initially binds to intrinsic factor which is release by parietal cells in the stomach. It is then reabsorbed in the terminal ileum. Thus, vitamin B12 deficiency can be seen in patients with pernicious anemia, destruction of gastric mucosa from H. pylori infection, inadequate intake in those on vegan diets, celiac disease due to involvement of the terminal ileum, infection with Diphyllobothrium latum, or certain medications. Vitamin B12 deficiency results in subacute combined degeneration of the dorsal and lateral spinal columns. Peripheral neuropathy can result, which initially presents with decrease proprioception and vibration in the legs or arms. Patients may have a wide based gait, and can have memory loss or dementia. A beefy, red, sore tongue is also common. Answer B: A deficiency in niacin, also known as vitamin B3, results in pellagra. Pellagra is characterized by diarrhea, dementia, and a photosensitive dermatitis in sun-exposed areas of the body. Pellagra tends to occur in alcoholics, and has been reported as a complication of anorexia nervosa. Niacin deficiency can also be seen in carcinoid syndrome, in which there is an increased metabolism of tryptophan into serotonin, which leads to a deficiency of niacin. The most characteristic finding in pellagra is the presence of a symmetric hyperpigmented rash that is present in the exposed areas of skin. Other clinical findings are a red tongue, diarrhea, and vomiting. Neurologic symptoms include disorientation, delusions, dementia, and encephalopathy. Answer C: A deficiency in pyridoxine, also known as vitamin B6, results in stomatitis, glossitis, cheilosis, irritability, confusion, and depression. Physical examination would reveal inflammation and fissuring of the lips, cheilosis occurring at the corners of the mouth, atrophy of the mucosa of the tongue, and seborrheic dermatitis. Pyridoxine is a cofactor in pyridoxal phosphate mediated enzymatic reactions. Pyridoxal phosphate is necessary for the synthesis of heme, as it serves as a coenzyme for the enzyme ALA synthase. Thus, vitamin B6 deficiency can result in a microcytic, hypochromic sideroblastic anemia with the presence of basophilic stippling seen on a peripheral blood smear. Pyridoxine deficiency may result from pregnancy or therapy with certain medications, such as the anti-tuberculosis drug isoniazid. Answer D: A deficiency of riboflavin, vitamin B2, is characterized by cheilosis, angular stomatitis, and glossitis. This can be seen on physical examination as cracked and red lips, inflammation of the lining of mouth and tongue, mouth ulcers, cracks at the corners of the mouth, and a sore throat. A deficiency of riboflavin may also cause dry and scaling skin, fluid accumulation in the mucous membranes of the mouth, and an iron-deficiency anemia. The eyes may also become sensitive to bright light. Additionally, patients may develop seborrheic dermatitis of the face or genitalia. Riboflavin deficiency may be noted in patients with anorexia nervosa or with malabsorptive syndromes such as celiac sprue. Bottom Line: A deficiency in thiamine is commonly seen in chronic alcoholics, and can result in Wernicke-Korsakoff syndrome. This is characterized by confusion, ataxia, nystagmus, confabulation, and necrosis of the mammillary bodies.

A 35-year-old male is brought to the emergency department by paramedics with mental status changes. The patient is oriented to person only and complains of abdominal pain. Laboratory testing reveals a high anion gap metabolic acidosis and a serum glucose of 700 mg/dL. The most likely electrolyte abnormality is A.hypercalcemia B. hyperkalemia C. hypernatremia D. hypokalemia E. hyponatremia

The correct answer is: E This patient is presenting with diabetic ketoacidosis (DKA). The most common electrolyte abnormality is hyperosmolar hyponatremia. This hyponatremia is not due to primary sodium loss but is due to the plasma dilutional effect of hyperglycemia. Therefore, the treatment of his hyponatremia should not be directed to the specific correction of sodium levels, but should be directed towards the correction of his hyperglycemia. Answer A: Calcium homeostasis plays little role in those with DKA or HHS. Answers B & D: Patients with DKA or HHS have low potassium secondary to urinary losses due to the osmotic diuresis of glucose and to the need to maintain electroneutrality as ketoacid anions are excreted. Despite these potassium losses, the serum potassium concentration is usually normal or, in one-third of patients, elevated on admission. It is important to note that insulin lowers the potassium concentration and may cause severe hypokalemia. Answer C: Most patients with DKA and HHS have mildly low sodium, however, those with HHS with serum glucose concentrations exceeding 1000 mg/dL may have normal or elevated serum sodium concentrations. These patients are extremely hyperosmolar and often have neurologic symptoms that can include seizures and coma. Bottom Line: Hyperglycemia causes hyperosmolar hyponatremia via plasma dilution of sodium concentration.

A 32-year-old male presents to the emergency department with acute onset of bilateral lower extremity tingling and weakness. History reveals he had a sudden onset of sharp low back pain last night. Later a tingling sensation began this morning in both legs with associated "heaviness" when he walks. He has an episode of urinary incontinence in the emergency room, which he says has never happened to him before. He denies any recent trauma to his back or lower extremities. He has not significant health issues and the only medication he is taking is acetaminophen for a recent influenza infection he is still recovering from. Physical examination reveals 3/5 muscle strength in the lower extremities bilaterally, absent ankle jerk reflex, and +1 patellar reflex. Light touch on any skin surface below L3 elicits extreme discomfort. The most likely diagnosis is A. complex regional pain syndrome B. Guillain-Barré syndrome C. myasthenia gravis D.poliomyelitis E. transverse myelitis

The correct answer is: E This patient is presenting with signs and symptoms consistent with transverse myelitis. Transverse myelitis is an inflammatory process that involves both sides of a spinal cord segment and causes damage to the myelin sheath, leading to altered nerve conduction. The exact cause is unknown but it is thought to be caused by an autoimmune reaction triggered by a recent viral or bacterial infection. Infections it has been associated with include syphilis, measles, Lyme disease, varicella zoster, herpes simplex, cytomegalovirus, Epstein-Barr, influenza, echovirus, human immunodeficiency virus (HIV), hepatitis A, and rubella. Transverse myelitis usually presents during recovery from the infection. Symptom onset may be acute (within hours to days) or subacute (within days to weeks). Classic clinical findings include localized low back pain, sudden paresthesias in the legs, sensory loss, paraparesis (weakness or partial paralysis) and urinary bladder and bowel dysfunction. Approximately 80% of patients experience allodynia, characterized by increased sensitivity to light touch. Diagnostic work-up usually includes spine and brain MRI, lumbar puncture and laboratory studies to rule out other etiologies including multiple sclerosis, spinal cord compression (due to tumor, disc herniation, stenosis, abscesses), and infectious myelitis. Spinal MRI often can detect the inflammatory lesion, which is usually localized to one or more vertebral segments. Management of transverse myelitis involves reducing spinal cord inflammation and alleviating associated symptoms. Patients are usually treated with intravenous corticosteroids. Severe cases may require plasma exchange. Physical rehabilitation is critical once patients begin to show neurological improvement. Prognosis varies greatly. Approximately one-third of patients show complete recovery, one-third is left with mild to moderate permanent deficits (e.g., spastic gait, incontinence, sensory loss), and one-third has no recovery and remains wheel chair or bed bound. Answer A: Complex regional pain syndrome (CRPS), formerly known as reflex sympathetic dystrophy, presents with autonomic dysfunction and usually only affects one extremity. Symptoms may include dyshidrosis, color or temperature changes, and marked hyperesthesia. Patients do not experience paralysis. Answer B: Guillain-Barré syndrome (GBS) is also a post-infectious polyneuropathy that causes demyelination and neurological deficit. It is most commonly associated with upper respiratory infections or gastroenteritis (usually caused by Campylobacter species). GBS is usually a purely motor deficit and does not cause altered sensation. Also, it is a progressive process that ascends to involve a large portion of the spinal cord, including cranial nerves and nerves that innervate the diaphragm. Transverse myelitis involves a specific segment of the spinal cord and symptoms are only present below the level of the lesion. Answer C: Myasthenia gravis is characterized by weakness of muscles innervated by the brainstem, causing difficulty chewing, ptosis, and proximal limb weakness. Symptoms usually worsen with increased muscle use. It is caused by antibodies that block post-synaptic acetylcholine receptors. It does not cause acute lower extremity paralysis and back pain. Answer D: Poliomyelitis is an infectious myelitis caused by poliovirus. Infection with poliovirus is very rare today due to vaccination; however, it used to be a common cause of aseptic meningitis in developing nations. Most patients would recover without sequelae, but a small percentage would develop a more severe illness characterized by myalgia, loss of reflexes, and flaccid paralysis of proximal limb muscles. Bottom Line: Transverse myelitis is caused by spinal cord inflammation within a specific region of the spinal cord triggered by a recent bacterial or viral infection. It is characterized by pain, sensory alteration, and motor dysfunction below the level of the lesion. Treatment includes intravenous corticosteroids to reduce inflammation followed by aggressive physical therapy to improve neurological dysfunction.

A 40-year-old woman presents with constant diarrhea of 1 months' duration described as watery and non-bloody. She has been taking an over-the-counter antidiarrheal medication without relief. On review of systems, she mentions that she has been experiencing episodes of flushing and wheezing for the past 3 months. Vitals reveal a temperature of 37°C (98.6°F), pulse at 65/min, respirations at 16/min, and blood pressure of 142/76 mm Hg. Physical examination reveals her abdomen is non-tender and without peritoneal signs. Basic laboratory studies are shown below: Hemoglobin 14 g/dL Platelets 250,000/mm3 Aspartate aminotransferase 130 U/L Alanine aminotransferase 125 U/L Total bilirubin 0.8 mg/dL Alkaline phosphatase 400 U/L A urinalysis reveals elevated levels of 5-hydroxyindoleacetic acid. Which of the following vitamin deficiencies is this patient at risk of developing? A. calcium B. vitamin A C. vitamin B1 D. vitamin B2 E. vitamin B3

The correct answer is: E This patient's clinical presentation is consistent with carcinoid syndrome, which classically presents with abdominal pain, flushing, and diarrhea and is confirmed with an elevated urinary level of 5-HIAA. Carcinoid tumors are slow-growing tumors that arise from the appendix, small intestine, or rectum. Carcinoid syndrome only occurs if the tumor metastasizes, which occurs in 2% of cases with tumors > 2cm in size. In addition to flushing and diarrhea, carcinoid syndrome can also present with facial telangiectasias, hepatomegaly, and tricuspid and pulmonary valve stenosis. Carcinoid tumors synthesize, store and release different polypeptides, amines, and prostaglandins; the most common secretory product is serotonin. The serotonin by-product is the result of tryptophan's, an amino acid, conversion by the tumor. Tryptophan is enzymatically synthesized from vitamin B3 (niacin), which can lead to niacin deficiencies in patient's with carcinoid tumors. Niacin deficiency will usually manifest as pellagra, which consists of dermatitis, dementia, and diarrhea. The next step in management of this patient with an elevated 5-HIAA, a serotonergic precursor, level is localization of the tumor with a computed tomography (CT) scan of the abdomen or somatostatin receptor scintigraphy (SRS). Treatment includes surgery for isolated tumors or octreotide for tumors that have metastasized, as is likely in this patient presenting with carcinoid syndrome. Answer A: Women at the age of 40 would not be expected to have calcium deficiencies or osteoporosis. Calcium is not depleted by carcinoid tumors. Answer B: Vitamin A can result from terminal ileal transection, where fat-soluble vitamins are absorbed. It can also occur as a result of fat malabsorption, seen in conditions such as pancreatic insufficiency. Answer C: Vitamin B1 (thiamine) deficiency is common in alcoholics, leading to Wernicke Korsakoff syndrome. Answer D: Vitamin B2 (riboflavin) deficiency typically does not occur in the developed world. Bottom Line: Carcinoid syndrome manifests with diarrhea, flushing, wheezing, and heart murmurs as a result of a serotonin-producing tumor that converts niacin to tryptophan to serotonin. This can ultimately result in niacin (vitamin B3) deficiency.

A 63-year-old male presents to the emergency room complaining of weakness, fatigue and decreased appetite for one months' duration. Past medical history is significant for chronic obstructive lung disease, type 2 diabetes mellitus, and depression. Home medications include tiotropium bromide, metformin and sertraline. Physical examination reveals generalized weakness but is otherwise normal. Laboratory studies reveal the following: Glucose 194 mg/dL Sodium 118 mEq/L Potassium 4.8 mEq/L Chloride 99 mEq/L Bicarbonate 24 mEq/L Blood urea nitrogen 4.0 mg/dL Creatinine 0.5 mg/dL Serum osmolality 207 mOsm/kg Urine osmolality 124 mOsm/kg Urine sodium 46 mmol/L The most likely underlying cause of his hyponatremia is A. adrenal insufficiency B. hyperglycemia C. pseudohyponatremia D. renal tubular acidosis type 4 E. syndrome of inappropriate ADH

The correct answer is: E This patient's hyponatremia is likely secondary to syndrome of inappropriate ADH (SIADH). SIADH is caused by the non non-physiologic release of ADH from the posterior pituitary or an ectopic source resulting in impaired renal free-water excretion without deregulation of sodium balance causing an isovolemic hypotonic hyponatremia. It is a diagnosis of exclusion and all other potential causes of hyponatremia must be ruled out. SIADH will result in a low serum osmolality (< 280) and high urine osmolality (>100) secondary to fluid retention and volume expansion. However, despite volume expansion, edema is not seen in SIADH due to increased atrial natriuretic peptide and decreased proximal tubular sodium absorption leading to appropriate sodium excretion (urine Na>20). Other lab findings that support the diagnosis include low serum uric acid levels, low BUN, normal or slightly decreased creatinine (secondary to dilution), and normal thyroid and adrenal function. The list of underlying causes of SIADH is very long. It is associated with certain cancers (lung, pancreas, prostate, bladder, lymphoma, leukemia), CNS disorders (stroke, head trauma, meningitis, encephalitis), pulmonary disease (COPD, TB, pneumonia), medications (morphine, ant-neoplastic agents, SSRIs, oxytocin), and the postoperative state (severe pain and nausea are triggers for ADH release). This patient's history of COPD and use of sertraline both placed him at risk for developing SIADH. Patients usually present with symptoms of chronic hyponatremia including anorexia, nausea, weakness, and lethargy. Treatment involves slowly correcting the sodium (no more than 0.5 mEq/L/hr) with water restriction. Slow correction should be implemented in anyone with hyponatremia >72 hours to avoid central pontine myelinolysis. Answers A & D: Adrenal insufficiency and renal tubular acidosis type 4 results in hyponatremia by a similar mechanism, decreased aldosterone function. Aldosterone is a steroid hormone responsible for regulating blood pressure by acting on the distal tubule and collecting duct of nephrons to stimulate sodium and water reabsorption and increase potassium excretion. Decreased aldosterone production (adrenal insufficiency) and/or increased renal resistance to aldosterone (RTA type 4) leads to excess free-water and sodium excretion with decreased potassium excretion leading to a hypovolemic hypotonic hyponatremia and hyperkalemia. There was no evidence of hypovolemia mentioned on this patient's physical examination and his potassium was within normal range. Answer B: Hyperglycemia results in a hypertonic (serum osmolality>295) hyponatremia by causing an osmotic shift of water of out cells leading to a dilutional hyponatremia. For every 100mg/dL increase in blood glucose level above normal, the serum sodium level should be expected to drop 3 mEq/L. This patient's glucose was elevated, however, not enough to cause such a drastic decrease in the sodium level. Answer C: Pseudohyponatremia is used to describe an isotonic (serum osmolality = 280-295) hyponatremia that results from increased plasma solids leading to lower plasma sodium concentrations, however the actual amount of serum sodium is normal. This can be seen in hyperlipidemia, hyperproteinemia, and hyperglycemia. These patients are asymptomatic and the sodium will correct once the underlying condition is appropriately treated. Bottom Line: Causes of hyponatremia can be classified into isotonic, hypotonic, and hypertonic based on the serum osmolality. Hypotonic hyponatremia can then further be divided into hypovolemic, isovolemic, or hypervolemic based on the patient's clinical volume status. Syndrome of inappropriate ADH is the most common cause of isovolemic hypotonic hyponatremia. It should be considered in any patient who presents with chronic hyponatremia without edema, decreased arterial volume or renal insufficiency.


संबंधित स्टडी सेट्स

Code, Standards, and Practices 1

View Set

Unit 5 - Care of Family Ch. 17, Unit 5 - Care of Family Ch. 18, Unit 5 - Care of Family Ch. 19

View Set

Themes in the Poetry of Keats, The Enlightmnt and Romanticism...

View Set

Trigonometry - Solving Trig Equations (° and π)

View Set